ചരിത്രം

//ചരിത്രം
വിമർശനം1: അന്ത്യ വേളയെ സംബന്ധിച്ച് ചില ഗ്രാമീണർ മുഹമ്മദ് നബിയോട് നിരന്തരം ചോദിച്ചു കൊണ്ടിരുന്നപ്പോൾ അവരിലെ ഏറ്റവും പ്രായം കുറഞ്ഞ കുട്ടിയെ ചൂണ്ടി അദ്ദേഹം ഇപ്രകാരം പറയുകയുണ്ടായി: “ഈ പയ്യൻ ജീവിക്കുകയാണെങ്കിൽ ഇവന് വാർധക്യം പ്രാപിക്കുമ്പോഴേക്കും അന്ത്യം സംഭവിചേക്കാം” (സ്വഹീഹു മുസ്‌ലിം: 7276) ആ ‘പയ്യൻ’ മരണപ്പെട്ട് നൂറ്റാണ്ടുകൾ കഴിഞ്ഞിട്ടും ലോകാവസാനം സംഭവിക്കാതിരുന്നത് മുഹമ്മദ് നബിയുടെ പ്രവചനം പരാജയപ്പെട്ടുവെന്നതിന് തെളിവല്ലെ ? മറുപടി: a) മനുഷ്യർ ആസകലം ഈ ലോകത്തു നിന്ന് ഉച്ചാടണം ചെയ്യപ്പെടുന്ന, ജൈവലോകം സമ്പൂർണമായി ഉന്മൂലനം ചെയ്യപ്പെടുന്ന, ഭൗതീക പ്രപഞ്ചത്തിന്റെ അന്തർദ്ധാനത്തെ കുറിക്കുന്ന ‘അസ്സാഅ: അൽഉള്മാ’ (മഹാ അന്ത്യം) എന്നറിയപ്പെടുന്ന ‘ലോകാവസാന’ത്തെ സംബന്ധിച്ച അറിവ്, ദൈവം മാത്രമാണ് ഉടമപ്പെടുത്തുന്നത്. മുഹമ്മദ് നബിയടക്കം (സ) സൃഷ്ടികളിൽ ഒരാളുമായി ഈ അഭൗതീക ജ്ഞാനം ദൈവം പങ്കു വെച്ചിട്ടില്ല എന്ന് ക്വുർആനും മുഹമ്മദ് നബിയും (സ) അടിക്കടി ആവർത്തിച്ചു ഉറപ്പിക്കുന്നുണ്ട്: “തീര്‍ച്ചയായും അല്ലാഹുവിന്‍റെ പക്കലാണ് അന്ത്യസമയത്തെപ്പറ്റിയുള്ള അറിവ്‌..” (ക്വുർആൻ: 31:34) “അന്ത്യസമയത്തെപ്പറ്റി അവര്‍ നിന്നോട് ചോദിക്കുന്നു; അതെപ്പോഴാണ് വന്നെത്തുന്നതെന്ന്‌. പറയുക (നബിയെ): അതിനെപ്പറ്റിയുള്ള അറിവ് എന്‍റെ രക്ഷിതാവിങ്കല്‍ മാത്രമാണ്‌. അതിന്‍റെ സമയത്ത് അത് വെളിപ്പെടുത്തുന്നത് അവന്‍ മാത്രമാകുന്നു. ആകാശങ്ങളിലും ഭൂമിയിലും അത് ഭാരിച്ചതായിരിക്കുന്നു. പെട്ടെന്നല്ലാതെ അത് നിങ്ങള്‍ക്കു വരുകയില്ല. നീ അതിനെപ്പറ്റി ചുഴിഞ്ഞന്വേഷിച്ചു മനസ്സിലാക്കിയവനാണെന്ന മട്ടില്‍ നിന്നോടവര്‍ ചോദിക്കുന്നു. പറയുക: അതിനെപ്പറ്റിയുള്ള അറിവ് അല്ലാഹുവിങ്കല്‍ മാത്രമാണ്‌. പക്ഷെ അധികമാളുകളും (കാര്യം) മനസ്സിലാക്കുന്നില്ല.” (ക്വുർആൻ: 7:187) “ജനങ്ങള്‍ അന്ത്യസമയത്തെപ്പറ്റി നിന്നോട് ചോദിക്കുന്നു. പറയുക (നബിയെ): അതിനെപ്പറ്റിയുള്ള അറിവ് അല്ലാഹുവിങ്കല്‍ മാത്രമാകുന്നു. നിനക്ക് (അതിനെപ്പറ്റി) അറിവുനല്‍കുന്ന എന്തൊന്നാണുള്ളത്‌? അന്ത്യസമയം ഒരു വേള സമീപസ്ഥമായിരിക്കാം.” (ക്വുർആൻ: 33:63) متى الساعة؟ فقال: ما المسئول عنها بأعلم من السائل ലോകാന്ത്യം എപ്പോഴാണ് എന്ന് ചോദിച്ച മലക്ക് ജിബ്‌രീലിനോട് പ്രവാചകൻ (സ) പറഞ്ഞ മറുപടി: “ചോദിക്കപ്പെട്ടവൻ ചോദ്യകർത്താവിനേക്കാൾ അതിനെ സംബന്ധിച്ച് അറിവുള്ളവനല്ല.” (സ്വഹീഹുൽ ബുഖാരി: 50, സ്വഹീഹു മുസ്‌ലിം: 99) ലോകാവസാനത്തെ സംബന്ധിച്ച അറിവ് തനിക്കില്ലെന്ന് ആവർത്തിച്ചാവർത്തിച്ച് പ്രഖ്യാപിക്കുന്ന മുഹമ്മദ് നബി (സ) തന്നെ അതിന്റെ സമയവും കാലവും ക്ലിപ്തമായി പ്രസ്ഥാവിച്ചു എന്ന് വാദിക്കുന്നതിലെ യുക്തിരാഹിത്യം സുവ്യക്തമാണ്. b) തനിക്ക് ശേഷം നൂറ്റാണ്ടുകൾ തന്റെ സമുദായം ജീവിക്കുമെന്ന് പ്രവാചകൻ (സ) തന്നെ വ്യക്തമാക്കിയതായി ഒട്ടനവധി ഹദീസുകൾ സ്വഹീഹുൽ ബുഖാരിയിലും സ്വഹീഹു മുസ്‌ലിമിലുമൊക്കെ ഉണ്ടെങ്കിലും അതിന്നും കാണാത്ത മട്ടിലാണ് വിമർശകർ. ഒരു ഉദാഹരണം കാണുക: خَيْرُ النَّاسِ قَرْنِي، ثُمَّ الَّذِينَ يَلُونَهُمْ، ثُمَّ الَّذِينَ يَلُونَهُمْ، ثُمَّ يَجِيءُ أَقْوَامٌ تَسْبِقُ شَهَادَةُ أَحَدِهِمْ يَمِينَهُ، وَيَمِينُهُ شَهَادَتَهُ “മനുഷ്യരിൽ ഏറ്റവും ഉത്തമർ എന്റെ തലമുറയാണ്, ശേഷം അവരുടെ പിന്നിൽ വരുന്ന തലമുറ, ശേഷം അവരുടെ പിന്നിൽ വരുന്ന തലമുറ. പിന്നീട് ചില ജനതകൾ വരും, അവരിൽ ഒരുവന്റെ സാക്ഷ്യത്തെ അവന്റെ ശപഥം മുൻകടക്കും…” (സ്വഹീഹുൽ ബുഖാരി: 2652, സ്വഹീഹു മുസ്‌ലിം: 2533) ഈ ഹദീസിനെ ‘ക്വർന്’ (قَرْن) എന്ന പദത്തിന് തലമുറ എന്നും നൂറ്റാണ്ട് എന്നും അർത്ഥമുണ്ട്. അപ്പോൾ മുഹമ്മദ് നബിക്ക്(സ) ശേഷം ചുരുങ്ങിയത് മൂന്നും അതിലധികവും തലമുറകൾ അല്ലെങ്കിൽ നൂറ്റാണ്ടുകൾ മനുഷ്യ ജീവിതം ദീർഘിപ്പിക്കപ്പെടുമെന്ന് അദ്ദേഹം തന്നെ വ്യക്തമാക്കി കഴിഞ്ഞു. പിന്നെ എങ്ങനെയാണ് ഒരു തലമുറയോടെ ‘ലോകാവസാനം’ സംഭവിക്കുമെന്ന് അദ്ദേഹം പറഞ്ഞതായി (ദുർ)വ്യാഖ്യാനിക്കപ്പെടുക ?! c) വിമർശന വിധേയമായ ഹദീസിലേക്ക് തന്നെ വരാം: അന്ത്യ വേളയെ സംബന്ധിച്ച് നിരന്തരമായി ചോദിച്ചു കൊണ്ടിരുന്നവരോട് അവരിലെ ഏറ്റവും പ്രായം കുറഞ്ഞ കുട്ടിയെ ചൂണ്ടി “ഈ പയ്യൻ ജീവിക്കുകയാണെങ്കിൽ ഇവന് വാർധക്യം പ്രാപിക്കുമ്പോഴേക്കും അന്ത്യം സംഭവിചേക്കാം” എന്ന് പ്രവാചകൻ പറഞ്ഞു എന്ന് പ്രവാചകാനുചരൻ അനസ് ബിൻ മാലിക് (റ) പറഞ്ഞ സംഭവം (സ്വഹീഹു മുസ്‌ലിം: 7276) വിശദവും വ്യക്തവുമായ രൂപത്തിൽ പ്രവാചകപത്നി ആഇശ (റ) ഉദ്ധരിക്കുന്നുണ്ട്. ആ നിവേദനവും സ്വഹീഹുൽ ബുഖാരിയിലും സ്വഹീഹു മുസ്‌ലിമിലും എഴുതപ്പെട്ടിട്ടുണ്ട്. പക്ഷെ അവ കൂടി ചേർത്ത് വെച്ച് അവതരിപ്പിച്ചാൽ ഹദീസിനെ ദുർവ്യാഖ്യാനിക്കാൻ കഴിയില്ല എന്നതുകൊണ്ട് സൗകര്യപൂർവ്വം ഒഴിവാക്കുകയാണ് ഇസ്‌ലാം വിമർശകരുടെ പതിവ്. ﻋﻦ ﻋﺎﺋﺸﺔ، ﻗﺎﻟﺖ: ﻛﺎﻥ ﺭﺟﺎﻝ ﻣﻦ اﻷﻋﺮاﺏ ﺟﻔﺎﺓ، ﻳﺄﺗﻮﻥ اﻟﻨﺒﻲ ﺻﻠﻰ اﻟﻠﻪ ﻋﻠﻴﻪ ﻭﺳﻠﻢ ﻓﻴﺴﺄﻟﻮﻧﻪ: ﻣﺘﻰ اﻟﺴﺎﻋﺔ؟ ﻓﻜﺎﻥ ﻳﻨﻈﺮ ﺇﻟﻰ ﺃﺻﻐﺮﻫﻢ ﻓﻴﻘﻮﻝ: «ﺇﻥ ﻳﻌﺶ ﻫﺬا ﻻ ﻳﺪﺭﻛﻪ اﻟﻬﺮﻡ ﺣﺘﻰ ﺗﻘﻮﻡ ﻋﻠﻴﻜﻢ ﺳﺎﻋﺘﻜﻢ»، ﻗﺎﻝ ﻫﺸﺎﻡ: ﻳﻌﻨﻲ ﻣﻮﺗﻬﻢ ആഇശ (റ) പറഞ്ഞു: അപരിഷ്കൃതരായ ഗ്രാമീണ അറബികളിൽ ചിലർ പ്രവാചകന്റെ(സ) അടുത്തു വന്ന് “എന്നാണ് അന്ത്യ സമയം ?” എന്ന് ചോദിക്കുമായിരുന്നു. അപ്പോൾ അവരിലെ ഏറ്റവും പ്രായം കുറഞ്ഞവനെ നോക്കി കൊണ്ട് അദ്ദേഹം ഇപ്രകാരം പറയും: “ഈ പയ്യൻ ജീവിക്കുകയാണെങ്കിൽ ഇവന് വാർധക്യം പ്രാപിക്കുമ്പോഴേക്കും നിങ്ങളുടെ അന്ത്യ സമയം സംഭവിക്കും” ഹിശാം പറഞ്ഞു: അഥവാ അവരുടെ മരണം. (സ്വഹീഹുൽ ബുഖാരി: 6511) ﺇﻥ ﻳﻌﺶ ﻫﺬا، ﻟﻢ ﻳﺪﺭﻛﻪ اﻟﻬﺮﻡ، ﻗﺎﻣﺖ ﻋﻠﻴﻜﻢ ﺳﺎﻋﺘﻜﻢ “ഇവൻ ജീവിക്കുകയാണെങ്കിൽ നിങ്ങളുടെ അന്ത്യ സമയം നിങ്ങളുടെ മേൽ സംഭവിച്ചിട്ടല്ലാതെ ഇവന് വാർധക്യം പ്രാപിക്കുകയില്ല.” (സ്വഹീഹുൽ മുസ്‌ലിം: 136) ദൈവത്തിനല്ലാതെ സൃഷ്ടികളിൽ മറ്റാർക്കും അറിവ് നൽകപ്പെട്ടിട്ടില്ലാത്ത മനുഷ്യരാശിയുടെ ‘അന്ത്യനാളിനെ’ സംബന്ധിച്ച് ആവശ്യത്തിനുപരി ചിന്തിച്ചും ചോദിച്ചും സമയവും ഊർജവും പാഴാക്കുന്ന ഗ്രാമീണരോട്, ‘അവരുടെ അന്ത്യ സമയത്തെ’ (അഥവാ മരണത്തെയും ലോകത്തു നിന്നുമുള്ള വിയോഗത്തെയും) സംബന്ധിച്ച് ചിന്തിക്കാൻ വഴി കാട്ടുകയാണ് പ്രവാചകൻ (സ) ചെയ്തത്. അല്ലാതെ ലോകാവസാനത്തിന്റെ തിയ്യതി കുറിച്ചു കൊടുക്കുകയല്ല. (ﻗﺎﻣﺖ ﻋﻠﻴﻜﻢ ﺳﺎﻋﺘﻜﻢ) “നിങ്ങളുടെ അന്ത്യ സമയം നിങ്ങളുടെ മേൽ സംഭവിക്കും” എന്ന് പ്രവാചകൻ (സ) പറഞ്ഞതായി ഹദീസിന്റെ വിശദരൂപത്തിൽ തന്നെ വ്യക്തമാക്കപ്പെട്ടല്ലൊ. കൂടാതെ ഹദീസ് ഉദ്ധരിക്കുന്ന റാവി ഹിശാം ഇബ്നു ഉർവ്വ (ജനനം: 61 ഹിജ്റാബ്ദം) തന്നെ “നിങ്ങളുടെ അന്ത്യ സമയം” എന്നതുകൊണ്ട് പ്രവാചകൻ (സ) ഉദ്ദേശിച്ചത് അദ്ദേഹത്തിന്റെ (സ) സംബോധിതരായ ആളുടെ മരണമാണെന്നും ഹദീസിൽ തന്നെ നാം കണ്ടു. ഹദീസിൽ തന്നെ പ്രവാചക വാചകത്തിൽ ഉദ്ദേശിക്കപ്പെട്ട ‘അസ്സാഅ:’യുടെ (അന്ത്യം) അർത്ഥം വ്യക്തമാണ് എന്നിരിക്കെ ഹദീസിന്റെ ഏറ്റവും സംഗ്രഹരൂപത്തിലുളള നിവേദനം മാത്രം എടുത്തു വെച്ച് ‘അസ്സാഅ:’ക്ക് ‘ലോകാവസാനം’ എന്ന് അർത്ഥം നൽകുന്നത് ദുരുദ്ദേശ്യപരമാണ്. ഇത് ഹദീസിന്റെ പൂർണ രൂപത്തിനും അറബി ഭാഷക്കും എല്ലാം എതിരാണ്. പൗരാണി അറബികൾ പ്രസ്ഥുത പദത്തെ ഉപയോഗിച്ചിരുന്ന അർത്ഥവ്യാപ്തിയെ സംബന്ധിച്ച് യാതൊരു വിവരവുമില്ലാത്തവർക്കു മാത്രമെ ഈ തെറ്റിദ്ധാരണ സംഭവിക്കു. ‘അസ്സാഅ:’ (الساعة) എന്ന പദത്തിന്റെ ഭാഷാർത്ഥം സമയം, മണിക്കൂർ, മണി (O’clock), വാച്ച്, ക്ലോക്ക് എന്നൊക്കെയാണ് ആധുനിക ഡിക്ഷ്ണറികളിൽ കാണുക. ക്വുർആനിലും ഹദീസിലും സാങ്കേതികമായി ‘അന്ത്യ സമയത്തെ’ കുറിക്കാൻ ഈ പദം ധാരാളമായി പ്രയോഗിച്ചിട്ടുണ്ട്. അന്ത്യ സമയം, അന്ത്യ ഘട്ടം, അന്ത്യ മണിക്കൂർ, അന്ത്യദിനം എന്നിങ്ങനെയൊക്കെ സാങ്കേതിക പദമായി ഉപയോഗിക്കപ്പെടുമ്പോൾ ‘അസ്സാഅ:’ക്ക് അർത്ഥം നൽകപ്പെടാറുണ്ട്. ലോകാവസാനത്തിന് പുറമെ ഒരു വ്യക്തിയുടെയൊ സംഘത്തിന്റെയൊ മരണത്തെയും, ഒരു തലമുറയുടെയൊ ജനതയുടെയൊ തിരോധാനത്തെയും, ഒരു നൂറ്റാണ്ടിന്റെയൊ സമുദായത്തിന്റെയൊ അവസാനത്തെയും എല്ലാം ‘അസ്സാഅ:’ അഥവാ ‘അന്ത്യ ഘട്ടം’ എന്ന് അറബി ഭാഷയിൽ പ്രയോഗിക്കപ്പെടും. ലോകാവസാനം, ജീവിതാവസാനം, തലമുറയുടെ അവസാനം എന്നൊക്കെയുളള ഉദ്ദേശ്യാർത്ഥം ഓരോ സാഹചര്യത്തിനനുസരിച്ച് ‘അസ്സാഅ:’ (അന്ത്യ സമയം) എന്ന പദം പ്രയോഗിക്കപ്പെടും. ഇബ്നു തീമിയ പറഞ്ഞു: “‘അസ്സാഅ:’ (അന്ത്യ വേള) എന്നതുകൊണ്ട് പ്രവാചകൻ (സ) ഉദ്ദേശിച്ചത് ഒരു തലമുറയുടെ അന്ത്യമാണ് എന്ന് സ്വഹീഹുൽ ബുഖാരിയിലൂടെ സ്ഥാപിതമായ കാര്യമാണ്… മനുഷ്യരെല്ലാം തങ്ങളുടെ ശ്മശാനങ്ങളിൽ നിന്ന് ദൈവ സന്നിധിയിലേക്ക് ഉയിർത്തെഴുന്നേൽക്കപ്പെടുന്ന ലോകാവസാനമല്ല ഇവിടെ ഉദ്ദേശിക്കപ്പെട്ടിരിക്കുന്നത്. ഈ പയ്യൻ വയസ്സാവുമ്പോഴേക്കും സംഭവിക്കുമെന്ന് പറഞ്ഞ ‘അസ്സാഅ:’ ശ്രോദ്ധാക്കളായ മനുഷ്യരുടെ മരണവും പ്രസ്ഥുത തലമുറയുടെ തിരോധാനവുമാണ്. പ്രവാചക ശിഷ്യൻ മുഗീറത്തിബ്നു ശുഅ്ബ പറഞ്ഞത് ഇതിന് തെളിവാണ്: ജനങ്ങളെ, നിങ്ങൾ ‘ക്വിയാമ:’ (ഉയിർത്തെഴുന്നേൽപ്പ്) എന്ന് വിളിക്കുന്നത് (സർവ്വ മനുഷ്യരും ദൈവസന്നിധിയിൽ ഹാജരാക്കപ്പെടുന്ന മഹാ) ഉയിർത്തെഴുന്നേൽപ്പിനെയാണ്. എന്നാൽ ആർ മരണപ്പെട്ടുവോ അവന്റെ ക്വിയാമത്ത് (ഉയിർത്തെഴുന്നേൽപ്പ്) സംഭവിച്ചു കഴിഞ്ഞു. (അൽ ഇസ്തികാമ: 1: 67) ഇമാം നവവി എഴുതി: “കാദി ഇയാദ് (ജനനം ഹിജ്റാബ്ദം:476) പറഞ്ഞു: ‘നിങ്ങളുടെ സാഅ:’ (അന്ത്യ വേള) എന്ന് പ്രവാചകൻ (സ) പറഞ്ഞതിന്റെ അർത്ഥം ശ്രോദ്ധാക്കളായ തലമുറയുടെ മരണവും വിയോഗവുമാണ്. പ്രസ്ഥുത അഭിസംബോധകർ അല്ലെങ്കിൽ ആ തലമുറയിൽ പെട്ടവരുടെ അന്ത്യമാണ് ഉദ്ദേശ്യം.” (ശർഹു മുസ്‌ലിം : 18:90) ‘അസ്സാഅ: അൽഉള്മാ’ ( الساعة العظمى മഹാ അന്ത്യം) എന്നറിയപ്പെടുന്ന ‘ലോകാവസാന’ത്തെ സംബന്ധിച്ച, ദൈവത്തിന് മാത്രം അറിവുള്ള ഒരു കാര്യത്തെ സംബന്ധിച്ച് ആവർത്തിച്ച് ചോദിച്ച് സമയവും ജീവിതാവസരങ്ങളും നഷ്ടപ്പെടുത്തരുത്. കാരണം നിങ്ങളുടെ ഓരോരുത്തരുടെയും ‘അസ്സാഅ:’ (അന്ത്യ വേള) ഏകദേശം നൂറു വർഷത്തിനിടയിൽ സംഭവിക്കും. അതിനാൽ സ്വന്തം അവസാന വേളയെ സംബന്ധിച്ച് ചിന്തിച്ച് അതിനായി ഒരുങ്ങുകയും സൽകർമ്മങ്ങളിൽ നിരതരാവുകയും ചെയ്യുക എന്നാണ് ഹദീസിലൂടെ പ്രവാചകൻ (സ) നൽകുന്ന പാഠം. ‘അസ്സാഅ: അൽഉള്മാ’ (മഹാ അന്ത്യം) എന്നറിയപ്പെടുന്ന ലോകാവസാനത്തെ സംബന്ധിച്ച ചോദ്യങ്ങളുടെ ധാരാളിത്വം പ്രവാചകന് (സ) പ്രിയമുള്ളതായിരുന്നില്ല എന്ന് ഹദീസുകളിൽ നിന്ന് തന്നെ വ്യക്തമാണ്. ഒരിക്കൽ അന്ത്യ വേളയെ സംബന്ധിച്ച് ആരാഞ്ഞ ഒരാളോട് പ്രവാചകൻ (സ), “കഷ്ടം ! ആ അവസരത്തിനായി നീ എന്താണ് (സൽകർമ്മങ്ങൾ) ഒരുക്കി വെച്ചിട്ടുള്ളത്?!” എന്ന് പ്രവാചകൻ (സ) തിരിച്ചു ചോദിക്കുകയാണുണ്ടായത്. (സ്വഹീഹുൽ ബുഖാരി:6167, സുനനു തുർമുദി: 2385) ഇബ്നു ഹജർ പറഞ്ഞു: “ഇസ്മാഈലി പറഞ്ഞു: ‘അസ്സാഅ:’ (അന്ത്യ വേള) എന്നത് കൊണ്ട് ഉദ്ദേശ്യം പ്രവാചകന്റെ അടുക്കൽ സന്നിഹിതരായവരുടെ അന്ത്യമാണ്. അഥവാ അവരുടെ മരണം. അവരുടെ മരണത്തെ ‘അസ്സാഅ:’ (അന്ത്യ വേള) എന്ന് പ്രയോഗിക്കപ്പെടുന്നതിന്റെ കാരണം മരണം പരലോക കാര്യങ്ങളിലേക്ക് അവരെ വഹിച്ചുകൊണ്ടു പോവുന്നു എന്നതാണ്. ‘അസ്സാഅ: അൽഉള്മാ’ (മഹാ അന്ത്യം) എന്നറിയപ്പെടുന്ന ലോകാവസാനത്തെ സംബന്ധിച്ച അറിവ് ദൈവത്തിന് മാത്രം അറിയുന്നതാണ് എന്ന് ഈ ഹദീസ് ഉറപ്പിക്കുന്നു. ക്വുർആനും ധാരാളം ഹദീസുകളും വ്യക്തമാക്കുന്നത് ലോകാവസാനത്തെ സംബന്ധിച്ച അറിവ് ദൈവത്തിന് മാത്രം അറിയുന്നതാണ് എന്ന വസ്തുതയാണ്… ‘അസ്സാഅ:'(അന്ത്യ വേള) എന്നതിനെ സംബന്ധിച്ച ഈ അർത്ഥാന്തരങ്ങൾ അറബികളുടെ അടുക്കൽ വളരെ പ്രചാരത്തിലുള്ളതാണ്…” (ഫത്ഹുൽ ബാരി:10:556) ഹദീസിന്റെ പൂർണരൂപമടങ്ങുന്ന ആഇശയുടെ നിവേദനത്തിന്റെ വെളിച്ചത്തിൽ ഇതേ വ്യാഖ്യാനം തന്നെ ‘അൽമസ്വാബീഹ്’, ‘അൽ മശാരിക്’ എന്നീ ഗ്രന്ഥങ്ങൾക്ക് വ്യാഖ്യാനം രചിച്ച പണ്ഡിതന്മാരും ഇമാം ദാവൂദിയും, കാദി ഇയാദും, ക്വുർതുബിയും എല്ലാം വിശദീകരിച്ചിട്ടുണ്ട്. (ഫത്ഹുൽ ബാരി: 10:572) ഇബ്നുകസീർ പറഞ്ഞു: “അനസിന്റെ(റ) നിവേദനത്തിലുള്ള പോലെ, ‘അസ്സാഅ:’ (അന്ത്യ വേള) എന്ന പദം നിരുപാധികമായി വന്ന നിവേദനങ്ങൾ, ആഇശയുടെ(റ) നിവേദനത്തിൽ സോപാധികമായി വന്ന “നിങ്ങളുടെ അന്ത്യ വേള” എന്ന പദപ്രയോഗത്തിലൂടെയാണ് വ്യാഖ്യാനിക്കപ്പെടേണ്ടത്.” (മആരിജുൽ കുബൂൽ: 2: 592) ‘അൽക്വിയാമ:’ (اﻟﻘﻴﺎﻣﺔ) അഥവാ ‘ഉയിർത്തെഴുന്നേൽപ്പ് ‘എന്ന സാങ്കേതിക പദത്തിന്റെ കാര്യം ഇപ്രകാരം തന്നെയാണ്. സർവ്വ മനുഷ്യരും ദൈവസന്നിധിയിൽ ഹാജരാക്കപ്പെടുന്ന ‘മഹാ ഉയിർത്തെഴുന്നേൽപ്പ്’ എന്നതിന് പുറമെ ഓരോ മനുഷ്യരുടെയും മരണത്തെ അവരവരുടെ ‘ഉയിർത്തെഴുന്നേൽപ്പ് ‘ എന്ന് ഇസ്‌ലാമിക പ്രമാണങ്ങൾ ധാരാളമായി വിളിച്ചിട്ടുണ്ട്. “ഓരോരുത്തരുടേയും ക്വിയാമത്ത് അഥവാ ‘ഉയിർത്തെഴുന്നേൽപ്പ് ‘ അവരവരുടെ മരണമാണ്… ഒരാളുടെ മൃതശരീരം മറമാടിയതിന് ശേഷം അൽക്വമ പറഞ്ഞു: എന്നാൽ ഇദ്ദേഹത്തിന്റെ ക്വിയാമത്ത് (ഉയിർത്തെഴുന്നേൽപ്പ്) സംഭവിച്ചു കഴിഞ്ഞു. (തഫ്സീറു ത്വബ്‌രി: 24:43) “ആർ മരണപ്പെട്ടുവോ അവന്റെ ക്വിയാമത്ത് (ഉയിർത്തെഴുന്നേൽപ്പ്) ആയി.” (ശർഹുസ്സുന്ന: ബഗ്‌വി: 10:97, തഖ്‌രീജുൽ അഹാദീസു വൽ ആസാർ: സൈല’ഇ: 1:436) d) ‘അന്ത്യ വേളയെ സംബന്ധിച്ചുള്ള അറിവ് അല്ലാഹുവിന്റെ അടുക്കൽ മാത്രമാണ് ഉള്ളത്. അല്ലാഹുവെ സാക്ഷിയാക്കി കൊണ്ട് ഞാൻ സത്യം ചെയ്യുന്നു. ഇപ്പോൾ ഭൂമിക്കു മുകളിൽ ശ്വസിക്കുന്ന ഒരാളുടെയും മേൽ നൂറ് വർഷത്തിനപ്പുറം കടന്നുപോകില്ല’ (സ്വഹീഹു മുസ്‌ലിം: 6363) എന്ന് മുഹമ്മദ് നബി (സ) പറഞ്ഞതായി വന്ന ഹദീസിലെയും ‘അസ്സാഅ:’ (അന്ത്യ വേള) എന്നത് ഈ അർത്ഥവ്യാപ്തിയെ ഉൾകൊണ്ടാണ് പ്രയോഗിച്ചിരിക്കുന്നത്. ‘അസ്സാഅ:’ (അന്ത്യ വേള) എന്നത് കൊണ്ട് ലോകത്തിന്റെ/ പ്രപഞ്ചത്തിന്റെ അവസാനമാണ് ഉദ്ദേശിക്കുന്നതെങ്കിൽ അത് എപ്പോഴാണ് സംഭവിക്കുക എന്നത് ദൈവത്തിന് മാത്രമെ അറിയു. തനിക്ക് ആ അറിവില്ല. ഇനി, ‘അസ്സാഅ:’ (അന്ത്യ വേള) എന്നത് കൊണ്ട് ഈ തലമുറയുടെ അവസാനമാണ് ഉദ്ദേശിക്കപ്പെടുന്നതെങ്കിൽ ഒരു നൂറ്റാണ്ടു കൊണ്ട് നിങ്ങൾ നിങ്ങളുടെ ‘അസ്സാഅ:’ (അന്ത്യ വേള)ക്ക് പാത്രീയരാവും എന്നാണ് മുകളിൽ ഉദ്ധരിക്കപ്പെട്ട ഹദീസിന്റെ അർത്ഥം. പ്രവാചകാനുചരൻ ഇബ്നു ഉമർ (റ) തന്നെ ഇത് വ്യക്തമാക്കുന്നുണ്ട്: ‘ഇന്ന് ഭൂമിക്കുമുകളിലുള്ള ആരും തന്നെ അവശേഷിക്കില്ല’ എന്ന് മാത്രമാണ് അല്ലാഹുവിന്റെ ദൂതൻ (സ) പറഞ്ഞത്. ( يريد بذلك أن ينخرم ذلك القرن) ഒരു തലമുറയുടെ അന്ത്യമാണ് അദ്ദേഹം ഈ പറഞ്ഞതു കൊണ്ട് ഉദ്ദേശിച്ചത്. (സ്വഹീഹു മുസ്‌ലിം : 6361) വിമർശനം2: രണ്ടു വിരലുകൾ ചേർത്തുപിടിച്ചു കൊണ്ട് താനും ലോകാന്ത്യവും ഇത്രയും അടുത്താണെന്ന് മുഹമ്മദ് നബി പറഞ്ഞിട്ട് പതിനാല് നൂറ്റാണ്ട് പിന്നിട്ടില്ലെ?! ഇത് പ്രവചനത്തിന്റെ പരാജയത്തിലേക്കല്ലെ വിരൽ ചൂണ്ടുന്നത്? മറുപടി: പ്രവാചകൻ (സ) തന്റെ നടുവിരലും ചൂണ്ടുവിരലും കാണിച്ചു കൊണ്ട് “ഞാൻ നിയോഗിക്കപ്പെട്ടതും അന്ത്യനാളും ഇപ്രകാരമായാണ്” എന്ന് പറഞ്ഞതായി ഹദീസിൽ വന്നിട്ടുണ്ട്. (സ്വഹീഹു മുസ്‌ലിം: 2951) ചൂണ്ടുവിരലിനും നടുവിരലിനുമിടയിൽ ഒരു വിരലില്ല എന്നത് പോലെ മുഹമ്മദ് നബിക്കും (സ) അന്ത്യനാളിനുമിടയിൽ പുതുതായി ഒരു പ്രവാചക നിയോഗമനം ഉണ്ടാവില്ല എന്നതാണ് ഹദീസിന്റെ അർത്ഥം. ഈ പ്രവചനം ഒരു നിലയിലും അസാധുവാക്കപ്പെട്ടിട്ടില്ല; ഭാവിയിൽ അസാധുവാക്കപ്പെടുകയുമില്ല. ഹദീസിനെ വ്യാഖ്യാനിച്ചു കൊണ്ട് ഇബ്നു ഹജർ ഇപ്രകാരം രേഖപ്പെടുത്തി: “കാദി ഇയാദും മറ്റും പറഞ്ഞു: മുഹമ്മദ് നബിയുടെ(സ) പ്രവാചകത്വവും ലോകാവസാനവും തമ്മിലുള്ള അടുപ്പമാണ് ഹദീസ് സൂചിപ്പിക്കുന്നത്. ഈ രണ്ട് കാര്യങ്ങൾക്കുമിടയിലെ ചുരുങ്ങിയ കാലയളവ് അല്ലെങ്കിൽ ഒന്നിന് ശേഷം വരുന്നതായ അടുത്ത കാര്യം എന്ന അർത്ഥത്തിൽ ഒക്കെയാണ് ഹദീസ്… ‘അന്ത്യനാളിനെ സംബന്ധിച്ച് ചോദിച്ചക്കപ്പെട്ടവന്, ചോദ്യകർത്താവിനേക്കാൾ അറിവില്ല’ എന്ന ഹദീസിന് ഈ ഹദീസ് എതിരല്ല. ഈ ഹദീസിന്റെ അർത്ഥം, ലോകാന്ത്യത്തിനും മുഹമ്മദ് നബിക്കും(സ) ഇടയിൽ പുതുതായി ഒരു നബിയും വരാനില്ല എന്നാണ്; ചൂണ്ടുവിരലിനും നടുവിരലിനും ഇടയിൽ മറ്റൊരു വിരലില്ല എന്നതു പോലെ… ദഹ്ഹാക് പറഞ്ഞു: ലോകാവസാനത്തിന്റെ അടയാളങ്ങളിൽ പെട്ടതാണ് മുഹമ്മദ് നബിയുടെ(സ) പ്രവാചക നിയോഗം. മതിമറന്നവരെ ഉൽബുദ്ധരാക്കുക അന്ത്യനാളിനായി തയ്യാറെടുപ്പിക്കുകയും പശ്ചാത്താപത്തിനായി പ്രോത്സാഹിപ്പിക്കുകയും ചെയ്യുക എന്നതാണ് ലോകാവസാനത്തിന് അടയാളങ്ങൾ നിയോഗിച്ചതിലെ യുക്തി.” (ഫത്ഹുൽ ബാരി: 11:357) ഹദീസിന് നൽകപ്പെട്ടിട്ടുള്ള മറ്റൊരു വ്യാഖ്യാനം, രണ്ടു വിരലുകൾക്കിടയിലുള്ള അടുപ്പം പോലെയാണ് മുഹമ്മദ് നബിയുടെ (സ) ആഗമനവും ലോകാന്ത്യവും തമ്മിലുള്ള കാല ചുരുക്കം എന്നതാണ്. ഈ വ്യാഖ്യാനവും പ്രവചനത്തെ അസാധുവാക്കുകയൊ ദുർബലപ്പെടുത്തുകയൊ ചെയ്യുന്നില്ല. കാരണം ലോകത്തിന്റെയും പ്രപഞ്ചത്തിന്റെയും മൊത്തം പ്രായവും പഴക്കവുമായി താരതമ്യം ചെയ്തു കൊണ്ടാണല്ലൊ മുഹമ്മദ് നബിയുടെയും (സ) ലോകാവസാനത്തിന്റെയും ഇടയിലുള്ള കാലയളവ് ചുരുക്കമൊ ദീർഘമൊ എന്ന് നിർണയിക്കാൻ. പ്രപഞ്ചത്തിന് ഏകദേശം 13.8 ബില്യൺ വർഷം പഴക്കമുണ്ടെന്നാണ് നിഗമിക്കപ്പെടുന്നത്. (https://www.google.com/amp/s/www.space.com/amp/universe-age-14-billion-years-old ഭൂമിയുടെ പ്രായം ഏകദേശം 4.54 ബില്യൺ വർഷങ്ങളായി കണക്കാക്കപ്പെടുന്നു, ഈ കണക്കിൽ നിന്ന് ഏകദേശം 50 ദശലക്ഷം വർഷങ്ങൾ കൂടിയൊ കുറഞ്ഞൊ വരാം. (https://www.nationalgeographic.org ) ഭൂമിയിൽ ജീവൻ ആരംഭിച്ചത് കുറഞ്ഞത് 3.7 ബില്യൺ വർഷങ്ങൾക്ക് മുമ്പാണെന്ന് നിഗമിക്കപ്പെടുന്നു. ആദ്യത്തെ മനുഷ്യ പൂർവ്വികർ പ്രത്യക്ഷപ്പെട്ടത് ഏഴ് ദശലക്ഷം മുതൽ നാലൊ രണ്ടൊ ദശലക്ഷം വർഷങ്ങൾക്ക് മുമ്പാണെന്നാണ് വാദിക്കപ്പെടുന്നത്. അപ്പോൾ ഇനിയും നൂറോ ആയിരമൊ വർഷങ്ങൾക്ക് ശേഷമാണ് പ്രപഞ്ചവും ലോകവും പ്രതിഭാസവ്യതിയാനമൊ ഉന്മൂലനാശമൊ പ്രാപിക്കുന്നതെങ്കിൽ പോലും പ്രപഞ്ചത്തിന്റെയൊ ഭൂമിയുടെയൊ ജൈവലോകത്തിന്റെയൊ മൊത്തം പ്രായത്തിന്റെയും കാലപഴക്കത്തിന്റെയും മുമ്പിൽ മുഹമ്മദ് നബിയുടെ (സ) ആഗമനവും ലോകാവസാനവും തമ്മിൽ വളരെ ചെറിയ കാല അകൽച്ച മാത്രമെ മനസ്സിലാക്കാൻ സാധിക്കു.

ന്ത്യൻ മുസ്‌ലിംകളുടെ ദേശക്കൂറും വിശ്വസ്തതയും ചോദ്യം ചെയ്തു കൊണ്ട് വർഗീയ ദ്രുവീകരണം സൃഷ്ടിച്ച് രാഷ്ട്രീയ നേട്ടങ്ങൾ കൊയ്യാൻ സംഘ് പരിവാറും കൂട്ടാളികളും അശ്രാന്ത പരിശ്രമത്തിലാണ്. ഇസ്‌ലാമിനെതിരെ നുണകളും അർദ്ധ സത്യങ്ങളും പ്രചരിപ്പിക്കുന്നതിനു പുറമെ ഇസ്‌ലാമിക പ്രമാണങ്ങളെ ദുർവ്യാഖ്യാനിച്ചും തെറ്റിദ്ധരിപ്പിച്ചും ജനമനസ്സുകളിൽ ഇസ്‌ലാം ഭീതി പടർത്താൻ നിരന്തരം പ്രയത്നിച്ചു കൊണ്ടിരിക്കുന്ന വെറുപ്പിന്റെ അപ്പോസ്തലന്മാർ ഏറ്റവുമൊടുവിൽ പുതിയ കുപ്പിയിലാക്കി ഇറക്കുമതി ചെയ്ത പഴയ വീഞ്ഞാണ് ‘ഗസ്‌വത്തുൽ ഹിന്ദ്’ (ഹിന്ദ് യുദ്ധം). ഇന്ത്യയോട് യുദ്ധം ചെയ്യാൻ പ്രവാചക കൽപ്പനയുണ്ടെന്നും ഏതു നിമിഷവും പൊട്ടാവുന്ന ബോംബാണ് മുസ്‌ലിംകളെന്നുമാണ് ദേശസ്നേഹികളും സമാധാന ചിത്തരുമായ ഇന്ത്യൻ മുസ്‌ലിംകളുടെ തലയിൽ വെച്ചുകെട്ടുന്ന പുതിയ ആരോപണം. ഈ അവസരത്തിൽ വാദപ്രതിവാദങ്ങളുടെ വൈകാരിക തലം ഒട്ടും സ്പർശിക്കാതെ, വിവാദ വിഷയകമായ ഹദീസിനെ സംബന്ധിച്ച ഒരു വൈചാരികമായ ചർച്ചയാണ് ഈ കുറിപ്പ് ലക്ഷ്യം വെക്കുന്നത്. ﻋﺼﺎﺑﺘﺎﻥ ﻣﻦ ﺃﻣﺘﻲ ﺃﺣﺮﺯﻫﻤﺎ اﻟﻠﻪ ﻣﻦ اﻟﻨﺎﺭ: ﻋﺼﺎﺑﺔ ﺗﻐﺰﻭ اﻟﻬﻨﺪ ﻭﻋﺼﺎﺑﺔ ﺗﻜﻮﻥ ﻣﻊ ﻋﻴﺴﻰ ﺑﻦ ﻣﺮﻳﻢ ﻋﻠﻴﻪ اﻟﺴﻼﻡ. “എന്റെ സമുദായത്തിലെ രണ്ട് സംഘത്തെ അല്ലാഹു നരകത്തിൽ നിന്നും സംരക്ഷിക്കും. ‘ഹിന്ദി’നോട് (ഇന്ത്യ) യുദ്ധം ചെയ്യുന്ന സംഘവും ഈസബ്നു മർയത്തോടൊപ്പം(അ) ഉണ്ടാകുന്ന സംഘവുമാണത്.” (നസാഈ: 2/64)

‘ഹിന്ദു’മായുള്ള യുദ്ധത്തെ സംബന്ധിച്ച് ചില ഹദീസ് ഗ്രന്ഥങ്ങളിൽ ഉദ്ധരിക്കപ്പെട്ടിട്ടുള്ള ഒരു നിവേദനമാണ് നാം മുകളിൽ ഉദ്ധരിച്ചത്. ഹദീസ് ‘സ്വഹീഹ്’ (സ്വീകാര്യതയുടെ ഹദീസ് നിദാന ശാസ്ത്ര മാനദണ്ഡങ്ങൾ പൂർത്തീകരിക്കപ്പെട്ടത്) ആണ് എന്ന് വന്നാൽ തന്നെ ഹദീസിന്റെ ഉള്ളടക്കത്തിൽ, ഇസ്‌ലാമോഫോബിയ പ്രചാരകർ ഊതി വീർപ്പിച്ച് ഉരുട്ടി കാണിക്കുന്നതു പോലെ ഇന്ത്യക്ക് ഭീഷണിയായ ഒന്നും തന്നെ അടങ്ങിയിട്ടില്ല എന്നതാണ് മറ്റൊരു സത്യം. ഹദീസിലെ ‘ഹിന്ദ്’ (اﻟﻬﻨﺪ) എന്ന പദത്തിനാണ് ‘ഇന്ത്യ’ എന്ന് പരിഭാഷ നൽകപ്പെടാറുള്ളത്.

In ancient times, India was much more extended to the North West and west (consisting of parts of modern Pakistan and Afghanistan). (https://www.culturalindia.net)

പൗരാണിക ഇന്ത്യയിൽ അധുനിക പാകിസ്ഥാനും അഫ്ഗാനിസ്ഥാന്റെ പല ഭാഗങ്ങളും ഉൾകൊണ്ടിരുന്നു എന്ന് നമുക്കേവർക്കും അറിയാമല്ലൊ. (ആധുനിക പാക്കിസ്ഥാനിൽ ഉൾപ്പെടുന്ന) സിന്ദു നദി തീരപ്രദേശങ്ങളെയാണ് പൗരാണിക കാലത്ത് ‘ഹിന്ദ്’ കൊണ്ട് ഉദ്ദേശിക്കപ്പെട്ടിരുന്നത്, നമ്മുടെ ഇന്നത്തെ ഇന്ത്യയെയല്ല എന്നതിനാൽ തന്നെ ‘ഹിന്ദി’നോടുള്ള യുദ്ധം എന്നതുകൊണ്ട് (ആധുനിക) ഇന്ത്യയോടുള്ള യുദ്ധമല്ല എന്ന് തിരിച്ചറിയാൻ വലിയ പ്രയാസമൊന്നുമില്ല.

കൂടാതെ, പ്രവാചക കാലഘട്ടത്തിലാവട്ടെ -പ്രവാചകനും പ്രവാചകാനുചരന്മാരും ഉൾപ്പെടെ- അറബികൾ ‘ഹിന്ദ്’ (ഇന്ത്യ) എന്ന് വിളിച്ചിരുന്നത് ആധുനിക ഇന്ത്യയെയല്ല. ഇന്ത്യയുടെ അന്നത്തെ ഭൂമിശാസ്ത്ര ഘടന പ്രകാരമായാലും ശരി പൗരാണിക അറേബ്യൻ മുസ്‌ലിംകളുടെ സാങ്കേതിക ഭാഷ പ്രകാരമായാലും ശരി, ഹദീസിൽ പറയപ്പെട്ടിട്ടുള്ള ‘ഹിന്ദ്’ അഥവാ ‘ഇന്ത്യ’ നമ്മുടെ രാജ്യമായ ഇന്ത്യ (ഭാരതം) അല്ലേയല്ല.

പൂർവ്വസൂരികളായ മുസ്‌ലിംകൾ ‘ഹിന്ദ്’ (ഇന്ത്യ) എന്ന് വിളിച്ചിരുന്നത് ‘ബസ്വറ’യെയാണ്. ഇന്നത്തെ ഇറാക്കിലെ പ്രസിദ്ധമായ ഒരു പട്ടണമാണ് ‘ബസ്വറ’. ‘ബസ്വറ’ ഉൾപ്പെടെയുള്ള പൗരാണിക ‘ഇറാക്’ -പ്രവാചക കാലഘട്ടത്തിൽ- പേർഷ്യൻ സാമ്രാജ്യത്തിന്റെ ഭാഗമായിരുന്നു. (https://mawdoo3.com)

അറേബ്യയിൽ നിന്നും ഇന്ത്യയിലേക്കുള്ള കരമാർഗ്ഗത്തിലാണ് ബസ്വറ നിലകൊള്ളുന്നത് എന്നത് കൊണ്ട്, ഇന്ത്യയുടെ ദിക്കിലുള്ള നാട് എന്ന നിലയിൽ ‘ബസ്വറ’യെ പ്രവാചക കാലഘട്ടം ‘ഹിന്ദ്’ (ഇന്ത്യ) എന്ന് വിളിച്ചു.

ﻭﻛﺎﻧﻮا ﻳﺴﻤﻮﻥ اﻟﺒﺼﺮﺓ ﻫِﻨﺪًا، ﻷﻧﻬﺎ ﻣﻦ ﺟﻬﺔ اﻟﻬﻨﺪ، ﻭﻣﻨﻬﺎ ﻳُﺴﻠﻚ ﺇِﻟَﻰ اﻟﻬﻨﺪ، ﻭﻟﻬﺬا ﻗﺎﻝ ﺧﺎﻟﺪ ﻟﻤﺎ ﻋﺰﻟﻪ ﻋﻤﺮ ﻋﻦ اﻟﺸﺎﻡ: ﺇﻥ ﻋﻤﺮ ﺃﻣﺮﻧﻲ ﺃﻥ [ ﺁﺗﻲ] اﻟﻬﻨﺪ. ﻗﺎﻝ اﻟﺮﻭاﻱ: ﻭﻛﺎﻧﺖ اﻟﻬﻨﺪ ﻋﻨﺪﻧﺎ اﻟﺒﺼﺮﺓ. ഇബ്നു റജബ് എഴുതി: പൂർവ്വകാല മുസ്‌ലിംകൾ ‘ബസ്വറ’ക്ക് ‘ഹിന്ദ്’ എന്നായിരുന്നു പേര് വെച്ചിരുന്നത്. ‘ബസ്വറ’, ഇന്ത്യയുടെ ദിക്കിലായതു കൊണ്ടും ബസ്വറയിലൂടെയാണ് ഇന്ത്യയിലേക്ക് എത്താനുള്ള കരമാർഗം എന്നതുകൊണ്ടുമായിരുന്നു അത്. ഉമർ, ഖാലിദിനെ ശാമിൽ നിന്നും നീക്കിയപ്പോൾ ഖാലിദ് ഇപ്രകാരം പറഞ്ഞത് അതുകൊണ്ടാണ്: എന്നോട് ഉമർ ‘ഹിന്ദി’ലേക്ക് (ഇന്ത്യ) ചെല്ലാൻ കൽപ്പിച്ചു. നിവേദകൻ പറയുന്നു: ‘ഹിന്ദ്’ (ഇന്ത്യ) എന്നാൽ ഞങ്ങളുടെ അടുക്കൽ ‘ബസ്വറ’യായിരുന്നു. (മജ്മൂഉ റസാഇലു ഇബ്നു റജബ്: 3:205)

‘ബസ്വറ’ക്കടുത്ത ‘ഉബുല്ല’ എന്ന സ്ഥലത്തെ ‘ഇന്ത്യൻ പുൽത്തകിടി’ (مرج الهند) എന്നാണ് വിളിക്കപ്പെട്ടിരിക്കുന്നത് എന്ന് ഇബ്നു ഖൽദൂൻ തന്റെ ‘താരീഖ്’ (2:507) ൽ പ്രസ്ഥാവിക്കുന്നുണ്ട്.

ഒട്ടനവധി ഹദീസ്-ചരിത്ര ഗ്രന്ഥങ്ങളിൽ ഉദ്ധരിക്കപ്പെട്ട സുപ്രധാനമായ ഒരു നിവേദനം ഇപ്രകാരമാണ്: “പ്രവാചക ശിഷ്യൻ ഖാലിദിബ്നു വലീദ്(റ) പറഞ്ഞു: ശാം അതിന്റെ സമൃതി ഇട്ടു തന്നതിന് ശേഷം വിശ്വാസികളുടെ നേതാവ്, ഉമർ ബിൻ ഖത്താബ് എനിക്ക് കത്തെഴുതി, ഞാൻ ഹിന്ദിലേക്ക് (ഇന്ത്യ) സഞ്ചരിക്കാൻ കൽപ്പന നൽകി – ഹിന്ദ് (ഇന്ത്യ) എന്നാൽ ഞങ്ങളുടെ മനസ്സിൽ ബസ്വറയാണ് – എനിക്കാകട്ടെ ഹിന്ദിലേക്ക് പോകാൻ വൈമനസ്യമുണ്ടായിരുന്നു…” (മുസ്നദു അഹ്മദ്: 4:90, ദലാഇലുന്നുബുവ്വ: 6:387, ജാമിഉൽ മസാനിദ്: 2: 389, അൽ ജിഹാദ്: ഇബ്നു അബീ ആസിം: 2: 666, മുഅ്ജമുൽ കബീർ: ത്വബ്റാനി: 4:137, മുഅ്ജമുൽ അവ്സത്വ് : 8:277, അൽ മഅ്’രിഫതു വത്താരീഖ് : ഫസ്വി: 3:115-116, അൽമുത്തഫകു വൽമുഫ്തറകു:3:1743-1744, താരീഖു ദിമശ്ക്: ഇബ്നു അസാകിർ: 40:310. ഒരു ഹദീസ് ആയിട്ടല്ലെങ്കിൽ, ചരിത്രപരമായ ഭൂമിശാസ്ത്ര (Historical geography) സാക്ഷ്യമായെങ്കിലും ഈ നിവേദനം പരിഗണിക്കപ്പെടേണ്ടതുണ്ട്.)

ﺃﻥ اﻟﺴﻠﻒ اﻟﺼﺎﻟﺢ «ﻛﺎﻧﻮا ﻳﺴﻤﻮﻥ (اﻟﺒﺼﺮﺓ) (ﻫﻨﺪاً)… ﺇﻥ اﻟﻬﻨﺪ ﻛﺎﻧﺖ ﻓﻲ ﻧﻔﻮﺳﻬﻢ اﻟﺒﺼﺮﺓ، ﻭﺑﻪ ﺗﻔﻬﻢ ﺳﺎﺋﺮ اﻷﺣﺎﺩﻳﺚ اﻟﻮاﺭﺩِ ﻓﻴﻬﺎ ﺫﻛﺮُ (اﻟﻬﻨﺪ) .

അബൂ ഉബൈദ മശ്ഹൂറിബ്നു ഹസൻ ഇബ്നു മഹ്മൂദ് ആലു സൽമാൻ വ്യക്തമാക്കുന്നു:

“പൂർവ്വസൂരികളായ സച്ഛരിതർ ‘ബസ്വറ’ക്ക് പേര് നൽകിയിരുന്നത് ‘ഇന്ത്യ’ (ഹിന്ദ്) എന്നായിരുന്നു… തീർച്ചയായും ഇന്ത്യയെന്നാൽ (ഹിന്ദ്) അവരുടെ മനസ്സിൽ ബസ്വറയാണ്. ഇന്ത്യയെ (ഹിന്ദ് ) സംബന്ധിച്ച് സ്മരിക്കുന്ന എല്ലാ ഹദീസുകളും ഇപ്രകാരം തന്നെയാണ് മനസ്സിലാക്കേണ്ടത്.” (അൽ ഇറാക്ക് ഫിൽ അഹാദീസി വ ആസാറുൽ ഫിതൻ: 1:360-364)

മസ്ഊദി (മരണം:346 ഹിജ്‌റ) പറഞ്ഞു: ബസ്വറയിൽ അതബതുബ്നു ഗസ്‌വാൻ ഹിജ്‌റ 16 അല്ലെങ്കിൽ 17 ന് ഖലീഫയുടെ കല്പന പ്രകാരം കടന്നു വന്നു. അന്ന് ബസ്വറയെ വിളിക്കപ്പെട്ടിരുന്നത് ‘ഇന്ത്യൻ ഭൂമി’ (അർദുൽ ഹിന്ദ് أرض الهند) എന്നായിരുന്നു. അത് വെളുത്ത പാറകളും ചരൽക്കല്ലുകളും നിറഞ്ഞ ഒരു ഭൂമിയായിരുന്നു. അതബതുബ്നു ഗസ്‌വാൻ ആണ് അതിനെ ഒരു പട്ടണമായി വാർത്തെടുക്കുന്നത്. ബസ്വറയും ചുറ്റുപാടുമുള്ള മറ്റു പട്ടണങ്ങളിൽ നിന്നും പേർഷ്യൻ സാമ്രാജ്യത്തേക്ക് ചെല്ലുന്ന യുദ്ധ സന്നാഹങ്ങളും സഹായങ്ങളും തടയുക കൂടി ബസ്വറക്ക് മേൽ ഉള്ള വിജയത്തിന് പിന്നിലെ ലക്ഷ്യത്തിൽ പെട്ടതായിരുന്നു.. (താരീഖുത്വബ്‌രി: 3:596, അത്തംബീഹ് വൽഇഷ്റാഫ് :1:310,)

At the time of the Muhammadan conquest, the county about Basra was called Arz-ul-Hind, the Land of India…

“മുഹമ്മദിയ അധിനിവേശ കാലഘട്ടത്തിൽ, ബസ്രയെ ‘അർസ്-ഉൾ-ഹിന്ദ്’, ഇന്ത്യൻ നാട് എന്നാണ് വിളിക്കപ്പെട്ടിരുന്നത്… ” എന്ന് പല ഇന്ത്യൻ ചരിത്രകാരന്മാരും വ്യക്തമാക്കിയിട്ടുണ്ട്. (The Indian Encyclopedia: Edited by Subodh Kapoor: Cosmo Publication: New Delhi: 2002, Vol: page: 4718)

ബ്രിട്ടീഷ് ആർമി ഓഫീസറും ഒറിയന്റലിസ്റ്റും പേർഷ്യൻ ഭാഷാ പണ്ഡിതനുമായ സർ എച്ച്. റൗളിൻസണും ഫ്രഞ്ച് ഒറിയന്റലിസ്റ്റും ചരിത്രകാരനുമായ ഹെൻറി കോർഡിയറും ഈ വസ്തുതയിലേക്ക് വിരൽ ചൂണ്ടുന്നുണ്ട്. (Cathay and the Way Thinker. Being a Collection of Medieval Notices of China)

പേർഷ്യൻ സാസാനിയ്യ ഭരണകൂടത്തിന്റെ കാലഘട്ടത്തിൽ (226 AD-651 AD) ഇറാഖിന്റെ ദക്ഷിണ ഭാഗത്തും ടൈഗ്രിസ് നദിയുടെ ഇടയിലുമായി ‘മീഷാൻ’ എന്ന പേരിൽ ഒരു ‘അമീർ ഭരണം’ (Emirate) നിലനിന്നിരുന്നു. മീഷാൻ രാജ്യത്തിന്റെ കേന്ദ്രസ്ഥാനം ബസ്വറയുടെ പേർഷ്യൻ ഗൾഫ് തീരത്തായിരുന്നു. ബസ്വറയുൾപ്പെടുന്ന ഇത്തരം എമിറേറ്റുകൾക്ക് ആ കാലഘട്ടത്തിൽ പേർഷ്യൻ സാമ്രാജ്യത്തിൽ തന്ത്രപ്രധാനവും അതി പ്രസക്തവുമായ സ്ഥാനമുണ്ടായിരുന്നു. ഇറാഖിൽ പേർഷ്യൻ സാമ്രാജ്യത്വ ശക്തികളോട് മുസ്‌ലിംകൾ ഏറ്റുമുട്ടുന്ന ആദ്യത്തെ യുദ്ധങ്ങളാണ് ഹഫീർ, ദാത്തു സലാസിൽ യുദ്ധങ്ങൾ. യുദ്ധത്തിനിടയിൽ തങ്ങളുടെ സൈന്യത്തിൽ നിന്നും ആരും ഓടി രക്ഷപ്പെടാതിരിക്കാൻ വലിയ ചങ്ങല വലയങ്ങളുമായാണ് പേർഷ്യക്കാർ യുദ്ധത്തിന് വന്നത് എന്നതിനാൽ ‘ദാത്തു സലാസിൽ’ അഥവാ ‘ചങ്ങല കെട്ടുകളുടെ യുദ്ധം’ എന്നാണ് ആ പോരാട്ടത്തെ മുസ്‌ലിംകൾ വിളിച്ചത്. (മുഖ്തസറു താരീഖിൽ ബസ്വറ :അലീ ളരീഫ് അൽ അഅ്‌സമി: 7-12)

ബസ്വറയെ ഇന്ത്യ എന്നായിരുന്നു പ്രവാചക കാലഘട്ടത്തിൽ വിളിക്കപ്പെട്ടിരുന്നത് എന്നത് ഒട്ടനവധി ചരിത്രജ്ഞർ തങ്ങളുടെ വിശ്വപ്രസിദ്ധ ചരിത്ര ഗ്രന്ഥങ്ങളിൽ വ്യക്തമാക്കിയിട്ടുണ്ട്:

* അബൂ യുസുഫ് (മരണം: 182: ഹിജ്റ): ‘അൽഖറാജ്’ (1:73).

* ഇബ്നു സഅ്ദ് (മരണം: 230: ഹിജ്റ): ‘ത്വബകാത്ത്’ (7:3).

* ഖലീഫ ബിൻ ഖയ്യാത്ത് (മരണം: 240: ഹിജ്റ): ‘താരീഖ് ‘ (1:117).

* ത്വബ്‌രി (മരണം: 310: ഹിജ്റ): ‘താരീഖ് ‘ (3:591).

* അദ്ദാരിമി അൽ ബുസ്തി (മരണം :354 ഹിജ്‌റ): ‘അസ്സീറത്തുന്നബവിയ്യ വ അഖ്ബാരിൽ ഖുലഫാ’ (2:476).

* മുത്വഹ്‌ഹിർ ഇബ്നു ത്യാഹിർ അൽമക്ദസി (മരണം: 355 ഹി): ‘അൽബദ്ഉ വത്താരീഖ്’ (5:175).

* ഇബ്നുൽ അസീർ (മരണം: 630 ഹി): ‘അൽ കാമിൽ ഫിത്താരിഖ് ‘ (2:316).

* ദഹബി (മരണം: 748 ഹി): ‘സിയറു അഅ്ലാമിന്നുബലാഅ്’ (2:393).

* ഇബ്നു കസീർ (മരണം: 774 ഹി): ‘അൽ ബിദായ വന്നിഹായ’ (7:57).

ഖലീഫ ഉമറിന്റെ കാലഘട്ടത്തിലാണ് ബസ്വറ മുസ്‌ലിംകൾ വിജയിച്ചടക്കുന്നതും പട്ടണമാക്കുന്നതും എന്നും ഇതിനെ പറ്റി പ്രവാചകൻ (സ) സുവിശേഷമറിയിച്ചിട്ടുണ്ട് എന്നും ഇബ്നുൽ വർദ്ദി (മരണം: 749) തന്റെ ‘താരീഖിൽ’ (1:137) രേഖപെടുത്തുന്നു. ഇന്ത്യയെ സംബന്ധിച്ച, നാം ചർച്ച ചെയ്യുന്ന ഹദീസിനെ സംബന്ധിച്ചാണ് ഇബ്നുൽ വർദ്ദി സൂചിപ്പിക്കുന്നത്.

വെള്ള കലർന്ന മിനുസമുള്ള പാറയെയും കല്ലുകളേയുമാണ് ‘ബസ്റ’ (اﻟﺒَﺼْﺮَﺓُ) എന്ന് അറബിയിൽ വിളിക്കപ്പെടുന്നത്. വളരെ കട്ടിയുള്ള ഭൂമിയെ ‘അൽ ബസ്റു’ എന്ന് പറയുമെന്ന് കസ്സാസ് തന്റെ ‘ജാമിഅ്’ ൽ ഭാഷാ പണ്ഡിതന്മാരിൽ നിന്നും ഉദ്ധരിക്കുന്നു. ഈ ഭൂമി മുസ്‌ലിംകൾ വിജയിച്ചടക്കിയപ്പോൾ നിറയെ കല്ലുകൾ ഉള്ളതിനാൽ അവർ ആ നാടിന് ‘ബസ്വറ’ എന്ന് പേര് മാറ്റി വിളിക്കാൻ തുടങ്ങി എന്ന് ഒട്ടനവധി ഭാഷാ പണ്ഡിതരും ചരിത്രകാരൻമാരും രേഖപ്പെടുത്തുന്നുണ്ട്. (താജുൽ ഉറൂസ്: 10:203, ഉംദത്തുൽ ക്വാരി: 6:57)

ശർഖി ഇബ്നുൽ ക്വുത്വാമി പറഞ്ഞു: മുസ്‌ലിംകൾ ബസ്വറയിലേക്ക് വന്നപ്പോൾ ദൂരെ നിന്ന് നിരീക്ഷിച്ച സന്ദർഭത്തിൽ കുറെ കല്ലുകളാണ് അവർ കണ്ടത്. അപ്പോൾ അവർ പറഞ്ഞു: ഇത് കല്ലുകൾ നിറഞ്ഞ ഭൂമി അഥവാ ബസ്വറ ആണ്. അങ്ങനെയാണ് മുസ്‌ലിംകൾക്കിടയിൽ ആ നാടിന് അപ്രകാരം പേര് വന്നത്. (താരീഖു മദീനത്തുൽ ബസ്വറ: അബ്ദുല്ലാഹിബ്നു ഈസബ്‌നു ഇസ്മാഈൽ അന്നജ്ദി :19)

ഹിന്ദ് അഥവാ ബസ്വറയോടുള്ള യുദ്ധം എന്തുകൊണ്ട് പ്രോത്സാഹിപ്പിക്കപ്പെട്ടു? ഹിന്ദ് അഥവാ ബസ്വറയോട് യുദ്ധം എന്തുകൊണ്ട് പ്രോത്സാഹിപ്പിക്കപ്പെട്ടു? എന്നതാണ് അവശേഷിക്കുന്ന ചോദ്യം.

മതേതരത്വവും ജനാധിപത്യവും രാഷ്ട്രങ്ങളുടെ ചട്ടക്കൂടും രാഷ്ട്രമീമാംസയുടെ അടിസ്ഥാന ശിലയുമായ, ആധുനിക ലോക വ്യവസ്ഥയിലല്ല ഹിന്ദ് അഥവാ ബസ്വറയോട് യുദ്ധം ചെയ്യുന്നത് പ്രവാചകൻ (സ) പ്രോത്സാഹിപ്പിച്ചത്. ഇസ്‌ലാമിന്റെ ആവിർഭാവകാലഘട്ടത്തിൽ പ്രവാചക ശിഷ്യന്മാർ നേതൃത്വം വഹിച്ച പടയോട്ടങ്ങളുടെ ലക്ഷ്യവും പശ്ചാത്തലവും ആദ്യമായി നാം മനസ്സിലാക്കണം. ആദർശ സഹവർത്തിത്വമോ ജനാധിപത്യ മൂല്യങ്ങളോ തൊട്ടു തീണ്ടിയിട്ടില്ലാത്ത തീവ്ര മത വികാരത്തിലും അന്ധമായ മൗലികവാദത്തിലും വേരുറച്ച മതാധിഷ്‌ഠിത ഏകാധിപത്യ ഭരണങ്ങളായിരുന്നു (Theocratic Autocracy) അന്നത്തെ രാഷ്ട്രങ്ങളിൽ നിലനിന്നിരുന്നത്. ചരിത്രത്തിന്റെ അപൂർവം ചില ദശകളിൽ ചില നാടുകളിൽ അപൂർവ്വം ഭരണാധികാരികൾ മാത്രമെ അപര വിശ്വാങ്ങൾക്കും ആദർശങ്ങൾക്കും പ്രത്യയ ശാസ്ത്രങ്ങൾക്കും സഹവർത്തിത്വത്തിന് അവസരം നൽകിയിരുന്നുള്ളു. മതസ്വാതന്ത്ര്യവും സഹസ്ഥിതിയും വിവിധ പ്രത്യയശാസ്‌ത്രങ്ങളോട് സമാധാനപരമായ സഹവർത്തിത്വവുമൊക്കെ തീർത്തും അസംഭവ്യങ്ങളായിരുന്ന പൗരാണിക ലോക വ്യവസ്ഥയുടെ പശ്ചാത്തലത്തിൽ നിന്നു കൊണ്ട് വേണം ഹദീസിലെ യുദ്ധാഹ്വാനത്തെ മനസ്സിലാക്കാൻ.

ഇസ്‌ലാം സ്വീകരിക്കുകയോ ആചരിക്കുകയോ അതിന്റെ ചിഹ്നങ്ങൾ പ്രകടിപ്പിക്കുകയോ, ഇസ്‌ലാം പ്രചരിപ്പിക്കുകയോ ചെയ്യുന്നതിനുള്ള സ്വാതന്ത്ര്യം നിഷേധിക്കപ്പെടുകയും അവയുടെ പേരിൽ മർദ്ദനങ്ങളും പീഢനങ്ങളും അനുഭവിക്കുകയും ചെയ്യേണ്ടിവരുന്ന അവസ്ഥ ഇസ്‌ലാമികേതര രാജ്യങ്ങളിൽ ഇല്ലാതാവുകയും ഇസ്‌ലാം മതം പൂർണമായും ഉൾക്കൊണ്ട് ജീവിക്കാൻ കഴിയുന്ന അവസ്ഥ സംജാതമാവുകയും ചെയ്യുന്നതിന് മാത്രമായിരുന്നു ഈ യുദ്ധങ്ങൾ.

“ഫിത്‌ന ഇല്ലാതാകുന്നതുവരെ യുദ്ധം നടത്തി കൊള്ളുക” ( وَقَاتِلُوهُمْ حَتَّىٰ لَا تَكُونَ فِتْنَةٌ ) എന്ന ഖുർആൻ വചനത്തിന് പ്രവാചക ശിഷ്യൻ ഇബ്നു ഉമർ (റ) നൽകിയ വ്യാഖ്യാനം കാണുക:

ﻓﻌﻠﻨﺎ ﻋﻠﻰ ﻋﻬﺪ ﺭﺳﻮﻝ اﻟﻠﻪ ﺻﻠﻰ اﻟﻠﻪ ﻋﻠﻴﻪ ﻭﺳﻠﻢ ﻭﻛﺎﻥ اﻹﺳﻼﻡ قليلا، ﻓﻜﺎﻥ اﻟﺮﺟﻞ ﻳﻔﺘﻦ ﻓﻲ ﺩﻳﻨﻪ: ﺇﻣﺎ ﻗﺘﻠﻮﻩ، ﻭﺇﻣﺎ ﻳﻌﺬﺑﻮﻧﻪ، ﺣﺘﻰ ﻛﺜﺮ اﻹﺳﻼﻡ ﻓﻠﻢ ﺗﻜﻦ ﻓﺘﻨﺔ

“അല്ലാഹുവിന്റെ തിരുദൂതരുടെ(സ) കാലത്ത് ഞങ്ങള്‍ അങ്ങനെ (ഫിത്‌ന ഇല്ലാതാകുന്നതുവരെ യുദ്ധം നടത്തുക എന്ന പ്രവർത്തനം) ചെയ്തിട്ടുണ്ട്. അന്ന് ഇസ്‌ലാം (മുസ്‌ലിംകള്‍) അല്‍പമായിരുന്നു. അതിനാല്‍, ഒരു മുസ്‌ലിം തന്‍റെ മത കാര്യത്തില്‍ ഫിത്‌നക്ക് (പരീക്ഷണത്തിന്/ കുഴപ്പത്തിന്) വിധേയനാകുമായിരുന്നു. ഒന്നുകില്‍ അവിശ്വാസികൾ അവനെ വധിക്കുമായിരുന്നു, അല്ലെങ്കില്‍ അവിശ്വാസികൾ അവനെ മര്‍ദ്ദനങ്ങൾക്കും പീഢനങ്ങൾക്ക് വിധേയമാക്കുകയും ചെയ്യുമായിരുന്നു. അങ്ങനെ, ഇസ്‌ലാം (മുസ്‌ലിംകള്‍) വര്‍ദ്ധിച്ചു, അപ്പോള്‍ ഈ ഫിത്‌ന ഇല്ലാതായി….’ (സ്വഹീഹുൽ ബുഖാരി: 4514)

ഇസ്‌ലാം സ്വീകരിക്കുവാനും ജീവിതത്തിൽ പ്രാവർത്തികമാക്കാനും, വസ്ത്രധാരണ രീതി, ബാങ്ക്, ആരാധനാലയങ്ങൾ, മതപഠനം, ആഘോഷങ്ങൾ തുടങ്ങിയ ഇസ്‌ലാമിക ചിഹ്നങ്ങൾ ആചരിക്കുവാനും, ഇസ്‌ലാമിക പ്രബോധന-പ്രചാരണങ്ങളിൽ ഏർപ്പെടാനും ഒരു വിശ്വാസിക്ക് ദൈവത്തിന്റെ ഭൂമിയിൽ എവിടെയും സാധ്യമാകുന്ന സ്ഥിതി വിശേഷം നിലവിൽ വരണം. അഥവാ മുസ്‌ലിം സമൂഹം ഇസ്‌ലാം വിരുദ്ധ രാഷ്ട്രങ്ങളിൽ നിന്ദ്യരും മർദ്ദിതരുമായി കഴിയുന്ന അവസ്ഥ ഇല്ലാതാവുകയും അല്ലാഹുവിന്റെ മതം ഭൂമിയിലെ ഏത് സാമൂഹിക വ്യവസ്ഥിതിയിലും പ്രൗഢിയോടെ പുലർത്താൻ സാധിക്കണം. (അതിനർത്ഥം മറ്റു മതങ്ങളോ ആദർശങ്ങളോ ഇസ്‌ലാമിന് കീഴ്‌പ്പെടുന്ന അവസ്ഥ സൃഷ്ടിക്കണമെന്നോ ഭൂമിയിൽ മുഴുവൻ ഇസ്‌ലാമിക ഭരണം സ്ഥാപിക്കപ്പെടണം എന്നോ അല്ല. ഇസ്‌ലാമും മുസ്‌ലിംകളും മറ്റു മതങ്ങളാലും ആദർശങ്ങളാലും അടിച്ചമർത്തപ്പെടുകയും നിന്ദ്യരാക്കപ്പെടുകയും ചെയ്യുന്ന ദുരവസ്ഥ ഇല്ലാതാവുകയും ഇസ്‌ലാമിക പ്രബോധനം സാധ്യമാവുകയും ചെയ്യണം എന്നാണ്.) അതിനു തടസ്സമായി നിൽക്കുന്ന ഓട്ടോക്രസികളോടാണ് -ഇസ്‌ലാമിക രാഷ്ട്രത്തിന്റെ ഭരണാധികാരി എന്ന നിലയിൽ- ചില ‘സ്വതന്ത്ര്യ സമരങ്ങൾ/ യുദ്ധങ്ങൾ’ നടത്താൻ മുഹമ്മദ് നബി (സ) ആഹ്വാനം ചെയ്തത്. ഇതാണ് ഈ സ്വാതന്ത്ര്യ യുദ്ധങ്ങളുടെ ലക്ഷ്യം.

“മര്‍ദ്ദനം ഇല്ലാതാവുകയും, മതം അല്ലാഹുവിന് വേണ്ടിയാവുകയും ചെയ്യുന്നത് വരെ നിങ്ങളവരോട് യുദ്ധം നടത്തിക്കൊള്ളുക. എന്നാല്‍ അവര്‍ (യുദ്ധത്തില്‍ നിന്ന്‌) വിരമിക്കുകയാണെങ്കില്‍ (അവരിലെ) അക്രമികള്‍ക്കെതിരിലല്ലാതെ പിന്നീട് യാതൊരു കയ്യേറ്റവും പാടുള്ളതല്ല.” (ഖുർആൻ: 2:193)

“എന്നാല്‍ അവര്‍ (യുദ്ധത്തില്‍ നിന്ന്‌) വിരമിക്കുകയാണെങ്കില്‍ (അവരിലെ) അക്രമികള്‍ക്കെതിരിലല്ലാതെ പിന്നീട് യാതൊരു കയ്യേറ്റവും പാടുള്ളതല്ല.” എന്ന ഭാഗം ഈ യുദ്ധങ്ങളുടെ പശ്ചാത്തലത്തിലേക്ക് കൂടുതൽ വെളിച്ചം വീശുന്നുണ്ട്. ആധുനിക മതേതര ജനാധിപത്യ രാഷ്ട്രങ്ങളിൽ ഇഷ്ടമുള്ള മതം സ്വീകരിക്കാനും അനുഷ്ഠിക്കാനും പ്രചരിപ്പിക്കാനും സ്വാതന്ത്ര്യമനുവദിക്കുന്ന അവസ്ഥ നിലനിൽക്കുമ്പോൾ ഇസ്‌ലാമിലെ സ്വാതന്ത്ര്യ യുദ്ധങ്ങൾക്ക് പ്രസക്തിയില്ല.

മുമ്പ് സൂചിപ്പിച്ചത് പോലെ, സഹവർത്തിത്വവും ജനാധിപത്യവും ഒരു നിലക്കും ഉൾകൊള്ളാത്ത തീവ്ര മത വികാരത്തിലധിഷ്ഠിതമായ തിയോക്രസികളായിരുന്നു ഇസ്‌ലാം ഉദയം കൊണ്ട കാലഘട്ടത്തിൽ ലോകത്തുടനീളം നിലനിന്നിരുന്നത്; പ്രത്യേകിച്ച് അറബ് ഉപഭൂഖണ്ഡത്തിൽ അധികാരം വാണിരുന്നത്. അറബ് ഉപഭൂഖണ്ഡത്തിലെ ഗോത്ര വ്യവസ്ഥ മതസ്വാതന്ത്രത്തെ മുഴുവനായും നിഷേധിച്ചിരുന്നു. പ്രസ്തുത സമൂഹങ്ങളിലെയും ഗോത്രങ്ങളിലേയും രാജാക്കന്മാർക്ക് പോലും സമൂഹ മനസ്സിനെതിരായ ആദർശങ്ങളോ മതമോ സ്വീകരിക്കാനും വിശ്വസിക്കാനുമുള്ള അവകാശവും സ്വാതന്ത്ര്യവും ഇല്ലാതിരിക്കാൻ മാത്രം തീവ്രമായിരുന്നു അന്ന് നിലനിന്നിരുന്ന വർഗീയതയും അസഹിഷ്ണുതയും.

റോമൻ ചക്രവർത്തിക്ക് പ്രവാചകൻ (സ) ഒരു കത്തയക്കുകയുണ്ടായി. ഹാരിസിബ്നു ഉമൈർ അൽ അസ്ദിയായിരുന്നു കത്തുമായി പുറപ്പെട്ട ദൂതൻ. റോമൻ ചക്രവർത്തിയുടെ, ശാമിലെ ഗവർണറായ ശർഹബീലിബ്നു അംറ് അൽ ഗസ്സാനി, പ്രവാചകന്റെ ദൂതനെ ബന്ദിയാക്കുകയും കെട്ടിയിട്ട് കഴുത്തറുക്കുകയും ചെയ്തു. തങ്ങളുടെ ക്രിസ്ത്യൻ സാമ്രാജ്യത്തിന്റെ അതിർത്തിയിൽ കാലു കുത്താൻ പോലും ഒരു മുസ്‌ലിമിനും അനുവാദമില്ലെന്ന്, ദൂതന്മാരെ വധിക്കുക എന്ന, അക്കാലഘട്ടത്തിലെ അസഹിഷ്ണുതയുടെ പാരമ്യരൂപത്തിലൂടെ റോമൻ സാമ്രാജ്യം പ്രഖ്യാപിച്ചു. ഇതാണ് ഹിജ്റ എട്ടാം വർഷം ശാമിലെ ക്രിസ്ത്യാനികളും മുസ്‌ലിംകളും തമ്മിൽ നടന്ന യുദ്ധമായ മുഅ്ത യുദ്ധത്തിലേക്ക് നയിച്ചത് എന്ന് ചില ചരിത്ര ഗ്രന്ഥങ്ങൾ സൂചിപ്പിക്കുന്നു. (മഗാസി: വാക്വിദി: 2:755, ത്വബകാത്തു ഇബ്നു സഅ്ദ്: 4:256)

പ്രേർഷ്യൻ സാമ്രാജ്യത്വത്തിന്റെ ആദർശ അസഹിഷ്ണുത ചരിത്ര പ്രസിദ്ധമാണ്. സ്വാതന്ത്ര്യ നിഷേധത്തിൽ പരസ്പരം മത്സരിച്ചിരുന്ന ക്രിസ്ത്യൻ റോമും പേർഷ്യൻ ജൂതന്മാരും തമ്മിലുള്ള അടിച്ചമർത്തലുകളും വർഗീയ യുദ്ധങ്ങളും കലാപങ്ങളും അക്കാലഘട്ടത്തിന്റെ പ്രധാന ഇതിവൃത്തം ആയിരുന്നു.

ഹദീസിൽ ഉദ്ദേശിക്കപ്പെട്ട ഹിന്ദ് അഥവാ ബസ്വറ ആധുനിക ഇറാഖിന്റെ ഭാഗമാണെന്നും ക്രിസ്താബ്ദം 638 ൽ ഖലീഫ ഉമറിന്റെ കാലഘട്ടത്തിൽ മുസ്‌ലിംകൾ വിജയിച്ചടക്കുന്നതു വരെ ഇറാഖ് പേർഷ്യൻ സാമ്രാജ്യത്വത്തിന്റെ കീഴിലായിരുന്നു എന്നും സൂചിപ്പിച്ചല്ലൊ.

എൻസൈക്ലോപീഡിയ ഇറാനിക (Encyclopedia iranica) പറയുന്നു:

സാസാനിയൻ രാജാക്കൻമാരാണ് പേർഷ്യൻ സാമ്രാജ്യത്തെ നയിച്ച അവസാന രാജവംശം. പേർഷ്യൻ സാമ്രാജ്യത്തിനു മേൽ ഇസ്‌ലാമിന്റെ അധിജയത്തിന് തൊട്ട് മുമ്പ് (224 CE–650 CE) വരെ ഭരണത്തിന് നേതൃത്വം വഹിച്ച വംശപരമ്പരയാണ് സാസാനിയൻ രാജവംശം. (https://iranicaonline.org/articles/sasanian-dynasty )

അബ്ദുശ്ശാഫി മുഹമ്മദ് അബ്ദുല്ലത്തീഫ് എഴുതി:

“പേർഷ്യൻ രാജാവായ കിസ്റക്ക് -അദ്ദേഹത്തെ ഇസ്‌ലാമിലേക്ക് ക്ഷണിച്ചു കൊണ്ട്- മുഹമ്മദ് നബി (സ) അയച്ച കത്ത് അഹങ്കാരത്തോടെയും ധാർഷ്ട്യത്തോടെയും പിച്ചിച്ചീന്തുകയാണ് ആ ഏകാധിപതി ചെയ്തത്. മാത്രമല്ല യമനിൽ ഭരണം നടത്തിയിരുന്ന തന്റെ പ്രതിനിധിയായ ബാദാനോട് മുഹമ്മദ് നബിയെ പിടികൂടാനും കൊല്ലാനും ആവശ്യപ്പെടുകയും ചെയ്തു. രാജാധിരാജനായ തന്നെ മറ്റൊരു മതത്തിലേക്ക് ക്ഷണിക്കാനുള്ള തന്റേടത്തിനു ശിക്ഷയായി കൊണ്ടാണ് മുഹമ്മദ് നബിയെ വധിക്കാൻ അദ്ദേഹം ഉത്തരവിട്ടത് (താരീഖു ത്വബ്‌രി 2:654). തുടക്കം മുതലേ അങ്ങേയറ്റം ശത്രുതാ മനോഭാവത്തോടെയാണ് പേർഷ്യൻ സാമ്രാജ്യത്വം ഇസ്‌ലാമിനെ അഭിമുഖീകരിച്ചത്. കാലാന്തരത്തിൽ ഈ അനീതി നിറഞ്ഞ ശത്രുത ഇസ്‌ലാമിക രാഷ്ട്രത്തിന്റെ അതിർത്തികളിലുള്ള അറബ് ഗോത്രങ്ങളെ ഭരണകൂടത്തിനെതിരെ ഇളക്കി വിടുകയും സായുധ വിപ്ലവങ്ങൾക്ക് പ്രോത്സാഹിപ്പിച്ചു കൊണ്ടിരിക്കുകയും ചെയ്തു. ഇത് യുദ്ധത്തിലാണ് കലാശിച്ചത്. ആഭ്യന്തര ലഹളക്കാരുമായും സഹായികളായ പേർഷ്യക്കാരുമായും മുസ്ന ഇബ്നു ഹാരിസ അശൈബാനിയുടെ നേതൃത്വത്തിലുള്ള മുസ്‌ലിം സൈന്യം അറബ് ഗൾഫ് തീരത്ത് നിരന്തരം ഏറ്റുമുട്ടി. ഇറാൻ മുഴുവൻ വിജയിച്ചടക്കുകയും ഇസ്‌ലാമിക നേതൃത്വത്തിനു കീഴിൽ സമ്പൂർണ്ണമായി പ്രവേശിക്കുകയും ചെയ്യുന്നത് വരെ പേർഷ്യക്കാരിൽ നിന്നുമുള്ള വൈദേശിക കലാപങ്ങൾ കെട്ടടങ്ങിയില്ല. മുസ്‌ലിംകൾ അല്ല ശത്രുതയ്ക്ക് തുടക്കം കുറിച്ചത് എന്നതിനാൽ തന്നെ നീതിയുള്ള നിഷ്പക്ഷരായ ആർക്കും പേർഷ്യക്കാരോടുള്ള യുദ്ധത്തിൽ മുസ്‌ലിംകളെ പഴിക്കാൻ കഴിയില്ല… പേർഷ്യൻ രാജ്യങ്ങളിലുള്ള വിജയത്തിന്റെ സുവിശേഷങ്ങളും യുദ്ധാർജിത സ്വത്തുക്കളും മദീനയിൽ ഖലീഫാ ഉമറിലേക്ക് മുസ്‌ലിം സൈന്യത്തിന്റെ നേതാക്കളിൽ ഒരാളായ സഅദ്ബ്നു അബീ വഖാസ് അയക്കുകയും പേർഷ്യൻ രാജ്യങ്ങളിൽ യുദ്ധം തുടരാനുള്ള അനുവാദം ആവശ്യപ്പെടുകയും ചെയ്തപ്പോൾ ഉമർ (റ) അത് നിരസിച്ചു കൊണ്ട് പറഞ്ഞു: “അവർക്കും നമുക്കുമിടയിൽ ഒരു വലിയ പർവതമുണ്ടാവുകയും നാം അവരിൽ നിന്നും, അവർ നമ്മിൽ നിന്നും സുരക്ഷിതരായിരിക്കുകയും ചെയ്തിരുന്നെങ്കിൽ എന്നാണ് ഞാൻ ആശിച്ചു പോവുന്നത്… യുദ്ധാർജിത സ്വത്തിനേക്കാൾ മുസ്‌ലിംകളുടെ സുരക്ഷയെയാണ് ഞാൻ മുന്തിക്കുന്നത്” (താരീഖു ത്വബ്‌രി 4:28). ഖലീഫയുടെ ഈ നിലപാട് മുസ്‌ലിംകൾ യുദ്ധക്കൊതിയന്മാർ അല്ലായിരുന്നു എന്നതിന് ഏറ്റവും വലിയ തെളിവാണ്. ശക്തിയും വാളും ഉപയോഗിച്ച് ഇസ്‍ലാം പ്രചരിപ്പിക്കൽ ആയിരുന്നില്ല മുസ്‌ലിംകളുടെ ഉദ്ദേശ്യം. (അസ്സീറത്തുന്നബവിയ്യ വത്താരീഖുൽ ഇസ്‌ലാം: 1:259)

എൻസൈക്ലോപീഡിയ ഇറാനിക (Encyclopedia iranica) പറയുന്നു:

സാസാനിയൻ രാജവംശത്തിലെ ഏറ്റവും കൂടുതൽ കാലം (r. 309-79 CE) ഭരിച്ച ചക്രവർത്തി ‘ശാഹ്പോർ രണ്ടാമൻ’ പേർഷ്യൻ സ്വേച്ഛാധിപത്യത്തിന്റെയും അസഹിഷ്ണുതയുടേയും പ്രതീകമായിരുന്നു.

ഇസ്‌ലാമിന് മുമ്പത്തെ അറബ് ഗോത്രങ്ങളെ അടിച്ചമർത്താനും സാമ്രാജ്യത്തിന്റെ അതിർത്തികൾ സുരക്ഷിതമാക്കാനും അയാൾ ഒരു സൈനിക പ്രചാരണം ആരംഭിച്ചു. (താരീഖുത്വബ്‌രി, തജാറുബുൽ ഉമം: മസ്കവൈഹി).

ﻓﺄﻓﺸﻲ ﻓﻴﻬﻢ اﻟﻘﺘﻞ، ﻭﺳﻔﻚ ﻓﻴﻬﻢ ﻣﻦ اﻟﺪﻣﺎء ﺳﻔﻜﺎ ﺳﺎﻟﺖ ﻛﺴﻴﻞ اﻟﻤﻄﺮ، ﺣﺘﻰ ﻛﺎﻥ اﻟﻬﺎﺭﺏ ﻣﻨﻬﻢ ﻳﺮﻯ ﺃﻧﻪ ﻟﻦ ﻳﻨﺠﻴﻪ ﻣﻨﻪ ﻏﺎﺭ ﻓﻲ ﺟﺒﻞ، ﻭﻻ ﺟﺰﻳﺮﺓ ﻓﻲ ﺑﺤﺮ….ﻭﺇﻥ ﺳﺎﺑﻮﺭ ﺿﺮﻱ ﺑﻘﺘﻞ اﻟﻌﺮﺏ، ﻭﻧﺰﻉ ﺃﻛﺘﺎﻑ ﺭﺅﺳﺎﺋﻬﻢ ﺇﻟﻰ ﺃﻥ ﻫﻠﻚ. ﻭﻛﺎﻥ ﺫﻟﻚ ﺳﺒﺐ ﺗﺴﻤﻴﺘﻬﻢ ﺇﻳﺎﻩ ﺫا اﻷﻛﺘﺎﻑ

ത്വബ്‌രി എഴുതുന്നു: “അറബികളെ അയാൾ പരക്കെ കൊന്നു. മഴ കുത്തിച്ചൊരിയുന്നതു പോലെ അറേബ്യയിൽ രക്തം ചിന്നി ചിതറി. മരണത്തിൽ നിന്ന് ഓടി രക്ഷപ്പെടാൻ ശ്രമിച്ചവർ തന്നെ മനസ്സിൽ മന്ത്രിച്ചു; ഒരു പർവ്വതത്തിനു മൂലയിലുള്ള ഗുഹയും ഒരു സമുദ്ര പ്രാന്തത്തിലുള്ള ദ്വീപും തന്നെ ഒരിക്കലും രക്ഷിക്കില്ല എന്ന്. ‘ശാഹ്പോർ’ ചക്രവർത്തി അറബികളെ കൂട്ടക്കുരുതി നടത്തി. അറബികളുടെ നേതാക്കളുടെ തോളെല്ലുകൾ വലിച്ചൂരി അവരെ ക്രൂരമായ രീതിയിൽ കൊലപ്പെടുത്തി. അക്കാരണത്താൽ അയാളെ ‘ദുൽ അക്താഫ്’ (ﺫﻱ اﻻﻛﺘﺎﻑ) ‘തോളെല്ലുകളുടെ ഉടമ’ എന്ന് അറബികൾ വിളിച്ചു.” (താരീഖു ത്വബ്‌രി: 2:57)

‘ശാഹ് പോർ’ രണ്ടാമൻ ഇറാഖിലെ അയാദുകളെ ആദ്യം ആക്രമിച്ചു. പിന്നീട് പേർഷ്യൻ ഗൾഫ് മുറിച്ച് കടന്ന് ബഹ്റൈൻ, കത്വർ എന്നീ പ്രദേശങ്ങളും, തമീം, ബകർ, വാഇൽ, അബ്ദുൽ കൈസ് തുടങ്ങിയ ഗോത്രക്കാരുടെ വാസസ്ഥലമായ ഹജറും ആക്രമിച്ചു. (താരീഖു ത്വബ്‌രി) ഒട്ടുമിക്ക ഗോത്രവർഗ്ഗക്കാരേയും കശാപ്പു ചെയ്യുന്നതിന് പുറമെ അറബികൾക്ക് വെള്ളം കിട്ടാതിരിക്കാൻ ജലസ്രോതസ്സുകൾ മണ്ണിട്ടു മൂടി.

തുടർന്ന് കിഴക്കൻ അറേബ്യയിലും സിറിയയിലും ആക്രമണം അഴിച്ചു വിടുകയും യമാമ, ബക്കർ, തഗ്‌ലിബ് നഗരങ്ങൾക്കുമെതിരെ ആഞ്ഞടിച്ചു. (ഗററു അഖ്ബാരി മുലൂകുൽ ഫറസ്: സഗാലിബി, തജാറുബുൽ ഉമം: മസ്കവൈഹി)

ചില അറബ് ഗോത്രങ്ങളെ നിർബന്ധിതമായി മാറ്റിപ്പാർപ്പിക്കുകയും സാസാനിയൻ സാമ്രാജ്യത്തിലേക്ക് നാടുകടത്തുകയും ചെയ്യപ്പെട്ടു.

അറബികളിൽ നിന്ന് തുടർന്ന് ആക്രമണങ്ങൾ ഉണ്ടാകാതിരിക്കാൻ, ‘ശാഹ്പോർ’ II, “അറബി മതിൽ” എന്ന പേരിൽ ഒരു പ്രതിരോധ സംവിധാനം നിർമ്മിച്ചു. (T. Daryaee, “Memory and History: The Construction of the Past in Late Antique Persia,” Nāma-ye Irān-e Bāstan/The International Journal of Ancient Iranian Studies 1/2, 2001-02, pp. 1-14)

‘ശാഹ് പോർ’ II ന്റെ ഭരണകാലത്ത് സാസാനിയൻ സാമ്രാജ്യത്തിലെ ക്രിസ്ത്യാനികൾ നേരിടേണ്ടി വന്ന കൊടും പീഡനങ്ങൾ അർമേനിയൻ സ്രോതസ്സുകൾ പോലും രേഖപ്പെടുത്തുന്നുണ്ട്.

‘ശാഹ് പോർ’ രണ്ടാമന്റെ വേട്ടക്കിരയായ പ്രശസ്ത ക്രിസ്ത്യൻ രക്തസാക്ഷികളുടെ പേരുകളിൽ അയാളുടെ പ്രധാന കരകൗശലത്തൊഴിലാളിയായ പോസി (പുസൈ), അദ്ദേഹത്തിന്റെ മകൾ മാർത്ത, തെക്ല, ദാനക്, ബൗത എന്നിവരും ഉൾപെടുന്നു. സ്ത്രീകൾ ഉൾപ്പെടെ സാൻ, മാമ, മെസഖ്യ, അന്ന, അബ്യത്, സാത്തായ്, മെസഖ്യ എന്നിങ്ങനെ മറ്റ് ഒരുപാട് രക്തസാക്ഷികളും അവരുടെ കൂട്ടത്തിൽ ഉണ്ടായിരുന്നു. (Brock and S. A. Harvey, Holy Women of the Syrian Orient, Berkeley, 1998, pp. 68-77) (https://iranicaonline.org/articles/shapur-ii )

സൊസോമന്റെ Ecclesiastical History ൽ, ‘ശാഹ്പോർ’ രണ്ടാമന്റെ കീഴിൽ രക്തസാക്ഷികളായ പേർഷ്യൻ ക്രിസ്ത്യാനികളെക്കുറിച്ച് ഗണ്യമായ വിശദാംശങ്ങൾ അടങ്ങിയിരിക്കുന്നു: “ഈ കാലയളവിൽ രക്തസാക്ഷികളായ പുരുഷന്മാരുടെയും സ്ത്രീകളുടെയും പേരുകളിൽ -സ്ഥിരീകരിക്കപ്പെട്ടവരുടെ എണ്ണം- പതിനാറായിരത്തിലധികമായി കണക്കാക്കപ്പെടുന്നു. എന്നാൽ ഈ കാലയളവിൽ പേരറിയാത്ത, രക്തസാക്ഷികളായ സ്ത്രീകളുടെയും പുരുഷന്മാരുടെയും എണ്ണം പതിനാറായിരത്തിലും എത്രയോ അധികമായതിനാൽ പേർഷ്യക്കാരും, സിറിയക്കാരും, എഡെസ്സ നിവാസികളും, എത്ര ശ്രമിച്ചിട്ടും ആ കണക്ക് നിജപ്പെടുത്താൻ സാധിക്കാതെ പരാജയമടഞ്ഞു.” ( Ecclesiastical History, Book II, Chapter XIV )

ഇസ്‌ലാമിന്റെ ആവിർഭാവത്തിനു മുമ്പ് തന്നെ പേർഷ്യൻ റോമൻ സാമ്രാജ്യങ്ങൾ മത സ്വേച്ഛാധിപത്യത്തിൽ കേളികേട്ടവരായിരുന്നു. ഇരു സാമ്രാജ്യങ്ങളേയും ആദർശപരമായി നയിച്ചിരുന്ന സൊറോസ്ട്രിയർ, ക്രിസ്ത്യാനികൾ, ജൂതന്മാർ തുടങ്ങി വ്യത്യസ്ഥ ആദർശവിശ്വാസികൾ – അവർ ഓരോരുത്തർക്കും ലഭ്യമാകുന്ന രാഷ്ട്രീയ അനുകൂലാവസ്ഥക്ക് അനുസൃതമായി- നടപ്പാക്കിയിരുന്ന അന്യമത ധ്വംസനങ്ങളും ഭരണ വടം വലികളും അന്നത്തെ ലോക വ്യവസ്ഥയിലെ അസഹിഷ്ണുതയുടെ ആഴം വ്യക്തമാക്കിത്തരുന്നുണ്ട്.

ഇസ്‌ലാംഭരണത്തിനു മുമ്പുള്ള പേർഷ്യയിലെ അവസാനസാമ്രജ്യമായിരുന്ന സാസാനിയൻ സാമ്രാജ്യത്തിന്റെ കാലത്ത് സൊറോസ്ട്രിയൻ മതം സാമ്രാജ്യത്തിലെ ഔദ്യോഗികമതമായിരുന്നു. കൂടാതെ ഈ സമയത്ത് സൊറോസ്ട്രിയൻ മതം സംഘടനാരൂപം കൈവരിക്കുകയും ചെയ്തു.

സാസാനിയൻ കാലഘട്ടത്തിലെ സൊറാസ്ട്രിയനിസം (224-651) പേർഷ്യൻ ചക്രവർത്തിയായ ശഹൻ-ശായുടെ (രാജാധി രാജൻ) രക്ഷാകർതൃത്വം ആസ്വദിച്ചു പോന്നു.

ഹഖാമനീഷിയാൻ പേർഷ്യൻ രാജാക്കന്മാരുടെ മഹത്വത്തിന്റെ നേരിട്ടുള്ള അവകാശികളായി സ്വയം കണ്ട സാസാനിയൻ സാമ്രാജ്യത്വ ഭരണകൂടം മസ്ദ-ആരാധനയിൽ അധിഷ്ടിതമായ, സോറോസ്ട്രിയൻ വിശ്വാസത്തിന്റെ സംരക്ഷകരായി സ്വയം അവരോധിച്ചു. (Zoroastrians and Christians in Sasanian Iran: A.V Williams: Department of Religions and Theology, University of Manchester: Page: 2)

ഈ കാലഘട്ടത്തിൽ അഹ്രിമാനും ദേവുകളും വലിയ പ്രഹരവും ഉപദ്രവവും ഭരണകൂടത്തിൽ നിന്ന് നേരിട്ടു. അവരുടെ വിശ്വാസങ്ങൾ ഭൂമിയിൽ നിന്ന് പുറത്താക്കപ്പെടുകയും വിശ്വാസ്യത നഷ്ടപ്പെടുകയും ചെയ്തു. യഹൂദരും ബുദ്ധമതക്കാരും ബ്രാഹ്മണരും അരാമിക്, ഗ്രീക്ക് ഭാഷകൾ സംസാരിക്കുന്ന ക്രിസ്ത്യാനികളും സ്നാപനമേറ്റവരും മനിക്കേയന്മാരും പേർഷ്യൻ ഭൂമിയിൽ ആക്രമിക്കപ്പെട്ടു. ചിത്രങ്ങൾ തകർക്കപ്പെട്ടു, ആരാധനാലയങ്ങളും വാസസ്ഥലങ്ങളും നശിപ്പിക്കപ്പെട്ടു, അഗ്നിക്ഷേത്രങ്ങൾ ധാരാളമായി സ്ഥാപിക്കപ്പെട്ടു. (Mary Boyce, Textual sources for the study of Zoroastrianism (Manchester: Manchester University Press, 1984), 112, translating from M. Back, Die sassanidischen Staatsinschriften (Acta Iranica, 18, 1978), 384ff.)

ഏറ്റവും കൂടുതൽ കുപ്രസിദ്ധമായ പീഡന പരമ്പരയുടെ കണക്കുകൾ ശാഹ്പോർ രണ്ടാമന്റെ ഭരണകാലഘട്ടത്തിൽ (309-79) നിന്നുള്ള ഇരുപത്തിയൊമ്പത് രക്തസാക്ഷികളാണ്. മുഖ്യ ആത്മീയ നേതാവായ അദുർബാദിന്റെ നേതൃത്വത്തിൽ ക്രിസ്ത്യാനികളെ നാൽപത് വർഷത്തോളം സുസ്ഥിരവും നിഷ്കരുണവും ദംശിച്ചു കൊണ്ടിരുന്നു. കോൺസ്റ്റന്റൈൻ ക്രിസ്തുമതം സ്വീകരിച്ച് ക്രിസ്തുമതത്തെ റോമൻ സാമ്രാജ്യത്തിന്റെ മതമാക്കിയതിന് തൊട്ടുപിന്നാലെയാണ് പീഡനത്തിന്റെ തുടക്കം.

ക്രിസ്ത്യൻ പീഡനത്തിന്റെ രണ്ടാം കാലഘട്ടം യാസ്ദെജർദ് ഒന്നാമന്റെയും (399-421) പിന്നീടുള്ള വർഷങ്ങളിൽ അദ്ദേഹത്തിന്റെ പിൻഗാമിയായ ബഹ്റാം അഞ്ചാമന്റെയും ആദ്യ വർഷങ്ങളിലുമായിരുന്നു (c. 421-39).

മൂന്നാമതായി, യാസ്ദെജർദ് രണ്ടാമന്റെ (439-57) ഭരണകാലത്ത്, ക്രിസ്ത്യൻ വിരുദ്ധ മത വിദ്വേഷം പീഡനവും വർദ്ധിച്ചു. ക്രിസ്ത്യൻ അർമേനിയയെ സൊറോസ്ട്രിയനിസത്തിലേക്ക് നിർബന്ധ മതപരിവർത്തനം ചെയ്യാനുളള ശ്രമം ശക്തിയാർജിച്ചു.

പേർഷ്യൻ ക്രിസ്ത്യാനികളുടെ, സുറിയാനി ക്രിസ്ത്യൻ രക്തസാക്ഷികളുടെ പ്രവർത്തനങ്ങളെ (Acts of the Persian Martyrs) സംബന്ധിച്ച രേഖകളിൽ പീഡനവും താഢനവും രക്തസാക്ഷിത്വവും വികാരഭരിതമായ വാക്കുകളിൽ രേഖപ്പെടുത്തപ്പെട്ടിട്ടുണ്ട്. (Zoroastrians and christians in Sasanian Iran: A.V Williams: Department of Religions and Theology, University of Manchester: Page: 4)

സോറാസ്ട്രിയൻ പുരോഹിതന്മാർ സാധാരണക്കാരോടൊപ്പം ഭക്ഷണം കഴിക്കുന്നത് ഒഴിവാക്കണം എന്ന ശക്തമായ കൽപ്പന പുറപ്പെടുവിക്കപ്പെട്ടു. ഒരു അവിശ്വാസിക്ക് എന്തെങ്കിലും ദാനം നൽകുന്നത് നിരോധിച്ചുകൊണ്ടുള്ള സോറോസ്ട്രിയൻ മതശാസനകൾ ഇറക്കപ്പെട്ടു. മരിക്കുമെന്ന് നിങ്ങൾ കരുതുന്നില്ലെങ്കിൽ അല്ലാതെ, ഒരു അവിശ്വാസിയ്ക്ക് നിങ്ങൾ ഭക്ഷണമുൾപ്പെടെ ഒന്നും നൽകരുത് എന്നായിരുന്നു പുരോഹിത ശാസന. (Pahlavi Rivayat, ed. Williams, 1990, 14.7, voL 2, 27.)

ശക്തനായ ഒരു പ്രഭു, ഉദാഹരണത്തിന് ഗിർഡർ, രാജാവിന്റെ മേൽ സ്വാധീനം നേടുമ്പോളെല്ലാം, പല കാരണങ്ങളും ആരോപിച്ച് ക്രിസ്ത്യാനികളും മറ്റ് ന്യൂനപക്ഷങ്ങളും രാഷ്ട്രീയവും മതപരവുമായ ബലിയാടുകളാക്കപ്പെടുക എന്നത് ഒരു സ്ഥിരം പ്രതിഭാസമായിരുന്നു. (Zoroastrians and christians in Sasanian Iran: A.V Williams: Department of Religions and Theology, University of Manchester: Page: 8)

മത പരിത്യാഗം മഹാ കുറ്റകൃത്യമായി നിശ്ചയിക്കപ്പെട്ടു. ഭീഷണികളും പീഡനങ്ങളും ഉപയോഗിച്ച് ക്രിസ്തുമതത്തിൽ നിന്ന് സോറാസ്ട്രിയൻ മതത്തിലേക്ക് നിർബന്ധപൂർവ്വം വലിച്ചിഴക്കപ്പെടുന്ന വർണനകൾ ക്രിസ്ത്യൻ സാധുചരിത്രവർണ്ണന (hagiography) രേഖകൾ മുഴുവൻ നിറഞ്ഞു നിൽക്കുന്നു.

സൂര്യൻ, അഗ്നി, ജലം എന്നിവയെ ആരാധിക്കാൻ പല ക്രിസ്ത്യാനികളേയും സോറാസ്ട്രിയൻ പൗരോഹിത്യം വധഭീഷണിയിലൂടെയും മർദ്ദനങ്ങളിലൂടെയും നിർബന്ധിച്ചു കൊണ്ടിരുന്നു.

ഈസ്റ്റ് ചർച്ച് കത്തോലിക്കരുടെയും ബിഷപ്പുമാരുടെയും തലവനായ ‘മാർ സിമൺ ബാർ സബ്ബാ’യുടെ വധത്തിന് കാരണം, ക്രിസ്ത്യാനികൾക്ക് ഇരട്ട നികുതി ചുമത്താൻ വിസമ്മതിച്ചതാണ്. സിമണേയും അദ്ദേഹത്തിന്റെ എല്ലാ സഹോദരന്മാരെയും വാളാൽ ശിരസ്സു ചേദിച്ച് വധിക്കാൻ രാജ കൽപ്പന നൽകപ്പെട്ടു. ഈ കൂട്ടകൊലക്ക് തൊട്ടു മുമ്പ് സോറാസ്ട്രിയൻ മുഖ്യ പുരോഹിതൻ ഇരകളോട് ഇപ്രകാരം ആവശ്യപ്പെടുകയുണ്ടായി: “ദൈവമായ സൂര്യനെ ആരാധിക്കുക, രാജാക്കന്മാരുടെ രാജാവും എല്ലാ കർത്താക്കളുടെയും കർത്താവുമായ ‘ശാഹ്പോറി’ന്റെ ഹിതം അനുസരിച്ച് പ്രവർത്തിക്കുക, എങ്കിൽ നിങ്ങൾക്ക് ജീവിക്കാം.” അപ്പോൾ അവർ എല്ലാവരും ഉറക്കെ മറുപടി നൽകി: “തീർച്ചയായും ഞങ്ങൾ രാജാക്കന്മാരുടെ രാജാവും മുഴുവൻ ഭൂമിയുടെയും അധിപനും, നിത്യതയുടെ ഉടമയുമായ (സാക്ഷാൽ ദൈവത്തിന്റെ) ഹിതം പ്രവർത്തിക്കും.”

സിമണിന്റെ രക്തസാക്ഷിത്വത്തിന് സാക്ഷ്യം വഹിച്ച പുസൈ ക്രിസ്ത്യൻ യുവാവ്, മറ്റൊരു ക്രിസ്ത്യൻ വയോധികനെ വധിക്കുന്ന ചടങ്ങിൽ വെച്ച് പരസ്യമായി പ്രതികരിക്കുകയുണ്ടായി. ഇത് പേർഷ്യൻ രാജാധിരാജനോടുള്ള ധിക്കാരമായി പരിഗണിക്കപ്പെട്ടു. പുസൈയെ ചങ്ങലകളിൽ വലിച്ചിഴക്കുകയും ക്രിസ്തുമതം ഉപേക്ഷിക്കാൻ നിർബന്ധിക്കുകയും ചെയ്തു. വിസമ്മതിച്ചപ്പോൾ വധശിക്ഷക്ക് വിധേയമാക്കി. മാർ അബാ, മാർ ഗിവർഗീസ് എന്നിവരും ക്രിസ്തുമത രക്തസാക്ഷികളിൽ എണ്ണപ്പെടുന്നു. (Zoroastrians and christians in Sasanian Iran: A.V Williams: Department of Religions and Theology, University of Manchester: Page: 12)

പേർഷ്യൻ സാമ്രാജ്യത്വത്തിന്റെ ക്രൂര രാഷ്ട്രീയത്തിന്റെ മറ്റൊരു ഉദാഹരണമാണ് വലേറിയൻ ചക്രവർത്തിയുടെ വധം: റോമൻ ചക്രവർത്തിയായ (ക്രിസ്ത്യൻ വിരുദ്ധനായിരുന്നു) വലേറിയൻ പണം വാഗ്ദാനം ചെയ്ത് സാസാനിയൻമാരുമായുള്ള യുദ്ധം അവസാനിപ്പിക്കാൻ ആഗ്രഹിച്ചു. സാസാനിയൻ രാജാവായ ശാഹ്പോർ ഒന്നാമൻ -ചക്രവർത്തി നേരിട്ട് വന്ന് അദ്ദേഹത്തോട് അഭ്യർത്ഥിക്കാൻ ആഗ്രഹിക്കുന്നു എന്ന സന്ദേശവുമായി- അംബാസഡർമാരെ വലേറിയനിലേക്ക് തിരിച്ചയച്ചു. വലേറിയൻ ഒരു ചെറിയ സംഘത്തോടൊപ്പം ശാഹ്പോറിനടുത്തേക്ക് ചെന്നപ്പോൾ, അദ്ദേഹത്തെ സാസാനിയക്കാർ പിടികൂടി. ബൈസന്റൈൻ ചരിത്രകാരനായ സോസിമസിന്റെ അഭിപ്രായത്തിൽ, പേർഷ്യക്കാർക്കിടയിൽ ഒരു അടിമയെന്ന നിലയിൽ വലേറിയൻ തന്റെ ജീവിതത്തോട് വിട പറഞ്ഞു. പുരാതന ക്രിസ്ത്യൻ എഴുത്തുകാരനും കോൺസ്റ്റന്റൈൻ ഒന്നാമന്റെ ഉപദേശകനുമായ ലാക്റ്റൻഷ്യസ് വാദിക്കുന്നത്, ശാഹ്പോർ ഒന്നാമൻ കുതിരപ്പുറത്ത് കയറാൻ ആഗ്രഹിക്കുമ്പോഴെല്ലാം വലേറിയൻ ചക്രവർത്തിയെ മനുഷ്യ പാദപീഠമായി ഉപയോഗിച്ചിരുന്നു എന്നാണ്.” അദ്ദേഹത്തെ കൊന്നുകളയുകയും, അദ്ദേഹത്തിന്റെ തൊലി മാംസത്തിൽ നിന്ന് ഉരിച്ച് വേർപെടുത്തി, സിന്ദൂരം കൊണ്ട് ചായം പൂശി, ബാർബേറിയൻമാരുടെ ദൈവങ്ങളുടെ ക്ഷേത്രത്തിൽ പ്രതിഷ്ടിക്കുകയും ചെയ്തു” എന്നും ലാക്റ്റൻഷ്യസ് വാദിക്കുന്നു. (https://www.ancient-origins.net/history/what-really-happened-valerian-was-roman-emperor-humiliated-and-skinned-hands-enemy-008598)

വലേറിയന്റെ തൊണ്ടയിലേക്ക് ഉരുക്കിയ സ്വർണ്ണ ദ്രാവകം ഒഴിച്ചാണ് പേർഷ്യക്കാർ വധിച്ചതെന്നും പറയപ്പെടുന്നു.

പേർഷ്യൻ സാമ്രാജ്യം ജൂതവിരുദ്ധമായിരുന്ന കാലഘട്ടത്തിൽ ആദർശ അസഹിഷ്ണുതയുടെ പേരിൽ ജൂതന്മാർ മത ദ്വംസനങ്ങൾക്കും അക്രമങ്ങൾക്കും പാത്രമായി. പഴയനിയമത്തിന്റെ ഭാഗമായ ഗ്രന്ഥങ്ങളിൽ ഒന്നായ ‘എസ്തേറിന്റെ പുസ്തക’ത്തിൽ ‘പൂരിം’ എന്ന പേരിൽ അറിയപ്പെടുന്ന ജൂതാഘോഷത്തിനു പിന്നിലുള്ള കഥ പ്രതിപാദിക്കുന്നുണ്ട്. പൂരിം തിരുനാളിലെ സായാഹ്നത്തിലും അടുത്ത പ്രഭാതത്തിലും ഈ പുസ്തകത്തിന്റെ സമ്പൂർണ്ണപാഠം ജൂതർ ഉറക്കെ വായിക്കുന്നു. പേർഷ്യൻ രാജാവിന്റെ കൊട്ടാരത്തിൽ, ഹാമാന്റെ നേതൃത്വത്തിൽ ജൂതർക്കെതിരെ നടന്ന ഒരു ഉപജാപത്തിന്റേയും, തൽഫലമായി ഉണ്ടാകാനിരുന്ന വംശഹത്യയിൽ നിന്ന്, ജൂതന്മാർ രക്ഷപെടുന്നതിന്റേയും കഥയാണിത്. (The Religious Policy of Xerxes and the “Book of Esther” Robert J. Littman)

ഒരു ജൂത ഉദ്യോഗസ്ഥനായ മൊർദ്ദെകായ്, രാജാവിന്റെ ഉന്നത സഹായിയായ ഹാമാന്റെ മുമ്പിൽ തലകുനിച്ച് വണങ്ങാൻ വിസമ്മതിച്ചു എന്ന കാരണത്താലാണ് രാജ്യത്തെ എല്ലാ ജൂതന്മാരെയും ഉന്മൂലനം ചെയ്യാൻ രാജാവ് പദ്ധതിയിട്ടത്. യഹൂദ മതം ഒരു ജൂതനും, ദൈവമല്ലാതെ മറ്റേതെങ്കിലും വ്യക്തിയെയോ വിഗ്രഹത്തെയോ തലകുനിച്ച് വണങ്ങാൻ അനുവദിക്കുന്നില്ല.

ലബ്‌നാനിലെ പ്രസിദ്ധ (ക്രിസ്ത്യൻ) സാഹിത്യകാരനും ചരിത്രകാരനുമായ ജോർജി സൈദാൻ എഴുതി: പേർഷ്യയും റോമും തമ്മിലുളള ശത്രുത പൗരാണികമാണ്. ഒരു പക്ഷെ ബി.സി അഞ്ചാം നൂറ്റാണ്ടിനുമപ്പുറം അതിന്റെ വേരുകൾ എത്തി നിൽക്കുന്നുണ്ടാകാം. ലോകത്തെ അടക്കി ഭരിക്കാനുള്ള ഇരു സാമ്രാജ്യങ്ങളുടെയും അത്യാഗ്രഹമായിരുന്നു ഈ ശത്രുതയുടെ അടിത്തറ. നൂറ്റാണ്ടുകളോളം നീണ്ടു നിന്ന ഈ അധികാര വടം വലി ഇസ്‌ലാമിന്റെ ആവിർഭാവ കാലഘട്ടത്തിലും തുടർക്കഥയായിരുന്നു. പേർഷ്യൻ സാമ്രാജ്യത്തിന്റെ ആധിപത്യം ഇസ്ര അനൂഷർവാൻ ചക്രവർത്തിയിൽ എത്തിച്ചേർന്നപ്പോൾ റോമൻ സാമ്രാജ്യത്തെ അൽപാൽപ്പമായി പിടിച്ചടക്കാൻ അദ്ദേഹം സൈന്യ വ്യൂഹത്തെ വിന്യസിച്ചു. സിറിയ പിടിച്ചടക്കുകയും അന്താഖിയ ചുട്ടു നശിപ്പിക്കുകയും ഏഷ്യാ മൈനർ കൊള്ളയടിക്കുകയും ചെയ്തു. അന്നത്തെ റോമൻ ചക്രവർത്തിയായിരുന്ന ജസ്റ്റീനിയൻ ചക്രവർത്തിയും വിട്ടുവീഴ്ചയ്ക്ക് തയ്യാറല്ലായിരുന്നു. ക്രിസ്താബ്ദം 541 മുതൽ 561 വരെ ഇരുപതു വർഷം ഇരു രാഷ്ട്രങ്ങളും യുദ്ധത്തിൽ മുഴുകി.

പർവേസ് ചക്രവർത്തിയുടെ കാലഘട്ടത്തിൽ തന്റെ സുഹൃത്ത് മോറിസിന്റെ കൊലപാതകത്തിന് പ്രതികാരമെന്ന പേരിൽ റോമൻ സാമ്രാജ്യത്തെ പർവേസ് ചക്രവർത്തി ആക്രമിക്കുകയുണ്ടായി. ക്രിസ്താബ്ദം 614 ൽ സിറിയ പിടിച്ചടക്കി…

(പേർഷ്യയിൽ ജൂതന്മാർക്ക് സ്വാധീനമുണ്ടായിരുന്ന കാലത്ത്) ജൂതന്മാരുടെ അകമഴിഞ്ഞ സഹായത്താൽ ബൈസാന്റിയൻ പടയെ പർവേസ് ചക്രവർത്തി ഒന്നൊന്നായി കീഴടക്കി. ഈജിപ്ത്, അന്താഖിയ, ദമാസ്ക്കസ്, ബൈത്തുൽ മുഖദസ്‌ തുടങ്ങിയവ പിടിച്ചടക്കി. ജറുസലേമിലെ ബൈത്തുൽ മുഖദസ്‌ കൊള്ളയടിക്കാനും ക്രിസ്ത്യൻ പള്ളികളും പുണ്യപുരുഷന്മാരുടെ കല്ലറകൾ തീയിടാനും അവിടെയുള്ള വിലമതിക്കാനാകാത്ത സ്വത്തുക്കൾ പിടിച്ചു പറിക്കാനും തന്റെ സൈന്യത്തിന് പർവേസ് ചക്രവർത്തി അനുവാദം നൽകി. സിറിയ വരെ ഈ കൊലയും കൊള്ളയും തുടർന്നു. 90000 ക്രിസ്ത്യാനികളെ സൈന്യം കൊന്നൊടുക്കി… ഇതെല്ലാം കണ്ടിട്ടും കാണാത്ത മട്ടിൽ ഭീരുവായി (അന്നത്തെ) ഹെറാക്ലിയസ് ചക്രവർത്തി കൊട്ടാരത്തിൽ തന്നെ ഇരുന്നു; ക്രിസ്താബ്ദം 632 ൽ ഏഷ്യാ മൈനറിൽ വെച്ച് കൊല്ലപ്പെടുന്നത് വരെ… (താരീഖുത്തമദ്ദുനുൽ ഇസ്‌ലാമി:ജോർജി സൈദാൻ: 1: 43-48)

ഹെറാക്ലിയസിന്റെ കാലഘട്ടത്തിൽ ജൂതന്മാർ തങ്ങളുടെ സഹായികളായ പേർഷ്യക്കാരിൽ നിന്നും 80000 ക്രിസ്ത്യൻ ബന്ദികളെ വിലയ്ക്ക് വാങ്ങി അറുത്തു കൊന്നു… (താരീഖുത്തമദ്ദുനുൽ ഇസ്‌ലാമി: ജോർജി സൈദാൻ: 1: 43-48)

ഈ രാഷ്ട്രീയ വ്യവസ്ഥയിലെ, പേർഷ്യൻ അധിനിവേശത്തിനു കീഴിലായിരുന്ന ‘ബസ്വറ’യോടാണ് (ഹിന്ദ്) മുഹമ്മദ് നബി (സ) യുദ്ധത്തിന് ആഹ്വാനം ചെയ്തത്. ഈ യുദ്ധ ആഹ്വാനത്തിന് പശ്ചാത്തലവുമായി ഗാഢമായ ബന്ധമുണ്ട് എന്ന് വ്യക്തമാണല്ലൊ. ഏതെങ്കിലും ഒരു രാഷ്ട്ര നേതാവ് ഒരു രാജ്യത്തോട് ഒരു പ്രത്യേക പശ്ചാത്തലത്തിലും സാഹചര്യത്തിലും യുദ്ധത്തിന് പ്രോത്സാഹനം നൽകിയ ഒരു വാചകം എടുത്തു വെച്ച്, അവ എക്കാലത്തേക്കുമുള്ള കൽപ്പനയായി അവതരിപ്പിക്കുന്നതിലെ ദുരുദ്ദേശം തിരിച്ചറിയപ്പെടണം. ബ്രിട്ടീഷ് സാമ്രാജ്യത്വ അധിനിവേശത്തോട് സമരവും യുദ്ധവും പ്രഖ്യാപിച്ച് കൊണ്ടുള്ള ഇന്ത്യൻ സ്വാതന്ത്ര്യ സമര നേതാക്കളുടെ വാചകങ്ങൾ എടുത്തുദ്ദരിച്ച് ഇന്നത്തെ, അല്ലെങ്കിൽ എക്കാലത്തേയും ബ്രിട്ടനോട് ശത്രുത വെച്ചുപുലർത്താനാണ് അവയുടെ ഉൾസാരമെന്ന് ദുർവ്യാഖ്യാനിക്കുന്നതു പോലെ തികച്ചും ബാലിശമാണ് ഈ ഹദീസ് ദുർവ്യാഖ്യാനം.

ഹദീസിൽ ഉദ്ദേശിക്കപ്പെട്ട ഹിന്ദ് അഥവാ ബസ്വറ നൂറ്റാണ്ടുകളായി മുസ്‌ലിംകളുടേതായി കഴിഞ്ഞു. അപ്പോൾ ബസ്വറയോടുള്ള യുദ്ധ കൽപ്പന എക്കാലത്തേയും ബാധിക്കുന്ന, പൊതു കൽപനയാണെന്ന് ദുർവ്യഖ്യാനിച്ചാൽ ‘മുസ്‌ലിം ഇറാഖി’നോട് തന്നെ യുദ്ധം ചെയ്യാനാണ് മുഹമ്മദ് നബി (സ) കൽപ്പിച്ചത് എന്ന് ഹദീസിന് വിരുദ്ധാത്ഥം കൽപ്പിക്കേണ്ടി വരില്ലേ ?! ഇസ്‌ലാം മുഹമ്മദ് നബിയിലൂടെ കടന്നുവന്ന കാലഘട്ടത്തിൽ ലോകത്ത് നിലനിന്നിരുന്ന രാഷ്ട്രീയ വ്യവസ്ഥയെ സംബന്ധിച്ച് ചരിത്രത്തിന്റെ താളുകളിലൂടെ അൽപ്പമെങ്കിലും കണ്ണോടിച്ചാൽ ജനാധിപത്യ മൂല്യങ്ങളും മത സഹവർത്തിത്വത്തിലും ഊന്നി, ഇന്ന് ലോകത്ത് നിലനിൽക്കുന്ന രാഷ്ട്ര ഘടനയായിരുന്നില്ല അന്നത്തേത് എന്ന് മനസ്സിലാക്കാവുന്നതെയുള്ളു.

അന്യ മതങ്ങളേയോ ആദർശങ്ങളേയോ സംസ്കാരങ്ങളേയോ വെച്ചുപൊറുപ്പിക്കാത്ത സ്വേച്ചാധിപത്യ ഭരണകൂടങ്ങളായിരുന്നു ലോകത്ത് ഭൂരിഭാഗവും രാജ്യങ്ങളിലും നിലനിന്നിരുന്നത്. ജനാധിപത്യമോ മത സഹിഷ്ണുതയോ തൊട്ടു തീണ്ടിയിട്ടില്ലാത്ത കിരാത വാഴ്ച്ച. ഇതാണ് ഇസ്‌ലാമിന്റെ ആവിർഭാവ ഭൂമിക. ഈ സാമൂഹിക രാഷ്ട്രീയ പശ്ചാത്തലത്തിൽ ഊന്നി നിന്നു കൊണ്ടേ ഇസ്‌ലാമിലെ യുദ്ധങ്ങളെയും യുദ്ധ ആഹ്വാനങ്ങളേയും വായിച്ചറിയാവൂ. പ്രതിരോധത്തിനും മത സ്വാതന്ത്ര്യത്തിനും വേണ്ടിയായിരുന്നു ഇസ്‌ലാമിക യുദ്ധങ്ങൾ മുഴുവനും. അതല്ലാതെ ഇന്ന് ലോകത്ത് നിലനിൽക്കുന്ന മതേതര സ്വഭാവമോ ആദർശ സഹവർത്തിത്വമോ രാഷ്ട്രീയ പ്രകൃതിയായി സ്വീകരിച്ച സാമൂഹിക പശ്ചാത്തലത്തോടായിരുന്നില്ല പ്രസ്തുത സമരങ്ങൾ. ഈ സമൂഹത്തോട് അവകാശങ്ങൾക്കും സ്വാതന്ത്ര്യത്തിനും വേണ്ടി സായുധ സമരത്തിൽ ഏർപ്പെടുകയല്ലാതെ മറ്റെന്താണ് മുസ്‌ലിംകളുടെ – ആവിർഭാവ ഘട്ടത്തിൽ – മുമ്പിലുണ്ടായിരുന്ന വഴി !!

പ്രതിരോധ യുദ്ധങ്ങളല്ലാത്ത ഇസ്‌ലാമിക യുദ്ധങ്ങൾ തന്നെയും നിർബന്ധിത മതപരിവർത്തനത്തിനോ അന്യ മത ധ്വംസനത്തിനോ വേണ്ടിയല്ല നടന്നത് എന്നർത്ഥം. മറിച്ച്, മത സ്വാതന്ത്ര്യത്തിനും അവകാശങ്ങൾക്കും വേണ്ടിയാണ്. ഇസ്‌ലാമിക ഭരണത്തിനു കീഴിൽ കൊണ്ടുവന്ന് അന്യ മത സമൂഹങ്ങളെ അടക്കിഭരിക്കുകയല്ല ലക്ഷ്യം. മറിച്ച്, അന്നത്തെ തിയോക്രാറ്റിക്ക് ഓട്ടോക്രസികളിൽ (തീവ്രമതവികാരത്തിൽ അധിഷ്ഠിതമായ സ്വേച്ഛാധിപത്യ ഭരണകൂടങ്ങൾ) ഇസ്‌ലാമിൽ ആകൃഷ്ടരാകുന്നവർക്ക് ഇസ്‌ലാം സ്വീകരിക്കാനുള്ള വഴിയൊരുക്കാനും ഇസ്‌ലാം അനുസരിച്ചുള്ള ജീവിതം എളുപ്പമാവാനും ഇസ്‌ലാമിക പ്രബോധനം സാധ്യമാകാനും അവരുടെ മേൽ ഭരണം നേടിയെടുക്കുകയല്ലാതെ മറ്റൊരു വഴിയും അന്നത്തെ രാഷ്ട്രീയ ഘടനയിൽ നില നിന്നിരുന്നില്ല. ഇസ്‌ലാമിന്റെ കടന്നുവരവിനെ ആ കാലഘട്ടത്തിലെ ഗോത്ര സമൂഹങ്ങളും നാട്ടുരാജ്യങ്ങളും മത സമുദായങ്ങളും എത്രത്തോളം അസഹിഷ്ണുതയോടെയാണ് സ്വീകരിച്ചത് എന്ന ചരിത്രസാക്ഷ്യങ്ങൾ മുമ്പ് പല ലേഖനങ്ങളിൽ നാം വിശദീകരിച്ചിട്ടുണ്ട്. (വിശദാശംങ്ങൾക്കായി: www.snehasamvadam.org: ‘ഇസ്‌ലാമിന്റെ യുദ്ധഭൂമിക’ എന്ന ലേഖനം വായിക്കുക: https://bit.ly/3BHqSuX)

ഇത്തരമൊരു ലോക വ്യവസ്ഥയിൽ ഇസ്‌ലാമിന്റെ അതിജീവനവും പ്രചാരണവും മറ്റെന്തു മാർഗത്തിലൂടെയാണ് സാധ്യമാകുമായിരുന്നത്? ഇത്തരമൊരു ഭൂമികയിൽ സമ്പൂർണ അഹിംസ രാഷ്ട്രീയ നയമായി ഇസ്‌ലാം സ്വീകരിച്ചിരുന്നുവെങ്കിൽ ഇതര സമൂഹങ്ങൾ ഇസ്‌ലാമിനെ വിഴുങ്ങുമായിരുന്നു. ഒരു നൂറ്റാണ്ടിനപ്പുറം വെളിച്ചം കാണാതെ അജ്ഞതയുടെ അഗണ്യകോടിയിൽ അലിഞ്ഞില്ലാതെയാവുമായിരുന്നു (;ദൈവത്തിൽ നിന്നുള്ള മതമായിരുന്നില്ല ഇസ്‌ലാം എങ്കിൽ.)

പക്ഷെ ഇസ്‌ലാം ദൈവത്തിൽ നിന്നുള്ള ആദർശമായത് കൊണ്ടും പ്രാവർത്തിക മതമായത് കൊണ്ടും അനിവാര്യ ഘട്ടങ്ങളിലുള്ള സായുധ സമരങ്ങൾക്ക് അനുവാദം നൽകപ്പെട്ടു. ഇസ്‌ലാം ദൈവത്തിൽ നിന്നുള്ള സന്മാർഗ സന്ദേശമാണ്. അത് ലോകം മുഴുവനും – കുടിലും കൊട്ടാരവും വ്യത്യാസമില്ലാതെ – എത്തിക്കുക എന്നത് ദൈവത്തിന്റെ ബാധ്യതയാണല്ലോ. അവൻ സ്വയം ഏറ്റെടുത്ത ബാധ്യത. അതിന് സാമൂഹിക അസഹിഷ്ണുതയും രാഷ്ട്രീയ തടസ്സങ്ങളും വിഘ്നങ്ങളായി കൂടാ. സത്യത്തിന്റെ ശത്രുക്കൾ രാഷ്ട്രങ്ങളുടേയും നാട്ടുരാജ്യങ്ങളുടേയും അതിർത്തികളിൽ വാളേന്തി നിൽക്കുകയും വാതിലുകൾ കൊട്ടിയടക്കുകയും ചെയ്തിരുന്ന കാലഘട്ടത്തിൽ വാളുകൊണ്ടല്ലാതെ ആ തടസ്സത്തെ നീക്കാൻ കഴിയില്ലായിരുന്നു. ഇന്നത്തെ ലോക-രാഷ്ട്ര വ്യവസ്ഥ അന്നത്തേതല്ല. രാഷ്ട്രങ്ങളുടെ വാതിലുകൾ ഏത് ആദർശങ്ങൾക്കും മതങ്ങൾക്കും മുമ്പിൽ – താത്ത്വികമായെങ്കിലും – മലർക്കേ തുറന്നിടപ്പെട്ടിരിക്കുകയാണ്, ഇന്ന്. ഇവിടെ, വാതിലുകളുടെ കാവൽക്കാർ വാളേന്തി നിൽക്കുന്നില്ല. പിന്നെയെന്തിന് മുസ്‌ലിംകൾ വാളെടുക്കണം ?!. വാതിലുകൾ കൊട്ടിയടക്കപ്പെട്ടാലല്ലേ ചവിട്ടി തുറക്കേണ്ടതുള്ളു ?!. തുറന്നു കിടക്കുന്ന വാതിൽ ആരെങ്കിലും ചവിട്ടി തുറക്കാൻ ശ്രമിക്കുമോ?!!

ഇസ്‌ലാമിക ഭരണത്തിനു കീഴിൽ പൂർണ്ണ മത സ്വാതന്ത്ര്യങ്ങളോടെയും അവകാശങ്ങളോടെയും ജീവിക്കാൻ ഒരു അമുസ്‌ലിമിന് വഴിയൊരുക്കണമെന്നാണ് മുഹമ്മദ് നബി (സ) അദ്ദേഹത്തിന്റെ അനുചരന്മാരെ പഠിപ്പിച്ചത് എന്ന് സാന്ദർഭികമായി സൂചിപ്പിക്കട്ടെ. ഇസ്‌ലാം മുന്നോട്ടുവെക്കുന്ന ഭരണ വ്യവസ്ഥ തീവ്രമതവികാരത്തിൽ അധിഷ്ഠിതമായ സ്വേച്ഛാധിപത്യ ഭരണകൂടമല്ല; ജനാധിപത്യ മൂല്യങ്ങളിൽ അധിഷ്ഠിതമായ ആത്മീയഭരണമാണ്. അവിടെയും ഇസ്‌ലാമിക ഭരണത്തിനു കീഴിൽ പ്രസ്തുത സമൂഹങ്ങളെ അടക്കിഭരിക്കുകയല്ല ലക്ഷ്യം; മുമ്പ് സൂചിപ്പിച്ചതു പോലെ അന്നത്തെ തിയോക്രാറ്റിക്ക് ഓട്ടോക്രസികളിൽ ജീവിക്കുന്ന മനുഷ്യർക്കിടയിൽ ഇസ്‌ലാം പ്രബോധനം ചെയ്യാനും, ഇസ്‌ലാം സ്വീകരിക്കാനും ഉള്ള വഴി വെട്ടുക മാത്രമാണ്.

‘ഹിന്ദി’നോടുള്ള യുദ്ധത്തെ സംബന്ധിച്ച പ്രവാചക പ്രവചനം കഴിഞ്ഞ കാലഘട്ടത്തിൽ പുലർന്നിട്ടില്ല എന്നും അത് ഭാവിയിൽ നടക്കാനിരിക്കുന്നതാണ് എന്നും ചില മുസ്‌ലിം പണ്ഡിതന്മാർ വ്യാഖ്യാനിക്കാറുണ്ട്. ഈ പണ്ഡിതാഭിപ്രായം അംഗീകരിച്ചാലും, സ്വതന്ത്രാനന്തര അതിർത്തിരേഖ ഉൾകൊള്ളുന്ന ആധുനിക മതേതര ജനാധിപത്യ ഇന്ത്യയോടുള്ള യുദ്ധമല്ല ഇതു കൊണ്ടും ഉദ്ദേശിക്കപ്പെടുന്നത്. ഈ പണ്ഡിത വ്യഖ്യാനം സ്വീകരിച്ചാലും ഇന്ത്യക്കെതിരെയുള്ള ഭീകരാക്രമണങ്ങൾ ഒരു സാഹചര്യത്തിലും ഈ ഹദീഥിൽ പ്രദിപാദിക്കുന്നില്ല. അന്ത്യനാളിനോടനുബന്ധിച്ച് തീർത്തും വ്യത്യസ്ഥമായ ഒരു ലോക ഘടനയിൽ, ലോകം മുഴുവൻ ബലപ്രയോഗത്തിലൂടെ അധിനിവേശം നടത്തുന്ന, മസീഹുദ്ദജ്ജാൽ എന്ന ഒരു ഏകശസനാധിപന്റെ കാലത്താണ് സംഭവം നടക്കുക. ഈ ഏകശാസകനിൽ നിന്നും ഹിന്ദിനെ മോചിപ്പിച്ച് സ്വാതന്ത്ര്യത്തിലേക്കും, നീതിയിലേക്കും തിരിച്ച് കൊണ്ടുപോകാനാണ് ഹിന്ദ് യുദ്ധം.

യുദ്ധം നടക്കുന്ന ലോകം നമ്മൾ ഇന്ന് ജീവിക്കുന്ന ലോകമല്ല. യുദ്ധം ചെയ്യപ്പെടുന്ന ‘ഹിന്ദ്’ ഇന്നത്തെ മതനിരപേക്ഷ ജനാധിപത്യ ഇന്ത്യയുമല്ല. ദജ്ജാലെന്ന ഏകാധിപതിയുടെ കീഴിൽ ഞെരിഞ്ഞമരുന്ന ഒരു ഹിന്ദിനോട് അന്ത്യദിനത്തോട് അടുത്തായി നടക്കുന്ന ഒരു യുദ്ധത്തിനെ (ഒരു സ്വാതന്ത്ര്യ സമരത്തിനെ) സ്വതന്ത്രാനന്തര അതിർത്തിരേഖ ഉൾകൊള്ളുന്ന ആധുനിക മതനിരപേക്ഷ ഇന്ത്യയോടുള്ള യുദ്ധമായി ദുർവ്യാഖ്യാനിക്കുകയാണ് ഇസ്‌ലാമോഫോബിക്കുകൾ ചെയ്തു കൊണ്ടിരിക്കുന്നത്. എന്നാൽ സമാധാന ചിത്തരായി ജീവിക്കുന്ന സാധാരണ അമുസ്‌ലിംകളോട് എന്ത് നിലപാടാണ് സ്വീകരിക്കേണ്ടത് എന്നതുമായി ബന്ധപ്പെട്ട ഇസ്‌ലാമിക നിയമവും പ്രവാചക പാഠവും മറ്റു ഹദീസുകളിൽ വ്യക്തമായി വായിക്കാൻ സാധിക്കും, അതിനോട് എതിരായി അന്യായമായി ഒരു നാടിനേയും ആക്രമിക്കാൻ പ്രവാചക ശ്രേഷ്ഠൻ (സ) ഒരിക്കലും കൽപ്പിക്കുകയില്ല.

അല്ലാഹു പറഞ്ഞു: “മതകാര്യത്തില്‍ നിങ്ങളോട് യുദ്ധം ചെയ്യാതിരിക്കുകയും, നിങ്ങളുടെ വീടുകളില്‍ നിന്ന് നിങ്ങളെ പുറത്താക്കാതിരിക്കുകയും ചെയ്യുന്നവരെ സംബന്ധിച്ചിടത്തോളം നിങ്ങളവര്‍ക്ക് നന്മ ചെയ്യുന്നതും നിങ്ങളവരോട് നീതി കാണിക്കുന്നതും അല്ലാഹു നിങ്ങളോട് നിരോധിക്കുന്നില്ല. തീര്‍ച്ചയായും അല്ലാഹു നീതി പാലിക്കുന്നവരെ ഇഷ്ടപ്പെടുന്നു. മതകാര്യത്തില്‍ നിങ്ങളോട് യുദ്ധം ചെയ്യുകയും നിങ്ങളുടെ വീടുകളില്‍ നിന്ന് നിങ്ങളെ പുറത്താക്കുകയും നിങ്ങളെ പുറത്താക്കുന്നതില്‍ പരസ്പരം സഹകരിക്കുകയും ചെയ്തവരെ സംബന്ധിച്ചു മാത്രമാണ് -അവരോട് മൈത്രികാണിക്കുന്നത്- അല്ലാഹു നിരോധിക്കുന്നത്‌. വല്ലവരും അവരോട് മൈത്രീ ബന്ധം പുലര്‍ത്തുന്ന പക്ഷം അവര്‍ തന്നെയാകുന്നു അക്രമകാരികള്‍.” (കുർആൻ: 60:8,9)

പ്രവാചകൻ മുഹമ്മദ് (സ) പറഞ്ഞു: مَنْ قَتَلَ مُعَاهَدًا لَمْ يَرَحْ رَائِحَةَ الْجَنَّةِ، وَإِنَّ رِيحَهَا لَيُوجَد مِنْ مَسِيرَةِ أَرْبَعِينَ عَامًا. “സമാധാന സന്ധിയിലുള്ള ഒരു അമുസ്‌ലിമിനെ ആരെങ്കിലും കൊന്നാൽ അവന് സ്വർഗത്തിന്റെ സുഗന്ധം പോലും ലഭിക്കില്ല.” (സ്വഹീഹുൽ ബുഖാരി: ഹദീസ് നമ്പർ: 3166)

عَنْ صَفْوَانَ بْنَ سُلَيْمٍ عَنْ رَسُولِ اللَّهِ صلى الله عليه وسلم قَالَ‏ “‏ أَلاَ مَنْ ظَلَمَ مُعَاهِدًا أَوِ انْتَقَصَهُ أَوْ كَلَّفَهُ فَوْقَ طَاقَتِهِ أَوْ أَخَذَ مِنْهُ شَيْئًا بِغَيْرِ طِيبِ نَفْسٍ فَأَنَا حَجِيجُهُ يَوْمَ الْقِيَامَةِ

സ്വഫ്‌വാനു ബ്‌നു സുലൈമില്‍(റ) നിന്ന് നിവേദനം: പ്രവാചകൻ (സ) പറഞ്ഞു: ‘അറിയണം, ആരെങ്കിലും സമാധാന സന്ധിയിലുള്ള അമുസ്‌ലിമിനെ ഉപദ്രവിക്കുകയോ, അവന് കിട്ടേണ്ട അവകാശങ്ങളില്‍ കുറവ് വരുത്തുകയോ, സാധ്യമാകുന്നതിലുപരി വഹിക്കാന്‍ അവനെ നിര്‍ബന്ധിക്കുകയോ, മനപ്പൊരുത്തമില്ലാതെ അവനില്‍ നിന്നും വല്ലതും കവര്‍ന്നെടുക്കുകയോ ചെയ്താൽ ഉയിര്‍ത്തെഴുന്നേല്‍പ്പ് നാളില്‍ ഞാന്‍ അവനുമായി (ആ അമുസ്‌ലിമിന്റെ അവകാശങ്ങളുടെ കാര്യത്തിൽ) തര്‍ക്കത്തിലേര്‍പെടും. (അബൂദാവൂദ്: 3052).

ഇതാണ് സമാധാന ചിത്തരായി ജീവിക്കുന്ന സാധാരണ അമുസ്‌ലിംകളോട് സ്വീകരിക്കേണ്ട നിലപാടായി ഇസ്‌ലാം പഠിപ്പിച്ച നിയമവും ശാസനയും. എത്ര തവണ വ്യക്തമാക്കിയാലും ഈ ആയത്തുകളും ഹദീസുകളും വിമർശകർ കാണാത്ത മട്ടാണ് !!

ദജ്ജാലിനോടും ദജ്ജാലിന്റെ അധിനിവേശത്തോടുമുള്ള യുദ്ധം അന്ത്യദിനത്തോടനുബന്ധിച്ചാണ്, ഈസാ നബിയുടെ(അ) (യേശു) നേതൃത്വത്തിലാണ് നടക്കുക. (ഫത്ഹുൽ ബാരി: 6: 610, അത്തൗദീഹു ലി ശർഹിൽ ജാമിഅ്: 17:663)

അന്ത്യനാളിനോടടുത്ത് പ്രത്യക്ഷനാകുമെന്ന് പല മത ഗ്രന്ഥങ്ങളും പ്രവചിച്ച, അന്തിക്രിസ്‌തു (antichrist) എന്ന പേരിൽ അറിയപ്പെടുന്ന ഒരു ഭാവി വ്യക്തിത്വമാണ് ‘മസീഹു ദ്ദജ്ജാൽ’. ലോകത്ത് ജീവിച്ചിരുന്നതും ജീവിച്ചിരിക്കുന്നവരുമായ ഏകാധിപതികളേക്കാൾ ഏറ്റവും കിരാതനും ക്രൂരനുമായ ഏകാധിപതിയായിരിക്കും (dictator) മസീഹു ദ്ദജ്ജാൽ എന്നാണ് ഹദീസുകൾ പഠിപ്പിക്കുന്നത്. മനുഷ്യരെ ഭൗതീകവും ആത്മീയവുമായ പാരതന്ത്ര്യത്തിലേക്ക് വലിച്ചിഴക്കുന്ന ഈ ദജ്ജാലിന്റെ ഉപദ്രവങ്ങളിൽ നിന്ന് രക്ഷ തേടാൻ പ്രവാചകൻ (സ) തന്റെ അനുചരന്മാരോട് പ്രത്യേകം ഉപദേശിക്കുക കൂടി ചെയ്തതായി കാണാം. (സ്വഹീഹു മുസ്‌ലിം: 924)

ദജ്ജാലിനാൽ ഭാവിയിൽ, ലോകത്ത് വിതക്കപ്പെടാനിരിക്കുന്ന കുഴപ്പങ്ങളും ഛിദ്രതകളും സമാനതകൾ ഇല്ലാത്തതായിരിക്കും.

مَا بَيْنَ خَلْقِ آدَمَ إِلَى أَنْ تَقُومَ السَّاعَةُ فِتْنَةٌ أَكْبَرُ مِنْ فِتْنَةِ الدَّجَّالِ . പ്രവാചകൻ (സ) പറഞ്ഞു: “(ആദ്യ മനുഷ്യൻ) ആദമിനെ സൃഷ്ടിച്ചതു മുതൽ അന്ത്യദിനം സംഭവിക്കുന്നതു വരെ ദജ്ജാലിന്റെ കുഴപ്പത്തേക്കാൾ വലിയ ഒരു ആപത്തും ഇല്ലതന്നെ.” (മുസ്നദു അഹ്മദ്: 15831)

ലോക രാജ്യങ്ങൾ മുഴുവൻ വെട്ടിപ്പിടിച്ച് തന്റെ അധികാരത്തിനും ആജ്ഞാപനത്തിനും കീഴിലാക്കുന്ന ലോകം കണ്ട ഏറ്റവും വലിയ ഏകാധിപതിയായിരിക്കും ‘മസീഹു ദ്ദജ്ജാൽ’.

ليس من بلد إلا سيطؤه الدجال إلا مكة والمدينة ليس له من نقابها نقب إلا عليه الملائكة صافين يحرسونها

ദജ്ജാൽ കാലു കുത്താത്ത ഒരു നാടും അവശേഷിക്കില്ല; മക്കയും മദീനയും ഒഴികെ. ആ രണ്ട് നാടുകളെയും സംരക്ഷിച്ചു കൊണ്ട് മലക്കുകൾ വലയം ചെയ്യുന്നുണ്ടാകും. (സ്വഹീഹുൽ ബുഖാരി: 1881, സ്വഹീഹു മുസ്‌ലിം: 2943)

നാടുകളിൽ കാലുകുത്തുക എന്നതുകൊണ്ടുദ്ദേശം പടയോട്ടത്തിലൂടെ അധികാരത്തിന് കീഴിലാക്കുക എന്നാണ്. فلا يبقى له موضع إلا ويأخذه غير مكة والمدينة “മക്കയും മദീനയും തുടങ്ങിയ സ്ഥലങ്ങളല്ലാതെ ഒരു സ്ഥലവും അവൻ പിടിച്ചടക്കാത്തതായി അവശേഷിക്കില്ല” എന്ന് ചില നിവേദനങ്ങളിൽ കാണാം. (ഉംദത്തുൽ കാരി: 10:244)

“എല്ലാ ജല തടത്തിലും അവന്റെ അധികാരമെത്തും” (يبلغ سلطانه كل منهل) എന്ന് മറ്റു ചില നിവേദനങ്ങളിലും വന്നിരിക്കുന്നു. (മുസ്നദു അഹ്മദ്: 23139)

(സ്വാഭാവികമായും ദജ്ജാലിന്റെ ഈ അധിനിവേശം ഇന്ത്യയിലുമെത്തുമല്ലൊ. ആദർശ സ്വാതന്ത്ര്യവും ജനാധിപത്യ മൂല്യങ്ങളും ദജ്ജാലിനാൽ തിരോധാനം ചെയ്യപ്പെടുന്ന കാലത്ത് അവ തിരിച്ചു പിടിക്കാൻ രക്ഷകരായി കൊണ്ടാണ് ഈ പണ്ഡിതാഭിപ്രായപ്രകാരം മുസ്‌ലിംകൾ ഇന്ത്യയിലേക്ക് കടന്നുവരിക എന്നർത്ഥം)

ദജ്ജാലിന്റെ സാമ്രാജ്യത്വ അധിനിവേശത്തിനു കീഴിൽ മനുഷ്യ ജീവിതങ്ങൾ ഞെരിഞ്ഞമരും. അങ്ങനെ ലോകം മുഴുവൻ അക്രമവും അനീതിയും അടിച്ചമർത്തലുകളും നിറയുന്ന ഘട്ടത്തിൽ മഹ്ദി എന്ന സ്ഥാനപേരിലുള്ള ഒരു ഭരണാധികാരിയുടേയും പ്രവാചകനായ ഈസാ(അ)യുടേയും നേതൃത്വത്തിൽ അന്ത്യദിനത്തോടടുത്ത് നടക്കുന്ന സ്വാതന്ത്ര്യ സമരങ്ങൾ ഒരുപാട് ഹദീസുകളുടെ ഇതിവൃത്തമായിട്ടുണ്ട്. (ഇക്കൂട്ടത്തിൽ തന്നെയാണ് ‘ഗസ്‌വത്തുൽ ഹിന്ദും’ എന്നാണ് ഈ വിഭാഗം പണിതന്മാരുടെ അഭിപ്രായം)

أبشركم بالمهدي يبعث على اختلاف من الناس وزلازل فيملأ الأرض قسطاً وعدلاً كما ملئت جوراً وظلماً “ലോകം മുഴുവൻ അക്രമങ്ങളും സ്വേച്ഛാധിപത്യവും കൊണ്ട് നിറഞ്ഞ സന്ദർഭത്തിൽ ലോകത്തെ നീതിയും ന്യായവും കൊണ്ട് മഹ്ദി നിറക്കുമെന്ന്” ഹദീസിൽ പ്രസ്ഥാവിക്കുന്നുണ്ട്. (മുസ്നദു അഹ്മദ്: 11344, അൽ അഹ്കാമുശറഇയ്യ അൽ കുബ്റാ: അബ്ദുൽ ഹക്ക് അൽ ഇശ്ബീലി: 4/532, മജ്മഉസ്സവാഇദ്: 7/316)

അത്ഭുത സിദ്ധികൾ പലതും പ്രദർശിപ്പിച്ച് ജനങ്ങളെ വശീകരിക്കുന്നതിന് പുറമെ, ദജ്ജാലെന്ന ഏകശസനാധികാരി ആളുകളെ ചതിയിലൂടെ അവരെ അഗ്നിയിലേക്ക് നയിക്കുമെന്ന സൂചനകൾ ഹദീസുകളിൽ കാണാം.

‘അയാളുടെ കൂടെ രണ്ട് ജല തടങ്ങളുണ്ടാകും. ഒന്ന് പ്രത്യക്ഷ ദൃഷ്ട്യാ വെള്ളമായിരിക്കും. മറ്റൊന്ന് ദൃഷ്ട്യാൽ അഗ്നിയായിരിക്കും. എന്നാൽ വെള്ളമെന്ന് തോന്നിപ്പിക്കപ്പെടുന്ന തടങ്ങൾ യഥാർത്ഥത്തിൽ അഗ്നിയായിരിക്കും എന്നും’ ഹദീസുകൾ സൂചിപ്പിക്കുന്നുണ്ട്. (സ്വഹീഹു മുസ്‌ലിം: 5223)

അത്ഭുത പ്രവർത്തനങ്ങൾ പ്രദർശിപ്പിച്ച് ദിവ്യത്വം വാദിക്കുക കൂടി ചെയ്യും. അധികാരത്തിലൂടെ തന്റെ ദിവ്യത്വം ജനങ്ങളിൽ അടിച്ചേൽപ്പിക്കുക എന്നത് ദജ്ജാലിന്റെ പദ്ധതികളിൽ ഒന്നായിരിക്കും.

“ദജ്ജാൽ ജനങ്ങളോട്‌ പറയും: ഈ വ്യക്തിയെ (ഒരു മുസ്‌ലിം) ഞാൻ കൊല്ലുകയും പിന്നീട് ജീവിപ്പിക്കുകയും ചെയ്താൽ (ഞാൻ നിങ്ങളുടെ ദൈവമാണെന്നതിൽ) നിങ്ങൾ സംശയിക്കുമോ? ജനങ്ങൾ പറയും: ഇല്ല. അപ്പോൾ ദജ്ജാൽ അയാളെ കൊല്ലുകയും ജീവിപ്പിക്കുകയും ചെയ്യും. അപ്പോൾ ജീവൻ തിരിച്ചു കിട്ടിയ വ്യക്തി പറയും: “അല്ലാഹുവാണേ, മുമ്പൊരിക്കലും നിന്നെക്കുറിച്ച്‌ എനിക്കില്ലാതിരുന്ന വ്യക്തമായ തിരിച്ചറിവ് ഇന്നെനിക്ക് ലഭിച്ചു.” (അഥവാ നബി പറഞ്ഞ ദജ്ജാൽ നീ തന്നെയാണു എന്ന് എനിക്കിപ്പോൾ ഉറപ്പായി എന്നയാൾ ആണയിടും. കാരണം ഇപ്രകാരം ഒരു സംഭവമുണ്ടാകുമെന്ന പ്രവാചകന്റെ ഹദീസിന് ജീവിക്കുന്ന തെളിവായി അയാൾ മാറുമല്ലൊ) അപ്പോൾ ദജ്ജാൽ അയാളെ വീണ്ടും കൊല്ലാൻ ശ്രമിക്കും. പക്ഷെ അതിന് ദജ്ജാലിനെ അല്ലാഹു അനുവദിക്കില്ല.” (സ്വഹീഹുൽ ബുഖാരി: 6599)

തന്റെ ദിവ്യത്വത്തിൽ ജനങ്ങളെ നിർബന്ധിച്ച് വിശ്വസിപ്പിക്കാനായി അധികാരവും മായാജാലവുമൊക്കെ ദജ്ജാൽ ഉപയോഗിക്കും. എന്നിട്ടും വിശ്വസിക്കാത്തവർക്ക് വധശിക്ഷയാണ് ദജ്ജാലിന്റെ ഭരണകൂടം വിധിക്കുക. വിശ്വാസ സ്വാതന്ത്ര്യം നിശ്ശേഷം നിഷേധിക്കപ്പെടുന്ന ഈ സാമ്രാജ്യത്വ സ്വേച്ഛാധിപത്യത്തോടാണ് മുസ്‌ലിംകൾ പടക്കിറങ്ങുന്നത് എന്നർത്ഥം.

ഈ സ്വേച്ഛാധിപതിയുടെ രാജാധികാരത്തോടും അയാളുടെ കിങ്കരന്മാരോടുമാണ് അന്ത്യ നാളിനോടടുത്ത് സ്വാതന്ത്ര്യ സമരങ്ങൾ നടക്കുക. അല്ലാതെ സമാധാനത്തോടെ ജീവിക്കുന്ന അമുസ്‌ലീംകളോടോ അവരുടെ നാടുകളോടോ അല്ല. ചുരുക്കത്തിൽ, ഗസ്വത്തുൽ ഹിന്ദ് പരാമർശിക്കുന്ന ഹദീഥുകളെ തെറ്റിദ്ധരിപ്പിച്ചു കൊണ്ട് ഇന്ത്യൻ മുസ്‌ലിംകൾ ഇന്ത്യയോട് യുദ്ധം ചെയ്യാൻ തക്കം പാർത്തിരിക്കുന്നവരാണെന്ന പ്രചാരണം അബദ്ധജടിലവും അടിസ്ഥാനരഹിതവുമാണ്. അകാരണമായും അന്യായമായും ഒരു നാടിനോടും യുദ്ധം ചെയ്യാനും അക്രമിക്കുവാനും ഈ ഹദീസിൽ എന്നല്ല ഇസ്‌ലാമിക പ്രമാണങ്ങളിൽ എവിടെയും പഠിപ്പിക്കുന്നില്ല.

ല്ല. മുഹമ്മദ് നബിക്കു (സ) ശേഷം രചിക്കപ്പെട്ടതാണെന്ന് വ്യക്തമാക്കുന്ന പരാമർശങ്ങളും ശാസ്ത്രീയമായ അബദ്ധങ്ങളും ഹദീഥുകളിലുണ്ടെന്ന് സ്ഥാപിക്കുവാൻ വേണ്ടിയുള്ള ശ്രമങ്ങൾ നടന്നിട്ടുണ്ട് എന്നത് ശരിയാണ്. അങ്ങനെ ഉണ്ടെന്നത് അടിസ്ഥാനരഹിതമായ ഒരു ആരോപണമാണ്.

ചരിത്രവിമര്‍ശന രീതിയില്‍ മത്‌നിനെ അപഗ്രഥിച്ച് അത് നബി(സ) പറഞ്ഞതല്ലെന്ന് സ്ഥാപിക്കണമെങ്കില്‍ അതില്‍ കാലാനുക്രമ പ്രമാദ (anarchonism) ങ്ങളെന്തെങ്കിലുമുണ്ടാവണം. നബി(സ)ക്കുശേഷം നിലവില്‍ വന്ന എന്തെങ്കി ലും കാര്യങ്ങളെക്കുറിച്ച പരാമര്‍ശങ്ങളുണ്ടാവുകയും പ്രസ്തുത പരാമര്‍ശങ്ങള്‍ നബി(സ)ക്കു ശേഷം രചിക്കപ്പെട്ടവയാണ് അതെന്ന് വ്യക്തമാക്കുകയും ചെയ്യുമ്പോഴാണ് അവയെ കാലാനുക്രമ പ്രമാദങ്ങളായി പരിഗണിക്കുന്നത്. അങ്ങനെയുള്ള എന്തെങ്കിലും സ്വഹീഹായ ഹദീഥുകളിലുള്ളതായി തെളിയിക്കുവാന്‍ വിമര്‍ശകര്‍ക്കൊന്നും കഴിഞ്ഞിട്ടില്ല. അത്തരം വല്ല പരാമര്‍ശങ്ങളും മത്‌ന് ഉള്‍ക്കൊള്ളുന്നു ണ്ടോയെന്ന പരിശോധന കൂടി കഴിഞ്ഞ ശേഷമാണ് ഒരു ഹദീഥ് സ്വഹീഹാണെന്ന് തീരുമാനിക്കുന്നത് എന്നതിനാലാണിത്. കാലാനുക്രമ പ്രമാദങ്ങളെ പരതിപ്പരിശോധിച്ച് ബൈബിളിനെപ്പോലെയുള്ള രചനകളുടെ ചരിത്രപരതയെ ചോദ്യം ചെയ്യുന്നതു പോലെ ഹദീഥുകളുടെ ചരിത്രപരതയെ നിഷേധിക്കുവാന്‍ ചരിത്ര വിമര്‍ശകര്‍ക്ക് കഴിയില്ല. ചരിത്ര വിമര്‍ശന രീതിയുടെ വക്താക്കള്‍ കാലാനുക്രമപ്രമാദങ്ങളെ പഠനവിധേയമാക്കി ഏതെങ്കിലുമൊരു പുരാതന സ്രോതസ്സിന്റെ ചരിത്രപരത നിര്‍ണയിക്കുവാനുള്ള സങ്കേതങ്ങള്‍ കണ്ടെത്തുന്നതിന് നൂറ്റാണ്ടുകള്‍ക്കു മുമ്പുതന്നെ പ്രസ്തുത സങ്കേതങ്ങള്‍ക്ക് പിടികൊടുക്കാത്തവണ്ണം ഹദീഥുകളെ അന്യൂനമാക്കുവാന്‍ ഹദീഥ് നിദാന ശാസ്ത്ര ജ്ഞര്‍ക്ക് കഴിഞ്ഞിരുന്നുവെന്നര്‍ഥം.

നബി(സ)യില്‍ നിന്നുള്ളതാണെന്ന രീതിയില്‍ നിവേദനം ചെയ്യപ്പെട്ട ഹദീഥുകളിലൊന്നും കാലാനുക്രമ പ്രമാദങ്ങളുണ്ടായിരുന്നില്ല എന്ന് ഇതിനര്‍ഥമില്ല. അന്തിമ പ്രവാചകന്റെ പേരില്‍ ആരോപിക്കപ്പെട്ടിരുന്ന വ്യാജ ഹദീഥുകളുടെ മത്‌നുകളില്‍ അത്തരം പ്രമാദങ്ങളുണ്ടാ യിരുന്നു. നബി(സ)ക്ക് പതിറ്റാണ്ടുകള്‍ക്കു ശേഷം രചിക്കപ്പെടുകയും നബി(സ)യുടെ പേരില്‍ ആരോപിക്കപ്പെടുകയും ചെയ്യുന്ന വൃത്താന്തങ്ങളില്‍ അത് സ്വാഭാവികമാണ്. വ്യാജഹദീഥുകള്‍ നിര്‍മിച്ചയാളുടെ ചരിത്രപരമായ അജ്ഞതയുടെ തോതനുസരിച്ച് അത്തരം പ്രമാദ ങ്ങളുടെ എണ്ണത്തില്‍ ഏറ്റക്കുറച്ചിലുകളുണ്ടാകുമെന്നേയുള്ളൂ. 'ഇരുനൂറു വര്‍ഷങ്ങള്‍ക്കു ശേഷമാണ് ലോകാവസാനത്തിന്റെ അടയാള ങ്ങള്‍ പ്രത്യക്ഷപ്പെടുക' എന്ന ഹദീഥ് തള്ളുന്നതിന് ഇരുനൂറു വര്‍ഷം കഴിഞ്ഞിട്ടും അത്തരം ലക്ഷണങ്ങളൊന്നും കണ്ടിട്ടില്ല എന്ന കാരണം കൂടി ഇമാം ബുഖാരി പറഞ്ഞതായി ഇമാം ദഹബി വ്യക്തമാക്കുന്നുണ്ട്.(അദ്ദഹബി: മീസാനുല്‍ ഇഅ്തിദാല്‍, വാല്യം 3, പുറം 306) 'പ്രചാരത്തിലിരിക്കുന്ന മുസ്‌ലിം നാണയങ്ങളെ പൊട്ടിക്കുന്നത് പ്രവാചകന്‍(സ) വിലക്കിയിരിക്കുന്നു' വെന്ന ഹദീഥ് നിവേദനം ചെയ്ത മുഹമ്മദ് ബിന്‍ ഫദാ സ്വീകരിക്കുവാന്‍ കൊള്ളാത്തവനാണെന്ന് വിധിക്കുവാനുള്ള കാരണങ്ങളിലൊന്നായി അമവിയ്യാക്കളുടെ കാലത്ത് മാത്രമാണ് മുസ്‌ലിംകള്‍ നാണയങ്ങള്‍ അടിച്ചിറക്കാനാരംഭിച്ചത് എന്ന വസ്തുത എടുത്തുപറയുകയും നബി(സ)യുടെ കാലത്ത് നാണയം നിലനിന്നിരുന്നില്ലെന്നതിനാല്‍ അദ്ദേഹം അസ്വീകാര്യനാണെന്ന് ഇമാം ബുഖാരി വ്യക്തമാക്കുകയും ചെയ്തിട്ടുണ്ട്.(ഇമാം ബുഖാരിയുടെ അത്താരീഖുല്‍ ഔസത്തില്‍ നിന്ന് (വാല്യം 2, പുറം 110 -119) ഖൈബറിലെ യഹൂദന്‍മാരോട് നികുതിവാങ്ങിയതിന് സാക്ഷി സഅദ് ബ്‌നു മുആദും (റ)കരാര്‍ എഴുതിയത് മുആവിയയേുമായിരുന്നു' എന്ന ഹദീഥ് തള്ളിക്കളയുന്നതിനു ള്ള കാരണമായി ഖൈബര്‍ യുദ്ധകാലത്ത് കപ്പം വാങ്ങുന്ന സമ്പ്രദായമുണ്ടായിരുന്നില്ലെന്നും സഅദ്ബ്‌നു മുആദ്‌ (റ)പ്രസ്തുത യുദ്ധത്തിന് മുമ്പു തന്നെ മരണപ്പെട്ടിട്ടുണ്ടെന്നും മുആവിയ (റ)മുസ്‌ലിമായത് യുദ്ധത്തിന് ശേഷവുമാണെന്ന വസ്തുതകള്‍ കൂടി പണ്ഡിതന്മാര്‍ എടുത്തു പറഞ്ഞതായി നടേ വ്യക്തമാക്കിയിട്ടുണ്ട്. ഇത്തരം പ്രമാദങ്ങള്‍ കണ്ടെത്തുകയും അവയെ അപഗ്രഥിച്ച് മാറ്റിവെക്കുകയും ചെയ്ത ശേഷ മാണ് ഹദീഥ് നിദാന ശാസ്ത്രജ്ഞന്‍മാര്‍ സ്വഹീഹായ ഹദീഥുകളെ വേര്‍തിരിച്ച് രേഖപ്പെടുത്തിയത്. സ്വഹീഹായ ഹദീഥുകളുടെ മത്‌നില്‍ ഇനിയുമൊരു ചരിത്ര വിമര്‍ശനാപഗ്രഥനം ആവശ്യമില്ലെന്നര്‍ഥം.

സ്വഹീഹായ ഹദീഥുകളില്‍ ആരോപിക്കപ്പെടുന്ന ശാസ്ത്രാബദ്ധങ്ങളുടെ സ്ഥിതിയും ഇതുതന്നെയാണ്. ഖണ്ഡിതമായി തെളിയിക്കപ്പെട്ട ശാസ്ത്രവസ്തുതകളൊന്നും സ്വഹീഹായ ഹദീഥുകള്‍ നിഷേധിക്കുന്നില്ല. എന്നാല്‍ ശാസ്ത്രീയമായി തെളിയിക്കപ്പെടാത്ത കാര്യങ്ങളെക്കു റിച്ച പരാമര്‍ശങ്ങള്‍ അവയിലുണ്ടാവാം. നിലനില്‍ക്കുന്ന അറിവിന്റെ മാത്രം അടിസ്ഥാനത്തില്‍ അത്തരം പരാമര്‍ശങ്ങളെ അബദ്ധമായി ഗണിച്ച് ഹദീഥുകള്‍ അസ്വീകാര്യമാണെന്ന് വിധിക്കുന്നത് അപകടകരമാണ്. മനുഷ്യന്റെ വൈജ്ഞാനിക മണ്ഡലം ഇനിയെത്ര വളരാനിരി ക്കുന്നു?! പ്രാപഞ്ചിക പ്രതിഭാസങ്ങളെക്കുറിച്ച അറിവിന്റെ ചുരുളുകള്‍ ഇനിയുമെത്ര നിവരാനിരിക്കുന്നു?! വിജ്ഞാനമഹാസാഗര ത്തിലെ തുള്ളികള്‍ മാത്രം ആസ്വദിക്കാന്‍ അവസരം ലഭിച്ച മനുഷ്യര്‍ക്കെങ്ങനെയാണ് പ്രസ്തുത സാഗരത്തിന്റെ ഉടമസ്ഥനായ അല്ലാഹു വിന്റെ ബോധനത്തെ തനിക്കു ലഭിച്ച ചെറിയ അറിവിന്റെ വെളിച്ചത്തില്‍ തള്ളിക്കളയാനാവുക? നമുക്ക് മനസ്സിലായിക്കഴിഞ്ഞിട്ടില്ലാത്ത കാര്യങ്ങളെക്കുറിച്ച് സ്വഹീഹായ ഹദീഥുകള്‍ എന്തെങ്കിലും പറയുന്നുവെങ്കില്‍, നമ്മുടെ വൈജ്ഞാനിക മണ്ഡലം അവിടെയെത്തിക്കഴി ഞ്ഞിട്ടില്ലെന്ന് കരുതി കാത്തിരിക്കുന്നതല്ലേ ശരി?! സ്വഹീഹായ ഹദീഥുകളില്‍ അശാസ്ത്രീയത പരതുന്നവര്‍ക്ക്, അറിയാനുള്ളതെല്ലാം അറിഞ്ഞുകഴിഞ്ഞവരാണ് തങ്ങളെന്ന് അഭിപ്രായമുണ്ടോ? ഇല്ലെങ്കില്‍, എങ്ങനെയാണ് ഹദീഥുകളെ തള്ളിക്കളയാന്‍ ശാസ്ത്രത്തിന്റെ ഉപകരണങ്ങള്‍ ഉപയുക്തമാവുക?

അബൂഹുറൈറ(റ) യുടെ യഥാർത്ഥ പേരെന്താണെന്നറിയില്ല; ഇസ്‌ലാം സ്വീകരണത്തിന് മുമ്പുള്ള അദ്ദേഹത്തിന്റെ ചരിത്രവും അറിയില്ല. അങ്ങനെയുള്ള ഒരാളുടെ ഇസ്‌ലാം ആശ്ലേഷം സത്യസന്ധമായിരുന്നുവെന്ന് എങ്ങനെ കരുത്താനാവും? സ്വഹാബിമാരിൽ പലരും അദ്ദേഹത്തെ വിമർശച്ചതായി കാണാനും കഴിയും. അക്ഷരജ്ഞാനം പോലുമില്ലാത്ത ഒരാളെ ഒരാളെ വിശ്വസിച്ചുകൊണ്ട് അയാൾ പറഞ്ഞതെല്ലാം നബി(സ)യിൽ നിന്നുള്ളവയാണെന്ന് എങ്ങനെ കരുതാനാവും ?

ബൂഹുറയ്‌റയുടെ(റ) യഥാര്‍ഥ നാമത്തെക്കുറിച്ചുള്ള അഭിപ്രായവ്യത്യാസങ്ങളും ഇസ്‌ലാം സ്വീകരണത്തിനുമുമ്പുള്ള അദ്ദേഹത്തിന്റെ ചരിത്രത്തെക്കുറിച്ചുള്ള അജ്ഞതയും അദ്ദേഹത്തിന്റെ സ്വീകാര്യതയെക്കുറിച്ച് സംശയങ്ങളുണ്ടാക്കുന്ന കാര്യങ്ങളായി എടുത്തുപറയുന്ന വിമർശിക്കുന്നവർക്ക് മറ്റു സ്വഹാബിമാരുടെ യഥാര്‍ഥനാമവും ചരിത്രവും എത്രത്തോളം അറിയാമെന്നാണ് തിരിച്ചു ചോദിക്കാനുള്ളത്. നബി(സ)യുടെ സന്തതസഹചാരിയും ആദ്യ ഖലീഫയുമായിരുന്ന അബൂബക്കര്‍െ(റ)ന്റ യഥാര്‍ഥ പേരിനെക്കുറിച്ച് ചരിത്രകാരന്‍മാ ര്‍ക്കിടയില്‍ അഭിപ്രായാന്തരങ്ങളുണ്ടെന്നത് അദ്ദേഹത്തിന്റെ വിശ്വാസ്യതയെ ബാധിക്കുമെന്ന് ഏതെങ്കിലും അബൂഹൂറയ്‌റാ വിമര്‍ശകന്‍ മാര്‍ വാദിച്ചുകണ്ടിട്ടില്ല. അബൂഉബൈദ(റ), അബൂദുജാന(റ), അബൂദര്‍റുല്‍ ഗിഫ്ഫാരി(റ), അബൂദര്‍ദാഅ്‌ (റ)തുടങ്ങിയ നാമങ്ങളില്‍ അറി യപ്പെടുന്ന പ്രമുഖരായ സ്വഹാബികളുടെ യഥാര്‍ഥ പേരെന്തായിരുന്നുവെന്ന് ചരിത്രകാരന്‍മാരില്‍ പലര്‍ക്കും അജ്ഞാതമാണ്.സ്വഹാബി മാരില്‍ പലരുടെയും ഇസ്‌ലാം സ്വീകരണത്തിന് മുമ്പുള്ള ചരിത്രത്തെക്കുറിച്ചും കൂടുതല്‍ അറിവൊന്നുമില്ല. അറബികള്‍ക്കിടയില്‍ പ്രശ സ്തമായിരുന്ന ബനൂദൗസ് ഗോത്രക്കാരനായ അബൂഹുറയ്‌റയേുടെ ചരിത്രവും കുടുംബ വേരുകളും മറ്റു പല സ്വഹാബികളുടേതിലുമ പേക്ഷിച്ച് അറിയപ്പെടുന്നവയാണ്.

അദ്ദേഹത്തിന്റെ ചരിത്രം തീരെ അജ്ഞാതമാണെന്ന് വന്നാല്‍പോലും അതിന്റെ അടിസ്ഥാനത്തില്‍ അബൂഹുറയ്‌റ(റ)യുടെ സത്യസന്ധതയെ ചോദ്യം ചെയ്യാന്‍ എങ്ങനെ സാധിക്കും? ജാഹിലിയ്യാ ചരിത്രം അജ്ഞാതമാണെന്ന കാരണത്താല്‍ അസ്വീകാര്യമായി വിലയിരുത്തുകയാണെങ്കില്‍ സ്വഹാബിമാരില്‍ മിക്കവരും അസ്വീകാര്യരായിത്തീരുമെന്നതാണ് വസ്തുത. അബൂഹുറ യ്‌റെ(റ)യെ തകര്‍ത്ത് അതിലൂടെ സ്വഹാബിമാരെയും ഹദീഥുകളെയുമെല്ലാം അസ്വീകാര്യമായി മുദ്രകുത്തി നബി(സ)യുടെ ചരിത്രപരത യെത്തന്നെ നിഷേധിക്കുന്നതിനുള്ള പാതയൊരുക്കിയവര്‍ നല്‍കിയ 'തെളിവുകളു' പയോഗിക്കുന്നവര്‍ യഥാര്‍ഥത്തില്‍ തകര്‍ക്കുവാന്‍ ശ്രമി ക്കുന്നത് ഇസ്‌ലാമിനെത്തന്നെയാണെന്ന വസ്തുതയാണിവിടെ അനാവൃതമാവുന്നത്.

ഓറിയന്റലിസ്റ്റുകള്‍ നല്‍കിയ 'തെളിവുകളു'പയോഗിച്ച് അബൂഹുറയ്‌റ(റ)യുടെ ഇസ്‌ലാം ആശ്ലേഷണം ആത്മാര്‍ഥമായിരുന്നില്ലെന്ന് വാദിക്കുന്നവരുടെ പക്കല്‍ ആത്മാര്‍ഥതയെ അളക്കുന്നതിനുള്ള മാനദണ്ഡമെന്താണ്? നബി(സ)യുടെ കൂടെ അദ്ദേഹത്തോടൊപ്പം നാലു വര്‍ ഷത്തിലധികം ജീവിച്ചിട്ട് അബൂഹുറയ്റയുടെ (റ)ആത്മാര്‍ഥതയില്‍ എന്തെങ്കിലുമൊരു സംശയം നബി(സ) പ്രകടിപ്പിച്ചതായി തെളിയി ക്കുന്ന ഒരു സംഭവം പോലും ഉദ്ധരിക്കുവാന്‍ അബൂഹുറയ്‌റാവിമര്‍ശകര്‍ക്ക് കഴിഞ്ഞിട്ടില്ല. വിദൂര ദേശത്തുനിന്ന് നബി(സ) ജീവിതത്തെ ക്കുറിച്ച് പഠിക്കുവാനായി മദീനയിലേക്ക് പലായനം ചെയ്‌തെത്തിയ അദ്ദേഹത്തിന്റെ ആത്മാര്‍ഥതയില്‍ മുഹാജിറുകളിലോ അന്‍സ്വാരി കളിലോ പെട്ട ആരെങ്കിലും സംശയം പ്രകടിപ്പിച്ചതായി യാതൊരു രേഖയുമില്ല. സകാത്തിന്റെ സ്വത്തു സൂക്ഷിക്കാന്‍ നബി(സ)ഏല്‍പ്പിച്ചി രുന്നത് അബൂഹുറയ്‌റെ(റ)യെ ആയിരുന്നു.(സ്വഹീഹുല്‍ ബുഖാരി, കിതാബു സ്‌സകാത്ത്) നബിജീവിതത്തെ നിരീക്ഷിക്കുന്നതിനായി പള്ളിവരാന്തയില്‍ കഴിഞ്ഞിരുന്നവരെ ക്വുര്‍ ആന്‍ പ്രശംസിക്കുകയാണ് ചെയ്തിട്ടുള്ളത് (2:273). അവരില്‍ കപട വിശ്വാസികളുള്ളതായി ക്വുര്‍ആന്‍ യാതൊരു സൂചനയും നല്‍കുന്നില്ല. അല്ലാഹുവിനോ റസൂലിനോല സ്വഹാബികള്‍ക്കോ മനസ്സിലാകാതിരുന്ന അബൂഹുറയ്‌റ(റ)യുടെ 'കാപട്യം' തങ്ങള്‍ക്കാണ് തിരിച്ചറിയാന്‍ കഴിഞ്ഞതെന്ന് ഗര്‍വ് നടിക്കുന്നവര്‍, ഓറിയന്റലിസ്റ്റുകള്‍ വെട്ടിവെടിപ്പാക്കിയ വഴിയിലൂടെ പോയി ക്വുര്‍ആനിനെയും നബി(സ)യെയു മാണ് യഥാര്‍ഥത്തില്‍ തള്ളിപ്പറയുന്നത്.

അക്ഷരജ്ഞാനമില്ലാതിരുന്ന അബൂഹുറയ്‌റെ(റ)ക്കെങ്ങനെയാണ് ഇത്രയധികം നബിചര്യകള്‍ ഓര്‍മിച്ചുവെക്കാനായതെന്ന് ആശ്ചര്യപ്പെട്ട് അദ്ദേഹത്തെ കളിയാക്കാനൊരുമ്പെടുന്നവര്‍ യഥാര്‍ഥത്തില്‍ പരിഹസിക്കുന്നത് നബി(സ)യെയും ക്വുര്‍ആനിനെയുമാണ്. വിശുദ്ധ ക്വുര്‍ ആന്‍ അവതരിപ്പിക്കപ്പെട്ടത് അക്ഷരാഭ്യാസമില്ലാത്ത മുഹമ്മദ് നബി(സ)ക്കായിരുന്നു. അദ്ദേഹവും അനുചരന്‍മാരില്‍ ചിലരും ക്വുര്‍ആന്‍ പൂര്‍ണമായും ഹൃദിസ്ഥമാക്കിയിരുെന്നന്ന വസ്തുത വിമര്‍ശകര്‍പോലും അംഗീകരിക്കുന്നതാണ്. മനഃപാഠമാക്കുവാനുള്ള, അറബിക ളുടെ ശേഷിയെക്കുറിച്ച് ഓറിയന്റലിസ്റ്റുകളില്‍ പലരും വാചാലരായിട്ടുണ്ട്. നബി(സ)യില്‍ നിന്ന് പഠിച്ച കാര്യങ്ങള്‍ പരസ്പരം പറ ഞ്ഞ് പരിശോധിച്ച് ഹൃദിസ്ഥമാക്കിയിട്ടുണ്ടോയെന്ന് ഉറപ്പുവരുത്തുന്ന പതിവ് സ്വഹാബിമാര്‍ക്കുണ്ടായിരുന്നുവെന്ന് ഡോ: മുഹമ്മദ് മുസ്തഫ അല്‍ അഅ്ദമി തെളിവുകളുദ്ധരിച്ചുകൊണ്ട് സമര്‍ഥിക്കുന്നുണ്ട്. നിവേദക പരമ്പരകളോടെ ഇമാം ബുഖാരി മൂന്ന് ലക്ഷവും ഇമാം അഹ്മദ് ആറ് ലക്ഷവും ഹദീഥുകള്‍ മനഃപാഠമാക്കിയിരുന്നുവെന്ന വസ്തുത ഹൃദിസ്ഥമാക്കുവാനുള്ള അറബികളുടെ കഴിവ് വ്യക്ത മാക്കുന്ന തെളിവുകളാണ്. പ്രവാചകനില്‍നിന്ന് താന്‍ പഠിച്ച മൂവായിരത്തോളം വസ്തുതകള്‍ മനസ്സില്‍ സൂക്ഷിക്കുകയും പില്‍കാലക്കാ ര്‍ക്ക് പകര്‍ന്നു നല്‍കുകയും ചെയ്ത അബൂഹുറയ്‌റ(റ)യുടെയും മനഃപാഠമാക്കുവാനുള്ള കഴിവ് മികച്ചതായിരുന്നു; അറബികള്‍ക്ക് അതൊരിക്കലും അസാധ്യമായിരുന്നില്ല.

മറ്റു സ്വഹാബിമാരെക്കാള്‍ നബി(സ)യുടെ ഹദീഥുകള്‍ മനഃപാഠമാക്കിയിരുന്നത് അബൂഹുറയ്‌റ യോണെന്ന് അബൂദുല്ലാഹിബ്‌നു അംറ്‌ (റ)പറഞ്ഞതായി ഇമാം ഇബ്‌നുകഥീര്‍ രേഖപ്പെടുത്തുന്നുണ്ട്. അമവീ ഖലീഫയായിരുന്ന മര്‍വാനു ബ്‌നുല്‍ ഹകം അബൂഹുറയ്‌റയേുടെ ഓര്‍മശക്തി പരിശോധിച്ച് ഉറപ്പുവരുത്തിയതായും അദ്ദേഹം ഉദ്ധരിക്കുന്നുണ്ട്. അക്ഷരാഭ്യാസമില്ലെങ്കിലും ഹൃദിസ്ഥമാക്കുന്നതില്‍ സമര്‍ഥനായിരുന്ന അബൂഹുറയ്‌റ (റ)നബിജീവിതത്തെപ്പറ്റി താന്‍ അറിഞ്ഞ കാര്യങ്ങള്‍ മനസ്സില്‍ സൂക്ഷിക്കുകയും അടുത്ത തലമുറക്ക് പകര്‍ന്നു നല്‍കുകയുമാണ് ചെയ്തതെന്ന് സാരം.

സ്വഹാബീപ്രമുഖരായ ഉമര്‍(റ), ഉഥ്മാന്‍(റ), അലി(റ), ആയിശ(റ) തുടങ്ങിയവര്‍ അബൂഹുറയ്‌റെ(റ)യെ വിമര്‍ശിക്കുകയും നിഷേധിക്കു കയും അദ്ദേഹത്തിന്റെ സത്യസന്ധതയില്‍ സംശയിക്കുകയും ചെയ്തിരുന്നുവെന്ന് സ്ഥാപിക്കാനായി വിമര്‍ശകര്‍ ഉദ്ധരിക്കുന്ന സംഭവ ങ്ങള്‍ വ്യാജമായി നിര്‍മിക്കപ്പെട്ടവയോ സന്ദര്‍ഭത്തില്‍ നിന്നടര്‍ത്തി മാറ്റി തെറ്റായി വ്യാഖ്യാനിക്കപ്പെട്ടവയോ ആണെന്ന് തെളിവുകളുദ്ധ രിച്ചുകൊണ്ട് പണ്ഡിതന്‍മാര്‍ വ്യക്തമാക്കിയിട്ടുണ്ട്. പ്രവാചകന്റെല നിര്‍ദേശാനുസരണം ഇസ്‌ലാമിലെ ആദ്യത്തെ ഹജ്ജിന് നേതൃത്വം നല്‍കിയ അബൂബക്കര്‍ (റ)തന്നോടൊപ്പമുണ്ടായിരുന്ന അബൂഹുറയ്‌റ(റ)യെയാണ് അടുത്ത വര്‍ഷം മുതല്‍ ബഹുദൈവാരാധകര്‍ക്ക് ഹജ്ജിന് വരാന്‍ അനുവാദമുണ്ടാവുകയില്ലെന്ന് പ്രഖ്യാപിക്കുവാന്‍ ചുമതലപ്പെടുത്തിയതെന്ന വസ്തുത അബൂബക്കര്‍ (റ)അബൂഹുറ യ്‌റെ(റ)യെ അംഗീകരിച്ചിരുന്നുവെന്ന് വ്യക്തമാക്കുന്നു.പച്ചകുത്തലുമായി ബന്ധപ്പെട്ട് നബി(സ) എന്തെങ്കിലും പറഞ്ഞിട്ടുണ്ടോയെന്ന ഉമറിന്റെ (റ)ചോദ്യത്തിന് അബൂഹുറയ്‌റ (റ)നല്‍കിയ മറുപടി ചോദ്യം ചെയ്യാതെ അദ്ദേഹം സ്വീകരിച്ചതുംഹസ്സാനുബ്‌നു സാബിത്തി ന്റെ കവിതകളെ നബി(സ) പുകഴ്ത്തിയതായുള്ള അബൂഹുറയ്‌റയുടെ സാക്ഷ്യം ഉമര്‍ (റ)അപ്പടി സ്വീകരിച്ചതുമായ സംഭവങ്ങള്‍ ഉമര്‍ (റ)അദ്ദേഹത്തെ അംഗീകരിച്ചിരുന്നുവെന്നതിനുള്ള വ്യക്തമായ തെളിവുകളാണ്. ഉഥ്മാനോ (റ)അലിയോ (റ)ആയിശയോ അബൂഹുറയ്‌ റെ(റ)യെ സംശയിക്കുകയോ വിമര്‍ശിക്കുകയോ ചെയ്തതായി വിശ്വാസയോഗ്യമായ തെളിവുകളെന്തെങ്കിലും ഉദ്ധരിക്കുവാന്‍ വിമര്‍ശക ര്‍ക്ക് കഴിഞ്ഞിട്ടില്ല. ഹദീഥുകള്‍ നിവേദനം ചെയ്യുമ്പോള്‍ അബൂഹുറയ്‌റ(റ)യോടൊപ്പമുണ്ടായിരുന്ന പ്രസിദ്ധ സ്വഹാബിയായ അബൂസ ഈദില്‍ ഖുദ്‌രി (റ)അദ്ദേഹത്തെ ശരിവെക്കുകയും സത്യപ്പെടുത്തുകയും ചെയ്തതായി ഇമാം മുസ്‌ലിം ഉദ്ധരിക്കുന്നുണ്ട്. അബൂഹു റയ്‌റ (റ)നിവേദനം ചെയ്ത ഹദീഥുകളെ നിരവധി സന്ദര്‍ഭങ്ങളില്‍ പ്രസിദ്ധ സ്വഹാബിയായ ജാബിറുബ്‌നു അബ്ദില്ല അനുകൂലി ക്കുകയും സത്യപ്പെടുത്തുകയും ചെയ്തതായി കാണാനാവും. സ്വഹാബിമാരെല്ലാം അബൂഹുറയ്‌റ(റ)യുടെ സത്യസന്ധത അംഗീകരിച്ചിരുന്നു വെന്ന് ഇവ വ്യക്തമാക്കുന്നു.

മക്കയിലും മദീനയിലുമെല്ലാംനബി(സ)യോടൊപ്പമുണ്ടായിരുന്ന അബൂബക്കറും(റ ) ഉമറും(റ) ഉഥ്മാനുമെല്ലാം (റ) ഏതാനും ഹദീഥുകൾ മാത്രം നിവേദനം ചെയ്തപ്പോൾ അബൂഹുറയ്‌റ(റ) ആയിരക്കണക്കിന് ഹദീഥുകൾ നിവേശനം ചെയ്തിട്ടുണ്ട്. ഇത് ഹദീഥുണ്ടാക്കുന്നതിൽ അബൂ ഹുറയ്റ(റ)അതിരുവിട്ട് ആവേശം കാണിച്ചിട്ടുണ്ടെന്നല്ലേ വ്യക്തമാക്കുന്നത്?

ആദ്യകാലത്തുതന്നെ ഇസ്‌ലാം സ്വീകരിക്കുകയും മക്കയിലും മദീനയിലുമെല്ലാംനബി(സ)യോടൊപ്പമുണ്ടാവുകയും ചെയ്ത അബൂബക്ക റും(റ) ഉമറും(റ)ഉഥ്മാനുമൊന്നും(റ)അബൂഹുറയ്‌റ(റ)നിവേദനം ചെയ്തതു പോലെ ധാരാളം ഹദീഥുകള്‍ നിവേദനം ചെയ്തിട്ടില്ലെന്നത് അബൂ ഹുറയ്റ(റ)ഇക്കാര്യത്തില്‍ അതിരുവിട്ട് ആവേശം കാണിച്ചിട്ടുണ്ടെന്ന് വ്യക്തമാക്കുന്നുണ്ടെന്നും അത് അദ്ദേഹത്തിന്റെ വിശ്വാസ്യ തയെ തകര്‍ക്കുന്നുണ്ടെന്നും വാദിക്കുന്നത് ഹദീഥ് ശേഖരണത്തിന്റെ ചരിത്രത്തെക്കുറിച്ച അജ്ഞതകൊണ്ടോ അജ്ഞത നടിച്ച് നിഷ്‌കളങ്കരെ തെറ്റു ധരിപ്പിക്കുവാനുള്ള വ്യഗ്രതകൊണ്ടോ ആണെന്ന് വ്യക്തമാണ്.

അബൂക്കറില്‍(റ)നിന്ന് 142ഉം ഉമറില്‍(റ)നിന്ന് 537ഉം ഉഥ്മാനിൽ(റ) നിന്ന് 146 ഉം അലിയില്‍(റ)നിന്ന് 586ഉം ഹദീഥുകള്‍ ഉദ്ധരിക്കപ്പട്ടപ്പോള്‍ അബൂഹുറയ്‌റേയില്‍നിന്ന് 5347ഉം അബ്ദുല്ലാഹിബ്‌നു ഉമറില്‍നിന്ന് 2630ഉം അനസ്ബ്‌നു മാലിക്കില്‍നിന്ന്(റ) 2300ഉം ആയിശയില്‍(റ) നിന്ന് 2200ഉം അബ്ദുല്ലാഹിബ്‌നു അബ്ബാസില്‍(റ)നിന്ന് 1665ഉം ജാബിറുബ്‌നു അബ്ദില്ല(റ) യില്‍നിന്ന് 1500ഉം ഹദീഥുകള്‍ ഉദ്ധരിക്കപ്പെടാനുള്ള കാരണം, രണ്ടാമത് പറഞ്ഞവര്‍ക്കാണ് അടുത്ത തലമുറയുമായി സമ്പര്‍ക്ക ത്തിലാവാന്‍ കൂടുതല്‍ അവസരമുണ്ടായത് എന്നതാണ്.

ഹദീഥുകളുടെ നിവേദനവും സംപ്രേക്ഷണവും നടന്നതെങ്ങനെയെന്ന് മനസ്സിലാ യാല്‍ ഇത്തരമൊ രു വിമര്‍ശനം തന്നെ അപ്രസക്തമാവുമെന്നുറപ്പാണ്. മുഹമ്മദ്നബി(സ)യില്‍നിന്ന് ഏതെങ്കിലുമൊരു കാര്യം മനസ്സിലാ ക്കിയ ഒരാള്‍ അത് മനസ്സി ലാക്കാത്ത മറ്റൊരാള്‍ക്ക് പറഞ്ഞുകൊടുക്കുമ്പോഴാണ് അത് ഹദീഥ് ആയിത്തീരുന്നത്. മനസ്സിലാകാത്തവരുടെ എണ്ണവര്‍ധനവിനനുസരിച്ച് ഈ പറഞ്ഞുകൊടുക്കല്‍ പ്രക്രിയയുടെ എണ്ണവും വര്‍ധിക്കുക സ്വാഭാവികമാണ്. പ്രവാചകവിയോഗത്തിന് 27 മാസങ്ങള്‍ കഴിഞ്ഞ്, ഹിജ്‌റ 13 ജുമാദുല്‍ ആഖിര്‍ 13 തിങ്കളാഴ്ച മരണപ്പെട്ട അബൂബക്കറിന്റെ(റ)ജീവിതകാലത്ത്നബി(സ)യില്‍ നിന്ന് കാര്യങ്ങള്‍ നേര്‍ക്കുനേരെ മനസ്സിലാ ക്കിയവരുടെ എണ്ണം അങ്ങനെ മനസ്സിലാക്കാത്തവരുടെ എണ്ണത്തേക്കാള്‍ വളരെ കൂടുതലായതിനാല്‍ അദ്ദേഹത്തിലൂടെ നിവേദനം ചെയ്യ പ്പെട്ട ഹദീഥുകളുടെ എണ്ണവും കുറവാണെന്നത് സ്വാഭാവികം മാത്രം. ഹിജ്‌റ 23 ദുല്‍ഹിജ്ജ 26ന് മരണ പ്പെട്ട ഉമറും(റ)ഹിജ്‌റ 36 മുഹര്‍റം 16ന് മരണപ്പെട്ട ഉഥ്മാനും(റ)ഹിജ്‌റ 40 ശവ്വാല്‍ 20ന് മരണപ്പെട്ട അലിയും(റ) ജീവിച്ചത് ധാരാളം സ്വഹാ ബിമാര്‍ ജീവിച്ചിരുന്ന കാലത്തായി രുന്നതിനാലും അവര്‍ ഭരണാധികാരികളും രാഷ്ട്രവ്യവഹാരത്തില്‍ ശ്രദ്ധ കേന്ദ്രീകരിക്കേണ്ടവരും ആയതിനാലുമാണ് അവരിലൂടെ നിവേദ നം ചെയ്യപ്പെട്ട ഹദീഥുകളുടെ എണ്ണം താരതമ്യേന കുറവായത്. ഇവരെല്ലാം പ്രവാചകവിയോഗം കഴിഞ്ഞ് ആദ്യത്തെ നാലു പതിറ്റാണ്ടുകള്‍ ക്കകം ജീവിച്ചവരാണ്. ധാരാളം സ്വഹാബിമാര്‍ ജീവിച്ചിരുന്ന അക്കാലത്ത്,നബി(സ)യില്‍ നിന്ന് നേര്‍ക്കുനേരെ കാര്യങ്ങള്‍ മനസ്സിലാക്കിയവ ര്‍ക്ക് ഇവര്‍ അതേ കാര്യത്തെക്കുറിച്ച് പറഞ്ഞു കൊടുക്കേണ്ടതില്ലായിരുന്നു.

എന്നാല്‍ ഹിജ്‌റ നാലാമത്തെ പതിറ്റാണ്ടു കഴിഞ്ഞപ്പോഴേക്ക് സ്ഥിതിമാറി.നബി(സ)യില്‍നിന്ന് നേര്‍ക്കുനേരെ കാര്യങ്ങള്‍ മനസ്സിലാക്കിയവരുടെ എണ്ണം കുറഞ്ഞുവന്നു. പുതിയ തലമുറക്ക് തങ്ങള്‍നബി(സ)യില്‍ നിന്ന് മനസ്സിലാക്കിയ കാര്യങ്ങള്‍ പറഞ്ഞുകൊടുക്കേണ്ടത് അന്നു ജീവിച്ചിരുന്ന സ്വഹാബിമാരുടെ ബാധ്യതയായിത്തീര്‍ന്നു. ഹിജ്‌റ 57ല്‍  ത ന്റെ 78ാമത്തെ വയസ്സില്‍ മരണപ്പെട്ട അബൂഹുറയ്‌റയും(റ)ഹിജ്‌റ 73ല്‍ തന്റെ 80ാമത്തെ വയസ്സില്‍ മരണപ്പെട്ട അബ്ദുല്ലാഹിബ്‌നു ഉമറും (റ)ഹിജ്‌റ 93ല്‍ തന്റെ 103ാമത്തെ വയസ്സില്‍ മരണപ്പെട്ട അനസ്ബ്‌നു മാലിക്കും (റ)ഹിജ്‌റ 58ല്‍ തന്റെ 65ാമത്തെ വയസ്സില്‍ മരണപ്പെട്ട പ്രവാചകപത്‌നി ആയിശയും ഹിജ്‌റ 68ല്‍ തന്റെ 71ാമത്തെ വയസ്സില്‍ മരണപ്പെട്ട അബ്ദുല്ലാഹിബ്‌നു അബ്ബാസും(റ)ഹിജ്‌റ 78ല്‍ തന്റെ 94ാമത്തെ വയസ്സില്‍ മരണപ്പെട്ട ജാബിറുബ്‌നു അബ്ദുല്ലയേും കൂടുതല്‍ ഹദീഥുകള്‍ നിവേദനം ചെയ്തത് തങ്ങളുടെ കാലത്ത്നബി(സ)യെ നേരില്‍ കാണുകയോ കേള്‍ക്കുകയോ ചെയ്ത ആളുകള്‍ താരതമ്യേന കുറവായതു കൊണ്ടായിരുന്നു.നബി(സ)യുടെ ജീവിതചര്യയെക്കുറിച്ച് പഠിപ്പി  ക്കേണ്ടതിന്റെ ആവശ്യകത അദ്ദേഹത്തെ നേരില്‍ അനുഭവിച്ചിട്ടില്ലാത്ത വരുടെ കാലത്ത് കൂടുതലായിരിക്കുമെന്ന സരളമായ വസ്തുത പോലും പരിഗണിക്കാതെയാണ് ഹദീഥ് നിവേദനത്തില്‍ അത്യാവേശം കാണിച്ച് അബദ്ധങ്ങളെഴുന്നള്ളിച്ചയാളായി അബൂഹുറയ്‌റ(റ)യെ അവതരിപ്പിക്കുവാന്‍ വിമര്‍ശകര്‍ ധാര്‍ഷ്ട്യം കാണിക്കുന്നത്.

ല്ല. അബൂഹുറൈറ (റ) ഹദീഥുകളൊന്നും കെട്ടിയുണ്ടാക്കിയിട്ടില്ല. നബിജീവിതത്തിന്റെ അവസാനത്തെ നാല് വർഷങ്ങളിൽ നബിയോടൊപ്പം അദ്ദേഹത്തിന്റെ സ്വകാര്യനിമിഷങ്ങളിലൊഴിച്ച് മുഴുസമയവും ജീവിച്ച ആ മഹാത്യാഗിയുടെ ശ്രമഫലമായാണ് ഏറ്റവുമധികം ഹദീഥുകൾ അടുത്ത തലമുറക്ക് ലഭിച്ചത് എന്നതാണ് സത്യം . ഏറ്റവുമധികം ഹദീഥുകള്‍ നിവേദനം ചെയ്ത അബൂഹുറയ്‌റ(റ) തന്നെയാണ് ഓറിയന്റലിസ്റ്റുകളുടെയും അവരുടെ ആശയങ്ങള്‍ക്ക്      മുസ്‌ലിം സമുദായത്തില്‍ വിലാസമുണ്ടാക്കുവാന്‍ ശ്രമിക്കുന്നവരുടെയും ആക്ഷേപങ്ങള്‍ക്ക് വിധേയനായ പ്രധാനപ്പെട്ട സ്വഹാബി. ഹദീഥ് അപഗ്രഥനത്തില്‍ അഗ്രഗണ്യരായി അറിയപ്പെടുന്ന ഇഗ്‌നാസ് ഗോള്‍ഡ് സീഹറും ഹരാള്‍ഡ് മോട്‌സ്കിയും ജോസഫ് സ്‌കാച്ച്ട്ടും ആല്‍ഫ്രഡ് ഗ്യുല്ല്യൂമുമൊന്നും അബൂഹുറയ്‌റ(റ)യെ വെറുതെ വിട്ടിട്ടില്ല. 'ഹദീഥുകള്‍  ബോധപൂര്‍വം കെട്ടിയുണ്ടാ ക്കുന്നയാളായിരുന്നു അബൂഹുറയ്‌റ' യെന്ന് പറഞ്ഞത് ഇബ്‌നു ഹിശാമിന്റെ നബിചരിത്രം ഇംഗ്ലീഷിലേക്ക് വിവര്‍ത്തനം ചെയ്ത ആല്‍ഫ്രഡ് ഗ്യുല്ല്യൂം ആണ്. ഓറിയന്റലിസ്റ്റുകള്‍ നല്‍കിയ ആയുധങ്ങളുപയോഗിച്ച് അബൂഹുറയ്‌റ(റ)യെ കണ്ണും മൂക്കും നോക്കാതെ ആക്രമിക്കുകയാണ് മോഡേണിസ്റ്റുകൾ ചെയ്തത്.

ഓറിയന്റലിസ്റ്റുകള്‍ നല്‍കിയ ആയുധങ്ങളുപയോഗിച്ച് തത്ത്വദീക്ഷയില്ലാതെ അബൂഹുറയ്‌റയുടെ നേരെ ആക്രമണങ്ങള്‍ അഴിച്ചുവിടുന്ന വര്‍ തങ്ങള്‍ ചെയ്യുന്നത് ആ മഹാസ്വഹാബിയെ തെറിപറയുക മാത്രമല്ല, നബി(സ)യുടെ ചരിത്രപരതയെ നിഷേധിക്കാനായി ഇസ്‌ലാമി ന്റെ ശത്രുക്കളുണ്ടാക്കിയ ആയുധത്തെ രാകി മൂര്‍ച്ചപ്പെടുത്തുകകൂടിയാണെന്ന വസ്തുത പലപ്പോഴും മനസ്സിലാക്കുന്നില്ല. നബിയോടൊപ്പം ജീവിച്ച് നബിജീവിതത്തെ സൂക്ഷ്മമായി നിരീക്ഷിക്കുകയും പഠിക്കുകയും ചെയ്ത് നബി(സ)യുടെ പ്രശംസയ്ക്ക് പാത്രീഭൂതനായ(ഫത്ഹുല്‍ബാരി) ഒരു അനുചരനെ ജൂതനും ജൂതചാരനുമെല്ലാമായി ചിത്രീകരിച്ച് ഹദീഥ്‌നിഷേധത്തിന് സൈദ്ധാന്തികമായ അടിത്തറയുണ്ടാക്കുവാന്‍ ശ്രമിക്കുന്ന വര്‍ യഥാര്‍ഥത്തില്‍ വീണിരിക്കുന്നത് മുഹമ്മദ് നബി(സ) ഒരു ചരിത്രപുരുഷനല്ലെന്നും ഒരു മിത്തു മാത്രമാണെന്നും വരുത്തിത്തീര്‍ക്കുന്നതിന് ഇസ്‌ലാമിന്റെ ശത്രുക്കൾ കുഴിച്ച കുഴിയിലാണ്.

കേവലം മൂന്നുകൊല്ലം മാത്രം നബി(സ)യോടൊപ്പം സഹവസിച്ച അബൂഹുറയ്‌റ(റ)യാണ് ഏറ്റവുമധികം ഹദീഥുകള്‍ നിവേദനം ചെയ്ത തെന്ന വസ്തുതയാണ് അദ്ദേഹത്തിന്റെ സത്യസന്ധതയില്‍ സംശയിക്കുന്നവര്‍ പ്രധാനമായി മുന്നോട്ടുവെക്കുന്ന ആക്ഷേപം. അബൂഹുറയ്‌ റയില്‍(റ)നിന്ന് 5374 ഹദീഥുകള്‍ നിവേദനം ചെയ്യപ്പെട്ടിട്ടുണ്ടെന്നതും അദ്ദേഹമാണ് ഏറ്റവുമധികം ഹദീഥുകള്‍ നിവേദനം ചെയ്ത സ്വഹാ ബിയെന്നതും ശരിയാണ്. അബൂഹുറയ്‌റ(റ)യില്‍നിന്ന് എന്ന രൂപത്തില്‍ നിവേദനം ചെയ്യപ്പെട്ട ഹദീഥുകളില്‍ കെട്ടിയുണ്ടാക്കപ്പെട്ടവയാണെന്ന് ഉറപ്പുള്ളവയും ദുര്‍ബലമായ ഇസ്‌നാദോടു കൂടി ഉദ്ധരിക്കപ്പെട്ട അസ്വീകാര്യമായവയുമുണ്ടെന്നതും സത്യമാണ്. പക്ഷേ, അതെങ്ങനെയാണ് അബൂഹുറയ്‌റ(റ)യുടെ വിശ്വാസ്യതയെ ബാധിക്കുന്നത്? ഹദീഥ് നിവേദനത്തില്‍ നിപുണനും സത്യസന്ധനെന്ന് സമൂഹം അംഗീകരിച്ചയാളുമായ അബൂഹുറയ്‌റ(റ)യുടെ പേരില്‍ പില്‍ക്കാലത്തുള്ളവര്‍ കെട്ടിയുണ്ടാക്കിയ ഹദീഥുകള്‍ അദ്ദേഹത്തിന്റെ സത്യസന്ധതയെ ബാധിക്കുന്നതെങ്ങനെയാണെന്ന് വ്യക്തമാക്കു വാന്‍ വിമര്‍ശകര്‍ക്ക് കഴിഞ്ഞിട്ടില്ല. യഥാര്‍ഥത്തില്‍, അബൂഹുറയ്‌റ(റ)യുടെ പേരിലാണ് ഏറ്റവുമധികം വ്യാജഹദീഥുകള്‍ പ്രചരിച്ചിട്ടു ള്ളതെന്ന കാര്യം അദ്ദേഹത്തിന്റെ സത്യസന്ധതയെയും സ്വഹാബിമാര്‍ക്കും താബിഉകള്‍ക്കുമിടയിലുണ്ടായിരുന്ന അദ്ദേഹത്തിന്റെ സ്വീകാര്യതയുമല്ലേ വ്യക്തമാക്കുന്നത്? വ്യാജ ഹദീഥ് നിര്‍മാതാക്കള്‍ തങ്ങളുടെ ഉല്‍പന്നം ജനങ്ങള്‍ക്കിടയില്‍ സ്വീകാര്യമാവുന്നതിനായി സമൂഹത്തില്‍ ഏറ്റവുമധികം ആദരിക്കപ്പെടുകയും അംഗീകരിക്കപ്പെടുകയും ചെയ്യുന്ന നിവേദകരിലേക്കായിരിക്കും ചേര്‍ത്തിപ്പറയുക യെന്നുറപ്പാണ്. അബൂഹുറയ്‌റ(റ)യില്‍നിന്ന് നിവേദനം ചെയ്യപ്പെട്ടതാണെങ്കില്‍ അത് സമൂഹത്തില്‍ വേഗം അംഗീകരിക്കപ്പെടുമെന്നതിനാ ലാണല്ലോ വ്യാജ ഹദീഥ് നിര്‍മാതാക്കള്‍ അത് അദ്ദേഹത്തില്‍നിന്നാണെന്ന മട്ടില്‍ ഉദ്ധരിക്കുന്നത്. അബൂഹുറയ്‌റ(റ)യുടെ പേരില്‍ വര്‍ധമാന മായ തോതില്‍ പ്രചരിപ്പിക്കപ്പെട്ട വ്യാജഹദീഥുകള്‍ അദ്ദേഹത്തിന്റെ സത്യസന്ധതയെ ചോദ്യം ചെയ്യുകയല്ല, പ്രത്യുത സ്വീകാര്യതയെ ഉറപ്പിക്കുകയാണ് ചെയ്യുന്നതെന്ന് സാരം.

യമനിലെ ബനൂദൗസ് ഗോത്രക്കാരനായിരുന്ന അബൂഹുറയ്‌റ(റ) തന്റെ ഗോത്രക്കാരനും നബി(സ)യുടെ അടുത്തെത്തി ഇസ്‌ലാം സ്വീകരിച്ച യാളുമായ തുഫൈലുബ്‌നു അംറിന്റെ പ്രബോധനം വഴി ഹിജ്‌റക്ക് മുമ്പുതന്നെ ഇസ്‌ലാം സ്വീകരിച്ചിരുന്നുവെങ്കിലും ഹിജ്‌റ ഏഴാം വര്‍ ഷം നടന്ന ഖൈബര്‍ യുദ്ധത്തിന്റെ സന്ദര്‍ഭത്തിലാണ് നബി(സ)യുമായി സന്ധിച്ചത്. അന്നുമുതല്‍ നബി(സ)യുടെ മരണം വരെ അദ്ദേഹം പൂര്‍ണമായും നബി(സ)യോടൊപ്പമായിരുന്നു. നബി(സ)യെ വിട്ടുപിരിയാതെ പള്ളിയുടെ തിണ്ണയില്‍ കഴിഞ്ഞിരുന്നവരായ അസ്ഹാ ബു സ്‌സ്വുഫ്ഫയുടെ നേതാവായ അദ്ദേഹം പലപ്പോഴും ഭക്ഷണം കഴിച്ചിരുന്നത് പോലും നബി(സ)യോടൊപ്പമായിരുന്നു. എപ്പോഴെ ങ്കിലും കാണാതിരുന്നാല്‍ നബി(സ) അദ്ദേഹത്തെ അന്വേഷിച്ചിരുന്നുവെന്നും അബൂഹുറയ്‌റെയെയും അദ്ദേഹത്തിന്റെ ഗോത്രത്തെയും നബി(സ) പുകഴ്ത്തിപ്പറഞ്ഞിരുന്നുവെന്നുമെല്ലാം ഹദീഥുകളില്‍ നിന്ന് മനസ്സിലാവുന്നുണ്ട്. ഹിജ്‌റ ഏഴാം വര്‍ഷം സ്വഫര്‍ മാസത്തില്‍ ഖൈബറില്‍ വെച്ച് നബി(സ)യോടൊപ്പം കൂടിയതിനുശേഷം പതിനൊന്നാം വര്‍ഷം റബീഉല്‍ അവ്വല്‍ മാസത്തില്‍ നബി(സ) മരണപ്പെടുന്നതു വരെ യാത്രാസന്ദര്‍ഭങ്ങളൊഴിച്ച് ബാക്കി പൂര്‍ണമായും നബി(സ)യോടൊപ്പം തന്നെയായിരുന്ന അദ്ദേഹത്തിന് നബിജീവിതത്തെ കുറിച്ച് നന്നായി മനസ്സിലാക്കാന്‍ കഴിഞ്ഞിരിക്കുമെന്നുറപ്പാണ്. ഈ നാല് വര്‍ഷങ്ങള്‍ക്കിടയില്‍ യാത്രാ സമയത്തും മറ്റും വിട്ടുനിന്നതൊഴിച്ചാല്‍ താന്‍ നബി(സ)യുമായി പൂര്‍ണമായും ഒന്നിച്ചുനിന്നത് മൂന്നു വര്‍ഷമാണെന്ന് അബൂഹുറയ്‌റ(റ) തന്നെ വ്യക്തമാ ക്കിയിട്ടുണ്ട്. ഈ മൂന്നു വര്‍ഷം നബിജീവിതത്തിന്റെ അവസാനനാളുകളായിരുന്നുവെന്ന കാര്യം പ്രത്യേകം ഓര്‍ക്കേണ്ടതുണ്ട്. കര്‍മകാ ര്യങ്ങള്‍ മറ്റുള്ളവര്‍ക്ക് പറഞ്ഞുകൊടുക്കുവാന്‍ നബി(സ) ഏറെ സമയം കണ്ടെത്തിയിരുന്ന നാളുകളാണവ.

മക്കാവിജയവും ഖൈബര്‍ യുദ്ധവും കഴിഞ്ഞതോടെ എതിരാളികളുടെ ശക്തി ക്ഷയിക്കുകയും ഇസ്‌ലാമിലേക്ക് ആളുകള്‍ കൂട്ടം കൂട്ടമായി വന്നുകൊണ്ടിരിക്കുകയും ചെയ്തപ്പോള്‍ മുസ്‌ലിംകള്‍ക്ക് ഇസ്‌ലാമിെനക്കുറിച്ച് പറഞ്ഞുകൊടുക്കുന്നതിലും കര്‍മകാര്യങ്ങള്‍ വിശദീകരിച്ചുകൊടുക്കുന്നതിലും നബി(സ) കൂടുതല്‍ ബദ്ധശ്രദ്ധനായി. തിണ്ണവാസിയായി നബി(സ)യോടൊപ്പമുണ്ടായിരുന്ന അബൂഹുറൈറ(റ) ഈ അവസരങ്ങള്‍ക്കെല്ലാം ദൃക്‌സാക്ഷിയായി. ഇസ്‌ലാമിക ജീവിതക്രമത്തെപ്പറ്റി നബി(സ)യില്‍നിന്ന് നേരിട്ടു പഠിക്കുവാന്‍ അവസരം ലഭിച്ച അബൂഹുറയ്‌റ(റ) അത് ഓര്‍ത്തുവെക്കുവാനും അടുത്ത തലമുറക്ക് പറഞ്ഞുകൊടുക്കുവാനും ശ്രദ്ധിച്ചതിനാലാണ് അദ്ദേഹത്തിലൂടെ കൂടുതല്‍ ഹദീഥുകള്‍ നിവേ ദനം ചെയ്യപ്പെട്ടത്. നീണ്ട മൂന്നു വര്‍ഷക്കാലം നബി(സ)യുടെ വാക്കുകളും പ്രവര്‍ത്തനങ്ങളും പെരുമാറ്റങ്ങളും നിരന്തരമായി ശ്രദ്ധിക്കു കയും സൂക്ഷ്മമായി ഓര്‍ത്തുവെക്കുകയും ചെയ്ത ഒരാള്‍ക്ക് നിവേദനം ചെയ്യാനാവുന്നതിലും കൂടുതല്‍ ഹദീഥുകള്‍ അബൂഹുറയ്‌റ(റ) നിവേദനം ചെയ്തിട്ടില്ലെന്നതാണ് വാസ്തവം.

അബൂഹുറൈറ(റ)യിലൂടെ നിവേദനം ചെയ്യപ്പെട്ട ഹദീഥുകളില്‍നിന്ന് വ്യാജമെന്ന് ഉറപ്പുള്ളവ ഒഴിവാക്കി ഇമാം അഹ്മദ്‌(റഹ്) രേഖപ്പെടുത്തിയിട്ടുള്ളത് 3848 ഹദീഥുകളാണ്. അവയില്‍ പലതും അബൂഹുറയ്‌റയില്‍ നിന്ന് വ്യത്യ സ്ത ഇസ്‌നാദുകളില്‍ നിവേദനം ചെയ്യപ്പെട്ട ഒരേ മത്‌നിന്റെ ആവര്‍ത്തനങ്ങളാണ്. ഇവയെയെല്ലാം സ്വതന്ത്ര ഹദീഥുകളായി പരിഗണി ച്ചാല്‍ പോലും നബിജീവിതത്തെ സൂക്ഷ്മമായി നിരീക്ഷിച്ച ഒരാള്‍ക്ക് ഇത്രയും ഹദീഥുകള്‍ നിവേദനം ചെയ്യാന്‍ കഴിയില്ലെന്ന് പറയാന്‍ എങ്ങനെ പറ്റും? മൂന്നു വര്‍ഷങ്ങളിലുള്ള ആയിരത്തിലധികം ദിവസവും നബി(സ)യെ നിരീക്ഷിച്ച ഒരാള്‍ക്ക് ഒരു ദിവസത്തില്‍ നബി(സ) യില്‍ നിന്ന് ശരാശരി നാല് കാര്യങ്ങളില്‍ പഠിക്കാന്‍ കഴിഞ്ഞുവെന്നത് വലിയൊരു അത്ഭുതമൊന്നുമല്ല. സംഭവബഹുലമായ നബിജീവി തത്തിന്റെ അവസാനനാളുകളില്‍ നബി(സ)യുടെ സന്തതസഹചാരിയായി അദ്ദേഹത്തോടൊപ്പമുണ്ടായിരുന്ന ഒരാള്‍ക്ക് ആ ജീവിതത്തില്‍ നിന്ന് ഒരു ദിവസം ശരാശരി നാലു കാര്യങ്ങള്‍ പഠിക്കാന്‍ കഴിഞ്ഞത് അസംഭവ്യമാണെന്ന് പറയാന്‍ ഓറിയന്റലിസമുപ യോഗിക്കുന്ന മാനദണ്ഡമെന്താണ്? ശാസ്ത്രീയാപഗ്രഥനത്തിന്റെ അകമ്പടിയൊന്നുമില്ലാതെത്തന്നെ ആര്‍ക്കും സംഭവ്യമെന്ന് ബോധ്യപ്പെടുന്ന കാര്യങ്ങളെ പ്പോലും നിഷേധിക്കുവാന്‍ ബുദ്ധിജീവനത്തിന്റെ മൊത്തക്കുത്തക അവകാശപ്പെടുന്നവരെ പ്രേരിപ്പിക്കുന്നതിന് പിന്നില്‍ സത്യമതത്തോടു ള്ള അന്ധമായ വിരോധമല്ലാതെ മറ്റെന്താണ്?

അബൂഹുറൈറ(റ)യെ ഹദീഥ് നിര്മാതാവായി പരിചചയപ്പെടുത്തതാൻ ശ്രമിക്കുന്നവർ ചരിത്രത്തോടോ സത്യങ്ങളോടോ നീതി പുലർത്താത്തവരാണെന്ന് സാരം.

ടുന്ന ഇഗ്‌നാസ് ഗോള്‍ഡ് സീഹറും(1) ഹരാള്‍ഡ് മോട്‌സ്കി(2) യും ജോ സഫ് സ്‌കാച്ച്ട്ടും(3) ആല്‍ഫ്രഡ് ഗ്യുല്ല്യൂമുമൊന്നും(4) അബൂഹുറയ്‌റ(റ)യെ വെറുതെ വിട്ടിട്ടില്ല. 'ഹദീഥുകള്‍  ബോധപൂര്‍വം കെട്ടിയുണ്ടാ ക്കുന്നയാളായിരുന്നു അബൂഹുറയ്‌റ'(5) യെന്നാണ് ഇബ്‌നു ഹിശാമിന്റെ നബിചരിത്രം ഇംഗ്ലീഷിലേക്ക് വിവര്‍ത്തനം ചെയ്തുകൊണ്ട് മുസ്‌ലിം ലോകത്തിന് ഏറെ വിലപ്പെട്ട സംഭാവന നല്‍കിയ വ്യക്തിത്വമായി കൊട്ടിഘോഷിക്കപ്പെടുന്ന ആല്‍ഫ്രഡ് ഗ്യുല്ല്യൂം എഴുതുന്നത്. ഓറിയന്റലിസ്റ്റുകള്‍ നല്‍കിയ ആയുധങ്ങളുപയോഗിച്ച് അബൂഹുറയ്‌റ(റ)യെ കണ്ണും മൂക്കും നോക്കാതെ ആക്രമിച്ചത് യഥാര്‍ഥത്തില്‍ മുസ്‌ലിം മോഡേണിസ്റ്റുകളായിരുന്നു. അബൂഹുറയ്‌റയെ തെറി വിളിക്കുന്ന കാര്യത്തിലും അദ്ദേഹത്തിനെതിരെ 'തെളിവുകള്‍' കണ്ടെത്തു ന്ന കാര്യത്തിലും അവര്‍ ഓറിയന്റലിസ്റ്റുകളെ തോല്‍പിച്ചുവെന്ന് വേണമെങ്കില്‍ പറയാം. തന്റെ മാസ്റ്റര്‍ പീസായി അറിയപ്പെടുന്ന അദ്വ്‌ വാഉന്‍ അലാസുന്നത്തില്‍ മുഹമ്മദിയ്യഃയെക്കൂടാതെ അബൂഹുറയ്‌റ(റ)യെ തെറിപറയാന്‍ വേണ്ടിമാത്രം 'ശൈഖുല്‍ മദീറ അബൂഹുറയ്‌റ' യെന്ന ഒരു ഗ്രന്ഥം കൂടി രചിച്ചിട്ടുണ്ട് ആധുനിക മുസ്‌ലിം ഹദീഥ് നിഷേധികളില്‍ അഗ്രേസരനായി അറിയപ്പെടുന്ന മഹ്മൂദ് അബൂ റയ്യ(6). അദ്ദേഹത്തെ പിന്‍തുടര്‍ന്ന്, മോഡേണിസ്റ്റ് ചിന്തകള്‍ക്ക് അറബ്‌നാടുകളില്‍ പ്രചാരം സിദ്ധിക്കുന്നതിനായി പരിശ്രമിച്ച മുഹമ്മദ് അമീനി ന്റെ ഫജ്‌റുല്‍ ഇസ്‌ലാമിലും അബൂഹുറയ്‌റ(റ)യെ അതിരുകള്‍ ലംഘിച്ച് വിമര്‍ശിക്കുകയും കളിയാക്കുകയും ചെയ്യുന്നുണ്ട്.(7)

ഓറിയന്റലിസ്റ്റുകള്‍ നല്‍കിയ ആയുധങ്ങളുപയോഗിച്ച് തത്ത്വദീക്ഷയില്ലാതെ അബൂഹുറയ്‌റയുടെ നേരെ ആക്രമണങ്ങള്‍ അഴിച്ചുവിടുന്ന വര്‍ തങ്ങള്‍ ചെയ്യുന്നത് ആ മഹാസ്വഹാബിയെ തെറിപറയുക മാത്രമല്ല, നബി(സ)യുടെ ചരിത്രപരതയെ നിഷേധിക്കാനായി ഇസ്‌ലാമി ന്റെ ശത്രുക്കളുണ്ടാക്കിയ ആയുധത്തെ രാകി മൂര്‍ച്ചപ്പെടുത്തുകകൂടിയാണെന്ന വസ്തുത പലപ്പോഴും മനസ്സിലാക്കുന്നില്ല. നബിയോടൊപ്പം ജീവിച്ച് നബിജീവിതത്തെ സൂക്ഷ്മമായി നിരീക്ഷിക്കുകയും പഠിക്കുകയും ചെയ്ത് നബി(സ)യുടെ പ്രശംസയ്ക്ക് പാത്രീഭൂതനായ(8) ഒരു അനുചരനെ ജൂതനും ജൂതചാരനുമെല്ലാമായി ചിത്രീകരിച്ച് ഹദീഥ്‌നിഷേധത്തിന് സൈദ്ധാന്തികമായ അടിത്തറയുണ്ടാക്കുവാന്‍ ശ്രമിക്കുന്ന വര്‍ യഥാര്‍ഥത്തില്‍ വീണിരിക്കുന്നത് ഇഗ്‌നാസ് ഗോള്‍ഡ് സീഹര്‍ എന്ന യഥാര്‍ഥ ജൂതനും അദ്ദേഹത്തിന്റെ പിന്‍ഗാമികളുംകൂടി കുഴിച്ച കുഴിയിലാണ്. മുഹമ്മദ് നബി(സ) ഒരു ചരിത്രപുരുഷനല്ലെന്നും ഒരു മിത്തു മാത്രമാണെന്നും വരുത്തിത്തീര്‍ക്കുന്നതിന് ഇസ്‌ലാമിന്റെ ശത്രുക്കള്‍ കുഴിച്ച കുഴിയില്‍ വീണുകിടക്കുമ്പോഴും തങ്ങള്‍ ഇസ്‌ലാമിനും മുഹമ്മദ് നബി(സ)ക്കും സേവനം ചെയ്യുകയാണെന്നാണ് ഇവര്‍ വിളിച്ചു പറഞ്ഞുകൊണ്ടിരിക്കുന്നത് എന്നതാണ് കഷ്ടം.

കേവലം മൂന്നുകൊല്ലം മാത്രം നബി(സ)യോടൊപ്പം സഹവസിച്ച അബൂഹുറയ്‌റ(റ)യാണ് ഏറ്റവുമധികം ഹദീഥുകള്‍ നിവേദനം ചെയ്ത തെന്ന വസ്തുതയാണ് അദ്ദേഹത്തിന്റെ സത്യസന്ധതയില്‍ സംശയിക്കുന്നവര്‍ പ്രധാനമായി മുന്നോട്ടുവെക്കുന്ന ആക്ഷേപം. അബൂഹുറയ്‌ റയില്‍(റ)നിന്ന് 5374 ഹദീഥുകള്‍ നിവേദനം ചെയ്യപ്പെട്ടിട്ടുണ്ടെന്നതും(9) അദ്ദേഹമാണ് ഏറ്റവുമധികം ഹദീഥുകള്‍ നിവേദനം ചെയ്ത സ്വഹാ ബിയെന്നതും ശരിയാണ്. ബുഖാരിയില്‍ മാത്രം അബൂഹുറയ്‌റ(റ) നിവേദനം ചെയ്ത 446 ഹദീഥുകളുണ്ട്.(10) ബുഖാരിയും മുസ്‌ലിമും കൂടി ആകെ 609 ഹദീഥുകളും രണ്ടു ഗ്രന്ഥങ്ങളിലും ഒരേപോലെ 326 ഹദീഥുകളും അദ്ദേഹത്തില്‍നിന്ന് നിവേദനം ചെയ്യപ്പെട്ടിട്ടുണ്ട്. ഇമാം അഹ്മ ദ്ബ്‌നു ഹന്‍ബലിന്റെ മുസ്‌നദില്‍ അബൂഹുറയ്‌റയില്‍ നിന്ന് 3848 ഹദീഥുകള്‍ നിവേദനം ചെയ്തിട്ടുണ്ട്.(11) അബൂഹുറയ്‌റ(റ)യില്‍നിന്ന് എന്ന രൂപത്തില്‍ നിവേദനം ചെയ്യപ്പെട്ട 5374 ഹദീഥുകളില്‍ കെട്ടിയുണ്ടാക്കപ്പെട്ടവയാണെന്ന് ഉറപ്പുള്ളവയും ദുര്‍ബലമായ ഇസ്‌നാദോടു കൂടി ഉദ്ധരിക്കപ്പെട്ട അസ്വീകാര്യമായവയുമുണ്ടെന്നത് ശരിയാണ്. പക്ഷേ, അതെങ്ങനെയാണ് അബൂഹുറയ്‌റ(റ)യുടെ വിശ്വാസ്യതയെ ബാധിക്കുന്നത്? ഹദീഥ് നിവേദനത്തില്‍ നിപുണനും സത്യസന്ധനെന്ന് സമൂഹം അംഗീകരിച്ചയാളുമായ അബൂഹുറയ്‌റ(റ)യുടെ പേരില്‍ പില്‍ക്കാലത്തുള്ളവര്‍ കെട്ടിയുണ്ടാക്കിയ ഹദീഥുകള്‍ അദ്ദേഹത്തിന്റെ സത്യസന്ധതയെ ബാധിക്കുന്നതെങ്ങനെയാണെന്ന് വ്യക്തമാക്കു വാന്‍ വിമര്‍ശകര്‍ക്ക് കഴിഞ്ഞിട്ടില്ല. യഥാര്‍ഥത്തില്‍, അബൂഹുറയ്‌റ(റ)യുടെ പേരിലാണ് ഏറ്റവുമധികം വ്യാജഹദീഥുകള്‍ പ്രചരിച്ചിട്ടു ള്ളതെന്ന കാര്യം അദ്ദേഹത്തിന്റെ സത്യസന്ധതയെയും സ്വഹാബിമാര്‍ക്കും താബിഉകള്‍ക്കുമിടയിലുണ്ടായിരുന്ന അദ്ദേഹത്തിന്റെ സ്വീകാര്യതയുമല്ലേ വ്യക്തമാക്കുന്നത്? വ്യാജ ഹദീഥ് നിര്‍മാതാക്കള്‍ തങ്ങളുടെ ഉല്‍പന്നം ജനങ്ങള്‍ക്കിടയില്‍ സ്വീകാര്യമാവുന്നതിനായി സമൂഹത്തില്‍ ഏറ്റവുമധികം ആദരിക്കപ്പെടുകയും അംഗീകരിക്കപ്പെടുകയും ചെയ്യുന്ന നിവേദകരിലേക്കായിരിക്കും ചേര്‍ത്തിപ്പറയുക യെന്നുറപ്പാണ്. അബൂഹുറയ്‌റ(റ)യില്‍നിന്ന് നിവേദനം ചെയ്യപ്പെട്ടതാണെങ്കില്‍ അത് സമൂഹത്തില്‍ വേഗം അംഗീകരിക്കപ്പെടുമെന്നതിനാ ലാണല്ലോ വ്യാജ ഹദീഥ് നിര്‍മാതാക്കള്‍ അത് അദ്ദേഹത്തില്‍നിന്നാണെന്ന മട്ടില്‍ ഉദ്ധരിക്കുന്നത്. അബൂഹുറയ്‌റ(റ)യുടെ പേരില്‍ വര്‍ധമാന മായ തോതില്‍ പ്രചരിപ്പിക്കപ്പെട്ട വ്യാജഹദീഥുകള്‍ അദ്ദേഹത്തിന്റെ സത്യസന്ധതയെ ചോദ്യം ചെയ്യുകയല്ല, പ്രത്യുത സ്വീകാര്യതയെ ഉറപ്പിക്കുകയാണ് ചെയ്യുന്നതെന്ന് സാരം.

യമനിലെ ബനൂദൗസ് ഗോത്രക്കാരനായിരുന്ന അബൂഹുറയ്‌റ(റ) തന്റെ ഗോത്രക്കാരനും നബി(സ)യുടെ അടുത്തെത്തി ഇസ്‌ലാം സ്വീകരിച്ച യാളുമായ തുഫൈലുബ്‌നു അംറിന്റെ പ്രബോധനം വഴി ഹിജ്‌റക്ക് മുമ്പുതന്നെ ഇസ്‌ലാം സ്വീകരിച്ചിരുന്നുവെങ്കിലും ഹിജ്‌റ ഏഴാം വര്‍ ഷം നടന്ന ഖൈബര്‍ യുദ്ധത്തിന്റെ സന്ദര്‍ഭത്തിലാണ് നബി(സ)യുമായി സന്ധിച്ചത്. അന്നുമുതല്‍ നബി(സ)യുടെ മരണം വരെ അദ്ദേഹം പൂര്‍ണമായും നബി(സ)യോടൊപ്പമായിരുന്നു.(12) നബി(സ)യെ വിട്ടുപിരിയാതെ പള്ളിയുടെ തിണ്ണയില്‍ കഴിഞ്ഞിരുന്നവരായ അസ്ഹാ ബു സ്‌സ്വുഫ്ഫ(13)യുടെ നേതാവായ അദ്ദേഹം പലപ്പോഴും ഭക്ഷണം കഴിച്ചിരുന്നത് പോലും നബി(സ)യോടൊപ്പമായിരുന്നു. എപ്പോഴെ ങ്കിലും കാണാതിരുന്നാല്‍ നബി(സ) അദ്ദേഹത്തെ അന്വേഷിച്ചിരുന്നുവെന്നും(14) അബൂഹുറയ്‌റെയെയും അദ്ദേഹത്തിന്റെ ഗോത്രത്തെയും നബി(സ) പുകഴ്ത്തിപ്പറഞ്ഞിരുന്നുവെന്നുമെല്ലാം(15) ഹദീഥുകളില്‍ നിന്ന് മനസ്സിലാവുന്നുണ്ട്. ഹിജ്‌റ ഏഴാം വര്‍ഷം സ്വഫര്‍ മാസത്തില്‍ ഖൈബറില്‍ വെച്ച് നബി(സ)യോടൊപ്പം കൂടിയതിനുശേഷം പതിനൊന്നാം വര്‍ഷം റബീഉല്‍ അവ്വല്‍ മാസത്തില്‍ നബി(സ) മരണപ്പെടുന്നതു വരെ യാത്രാസന്ദര്‍ഭങ്ങളൊഴിച്ച് ബാക്കി പൂര്‍ണമായും നബി(സ)യോടൊപ്പം തന്നെയായിരുന്ന അദ്ദേഹത്തിന് നബിജീവിതത്തെ കുറിച്ച് നന്നായി മനസ്സിലാക്കാന്‍ കഴിഞ്ഞിരിക്കുമെന്നുറപ്പാണ്. ഹിജ്‌റ 7 മുതല്‍ 11 വരെയുള്ള നാല് വര്‍ഷങ്ങള്‍ക്കിടയില്‍ യാത്രാ സമയത്തും മറ്റും വിട്ടുനിന്നതൊഴിച്ചാല്‍ താന്‍ നബി(സ)യുമായി പൂര്‍ണമായും ഒന്നിച്ചുനിന്നത് മൂന്നു വര്‍ഷമാണെന്ന് അബൂഹുറയ്‌റ(റ) തന്നെ വ്യക്തമാ ക്കിയിട്ടുണ്ട്.(16) ഈ മൂന്നു വര്‍ഷം നബിജീവിതത്തിന്റെ അവസാനനാളുകളായിരുന്നുവെന്ന കാര്യം പ്രത്യേകം ഓര്‍ക്കേണ്ടതുണ്ട്. കര്‍മകാ ര്യങ്ങള്‍ മറ്റുള്ളവര്‍ക്ക് പറഞ്ഞുകൊടുക്കുവാന്‍ നബി(സ) ഏറെ സമയം കണ്ടെത്തിയിരുന്ന നാളുകളാണവ. മക്കാവിജയവും ഖൈബര്‍ യുദ്ധവും കഴിഞ്ഞതോടെ എതിരാളികളുടെ ശക്തി ക്ഷയിക്കുകയും ഇസ്‌ലാമിലേക്ക് ആളുകള്‍ കൂട്ടം കൂട്ടമായി വന്നുകൊണ്ടിരിക്കുകയും ചെയ്തപ്പോള്‍ മുസ്‌ലിംകള്‍ക്ക് ഇസ്‌ലാമിെനക്കുറിച്ച് പറഞ്ഞുകൊടുക്കുന്നതിലും കര്‍മകാര്യങ്ങള്‍ വിശദീകരിച്ചുകൊടുക്കുന്നതിലും നബി(സ) കൂടുതല്‍ ബദ്ധശ്രദ്ധനായി. തിണ്ണവാസിയായി നബി(സ)യോടൊപ്പമുണ്ടായിരുന്ന അബൂഹുറൈറ(റ) ഈ അവസരങ്ങള്‍ക്കെല്ലാം ദൃക്‌സാക്ഷിയായി. ഇസ്‌ലാമിക ജീവിതക്രമത്തെപ്പറ്റി നബി(സ)യില്‍നിന്ന് നേരിട്ടു പഠിക്കുവാന്‍ അവസരം ലഭിച്ച അബൂഹുറയ്‌റ(റ) അത് ഓര്‍ത്തുവെക്കുവാനും അടുത്ത തലമുറക്ക് പറഞ്ഞുകൊടുക്കുവാനും ശ്രദ്ധിച്ചതിനാലാണ് അദ്ദേഹത്തിലൂടെ കൂടുതല്‍ ഹദീഥുകള്‍ നിവേ ദനം ചെയ്യപ്പെട്ടത്. നീണ്ട മൂന്നു വര്‍ഷക്കാലം നബി(സ)യുടെ വാക്കുകളും പ്രവര്‍ത്തനങ്ങളും പെരുമാറ്റങ്ങളും നിരന്തരമായി ശ്രദ്ധിക്കു കയും സൂക്ഷ്മമായി ഓര്‍ത്തുവെക്കുകയും ചെയ്ത ഒരാള്‍ക്ക് നിവേദനം ചെയ്യാനാവുന്നതിലും കൂടുതല്‍ ഹദീഥുകള്‍ അബൂഹുറയ്‌റ(റ) നിവേദനം ചെയ്തിട്ടില്ലെന്നതാണ് വാസ്തവം. അദ്ദേഹത്തിലൂടെ നിവേദനം ചെയ്യപ്പെട്ട 5374 ഹദീഥുകളില്‍നിന്ന് വ്യാജമെന്ന് ഉറപ്പുള്ളവ ഒഴിവാക്കി ഇമാം അഹ്മദ്‌(റഹ്) രേഖപ്പെടുത്തിയിട്ടുള്ളത് 3848 ഹദീഥുകളാണ്.(17) അവയില്‍ പലതും അബൂഹുറയ്‌റയില്‍ നിന്ന് വ്യത്യ സ്ത ഇസ്‌നാദുകളില്‍ നിവേദനം ചെയ്യപ്പെട്ട ഒരേ മത്‌നിന്റെ ആവര്‍ത്തനങ്ങളാണ്. ഇവയെയെല്ലാം സ്വതന്ത്ര ഹദീഥുകളായി പരിഗണി ച്ചാല്‍ പോലും നബിജീവിതത്തെ സൂക്ഷ്മമായി നിരീക്ഷിച്ച ഒരാള്‍ക്ക് ഇത്രയും ഹദീഥുകള്‍ നിവേദനം ചെയ്യാന്‍ കഴിയില്ലെന്ന് പറയാന്‍ എങ്ങനെ പറ്റും? മൂന്നു വര്‍ഷങ്ങളിലുള്ള ആയിരത്തിലധികം ദിവസവും നബി(സ)യെ നിരീക്ഷിച്ച ഒരാള്‍ക്ക് ഒരു ദിവസത്തില്‍ നബി(സ) യില്‍ നിന്ന് ശരാശരി നാല് കാര്യങ്ങളില്‍ പഠിക്കാന്‍ കഴിഞ്ഞുവെന്നത് വലിയൊരു അത്ഭുതമൊന്നുമല്ല. സംഭവബഹുലമായ നബിജീവി തത്തിന്റെ അവസാനനാളുകളില്‍ നബി(സ)യുടെ സന്തതസഹചാരിയായി അദ്ദേഹത്തോടൊപ്പമുണ്ടായിരുന്ന ഒരാള്‍ക്ക് ആ ജീവിതത്തില്‍ നിന്ന് ഒരു ദിവസം ശരാശരി നാലു കാര്യങ്ങള്‍ പഠിക്കാന്‍ കഴിഞ്ഞത് അസംഭവ്യമാണെന്ന് പറയാന്‍ ഓറിയന്റലിസമുപ യോഗിക്കുന്ന മാനദണ്ഡമെന്താണ്? ശാസ്ത്രീയാപഗ്രഥനത്തിന്റെ അകമ്പടിയൊന്നുമില്ലാതെത്തന്നെ ആര്‍ക്കും സംഭവ്യമെന്ന് ബോധ്യപ്പെടുന്ന കാര്യങ്ങളെ പ്പോലും നിഷേധിക്കുവാന്‍ ബുദ്ധിജീവനത്തിന്റെ മൊത്തക്കുത്തക അവകാശപ്പെടുന്നവരെ പ്രേരിപ്പിക്കുന്നതിന് പിന്നില്‍ സത്യമതത്തോടു ള്ള അന്ധമായ വിരോധമല്ലാതെ മറ്റെന്താണ്?

കുറിപ്പുകൾ

  1. Ignaz Goldziher: Muslim Studies, Vol.2, Page 41-48.
  2. Harald Motzki: The Biography of Muhammed, The Issue of Sources, Brill, 2000.
  3. Joseph Schacht: The Origins of Muhammadan Juriprudance, ACLS, 2001 (www. historybook.org).
  4. Guillaume: Traditions of Islam: An Introduction to the studies of the Hadith Literature, Montana, 2003.
  5. Ibid Page 78.
  6. sunnidefence.com.
  7. hadithcommunity-wordpress.com.
  8. ഫത്ഹുല്‍ബാരി.
  9. തഖിയ്യിബ്‌നു മഖലദിന്റെ മുസ്‌നദില്‍നിന്ന് ഡോക്ടര്‍ മുസ്തഫസ്സബാഈ ഉദ്ധരിച്ചത്. സുന്നത്തും ഇസ്‌ലാം ശീഅത്തില്‍ അതിന്റെ സ്ഥാനവും, പുറം
  10. ഫത്ഹുല്‍ബാരി.
  11. മുസ്‌നദ് ഇമാം അഹ്മദ് (ekabakti.com).
  12. ഹാഫിദ് അഹ്മദ്ബ്‌നു അലിയുബ്‌നു ഹജറുല്‍ അസ്ഖലാനി: അല്‍ ഇസ്വാബതു ഫീ തംയീസി സ്‌സ്വഹാബഃ
  13. 'അസ്ഹാബുസ്‌സ്വുഫ്ഫ'യെന്നാല്‍ തിണ്ണവാസികള്‍ എന്നര്‍ഥം. മറ്റു ജോലികളില്‍ വ്യാപൃതരാവാതെ മദീനയിലെ പള്ളിയുടെ തിണ്ണയില്‍ കഴിഞ്ഞിരുന്നവരെ വിളിച്ചിരുന്ന പേരാണിത്. ക്വുര്‍ആനില്‍ 'ഭൂമിയില്‍ സഞ്ചരിച്ച് ഉപജീവനം നേടാന്‍ കഴിയാത്തവിധം അല്ലാഹുവിന്റെ മാര്‍ഗത്തില്‍ ബന്ധിതരായിരിക്കുന്നവര്‍' (2:273) എന്ന് പ്രശംസിച്ച് പറഞ്ഞിരിക്കുന്നത് ഇവരെക്കുറിച്ചാണെന്ന് വ്യാഖ്യാതാക്കള്‍ പറഞ്ഞിട്ടുണ്ട്.
  14. സുനനു അബീദാവൂദ് (ekabakti.com).
  15. സ്വഹീഹുല്‍ ബുഖാരി, കിത്താബുല്‍ മഗാസി.
  16. സ്വഹീഹുല്‍ ബുഖാരി, ഫദാഇലുര്‍റസൂലി വ സ്വഹാബത്തിഹി.
  17. മുസ്‌നദ് ഇമാം അഹ്മദ് (ekabakti.com).

മുഹമ്മദ് നബിയിൽ(സ) നിന്ന് സ്വഹാബിമാർ നിവേദനം ചെയ്ത കാര്യങ്ങളാണ് ഹദീഥുകൾ. സ്വഹാബിമാരൊന്നും നബിയുടെ (സ) പേരിൽ കള്ളം പറയില്ലെന്ന സങ്കല്പത്തിന്റെ അടിസ്ഥാനത്തിലാണ് ഹദീഥ് നിദാനശാസ്ത്രം അതിന്റെ അടിത്തറ പണിതിരിക്കുന്നത്. ഈ സങ്കൽപം തന്നെ അടിസ്ഥാനരഹിതമല്ലേ? സ്വഹാബിമാരുടെ സത്യസന്ധതയിൽ വിശ്വാസികൾക്ക് സംശയമില്ലായിരിക്കാം. എന്നാൽ തികച്ചും വ്യക്തിനിഷ്ഠവും ആത്മനിഷ്ഠവുമായ ഈ വിശ്വാസം എങ്ങനെയാണ് ശാസ്ത്രീയമായിത്തത്തീരുന്നത്?

ഖുര്‍ആന്‍ ദൈവവചനവും മുഹമ്മദ് നബി(സ) ദൈവദൂതനുമാണെന്ന് മനസ്സിലാക്കുന്നവരെ സംബന്ധിച്ചിടത്തോളം സ്വഹാബിമാരുടെ സത്യസന്ധതയില്‍ യാതൊരു സംശയവുമുണ്ടാകുവാന്‍ തരമില്ല. മക്കയില്‍ വെച്ച് നബി(സ)യില്‍ വിശ്വസിക്കുകയും ത്യാഗങ്ങള്‍ സഹിച്ച് പലായനം നടത്തുകയും ചെയ്ത മുഹാജിറുകളും മദീനയില്‍ അവര്‍ക്ക് ആതിഥ്യമരുളുകയും അവിടെ ഒരു ഇസ്‌ലാമിക സമൂഹത്തിന് രൂപം നല്‍കാന്‍ സഹായിക്കുകയും ചെയ്ത അന്‍സ്വാറുകളും ഇവരുടെ പിന്‍ഗാമികളായി ഇസ്‌ലാമിലെത്തിച്ചേര്‍ന്നവരുമടങ്ങുന്ന സ്വഹാ ബീസഞ്ചയത്തെ പ്രശംസിക്കുകയും അവരില്‍ അല്ലാഹു സംപ്രീതനായിരിക്കുന്നുവെന്ന് പ്രഖ്യാപിക്കുകയും ചെയ്യുന്നുണ്ട്, ഖുര്‍ആന്‍: ''മുഹാജിറുകളില്‍ നിന്നും അന്‍സ്വാറുകളില്‍ നിന്നും ആദ്യമായി മുന്നോട്ട് വന്നവരും, സുകൃതം ചെയ്തുകൊണ്ട് അവരെ പിന്തുടര്‍ന്നവരും ആരോ അവരെപ്പറ്റി അല്ലാഹു സംതൃപ്തനായിരിക്കുന്നു. അവനെപ്പറ്റി അവരും സംതൃപ്തരായിരിക്കുന്നു. താഴ്ഭാഗത്ത് അരുവികള്‍ ഒഴുകിക്കൊണ്ടിരിക്കുന്ന സ്വര്‍ഗത്തോപ്പുകള്‍ അവര്‍ക്ക് അവന്‍ ഒരുക്കിവെക്കുകയും ചെയ്തിരിക്കുന്നു. എന്നെന്നും അവരതില്‍ നിത്യവാ സികളായിരിക്കും. അതത്രെ മഹത്തായ ഭാഗ്യം'' (9:100).

''വിശ്വസിക്കുകയും സ്വദേശം വെടിഞ്ഞ് പോകുകയും അല്ലാഹുവിന്റെ മാര്‍ഗത്തില്‍ സമരത്തില്‍ ഏര്‍പെടുകയും ചെയ്തവരും, അവ ര്‍ക്ക് അഭയം നല്‍കുകയും സഹായിക്കുകയും ചെയ്തവരും തന്നെയാണ് യഥാര്‍ഥത്തില്‍ സത്യവിശ്വാസികള്‍. അവര്‍ക്ക് പാപമോചനവും മാന്യമായ ഉപജീവനവും ഉണ്ടായിരിക്കും'' (8:74).

അനുചരന്‍മാരെക്കുറിച്ച പ്രവാചക പരാമര്‍ശങ്ങളിലും അവര്‍ സത്യസന്ധരും സന്‍മാര്‍ഗനിഷ്ഠരുമാണെന്ന വസ്തുത ഊന്നിപ്പറഞ്ഞത് കാണാനാവും. ''അബൂബുര്‍ദാ(റ)അബൂമൂസല്‍ അശ്അരിയി(റ)വില്‍ നിന്ന് നിവേദനം: നബി(സ)തന്റെ മുഖം ആകാശത്തേക്ക് ഉയര്‍ത്തി ക്കൊണ്ട് പറഞ്ഞു: ''നക്ഷത്രങ്ങള്‍ ആകാശത്തിനുള്ള സുരക്ഷിതത്വമാണ്. നക്ഷത്രങ്ങള്‍ നശിച്ചുകഴിഞ്ഞാല്‍ ആകാശത്തിന് മുന്നറിയിപ്പ് നല്‍ കപ്പെട്ടത് വന്നു ഭവിക്കുകയായി. ഞാന്‍ എന്റെ അനുചരന്മാര്‍ക്കുള്ള സുരക്ഷിതത്വമാണ്. ഞാന്‍ പോയിക്കഴിഞ്ഞാല്‍ എന്റെ അനുചര ന്മാര്‍ക്ക് മുന്നറിയിപ്പ് നല്‍കപ്പെട്ടത് വന്നു ഭവിക്കുകയായി. എന്റെ അനുചരന്മാര്‍ എന്റെ സമുദായത്തിനുള്ള സുരക്ഷിതത്വമാണ്. എന്റെ അനുചരന്മാര്‍ പോയിക്കഴിഞ്ഞാല്‍ എന്റെ സമുദായത്തിനും മുന്നറിയിപ്പ് നല്‍കപ്പെട്ടത് വന്നു ഭവിക്കുകയായി''(സ്വഹീഹുമുസ്‌ലിം, കിത്താബു ഫദാഇലി സ്‌സ്വഹാബ)

''അബ്ദുല്ലാ(റ)നിവേദനം: നബി(സ)പറഞ്ഞു: ''ജനങ്ങളില്‍ ഏറ്റവും ഉത്തമര്‍ എന്റെ തലമുറയാണ്. പിന്നീട് അവരെ തുടര്‍ന്ന് വരുന്നവരും പിന്നീട് അവരെ തുടര്‍ന്ന് വരുന്നവരും''(സ്വഹീഹുല്‍ ബുഖാരി, കിത്താബു സ്‌സ്വഹാബ.)

''അബൂസഈദ് അല്‍ ഖുദ്‌രി(റ)നിവേദനം: തിരുമേനി(സ)പറഞ്ഞു: ''എന്റെ അനുചരന്മാരെ നിങ്ങള്‍ പഴി പറയരുത്. നിങ്ങളില്‍ ഒരാള്‍ ഉഹ്ദ് മലയോളം സ്വര്‍ണം ചെലവഴിച്ചാലും അവരിലൊരാള്‍ ചെലവഴിച്ച ഒരു മുദ്ദിനോ (രണ്ട് കൈപ്പത്തികള്‍ ചേര്‍ത്തുവെച്ചുകൊണ്ടുള്ള ഒരു വാരല്‍) അതിന്റെ പകുതിക്കുപോലുമോ എത്തുകയില്ല''(സ്വഹീഹുല്‍ ബുഖാരി, കിത്താബു സ്‌സ്വഹാബ)

ക്വുര്‍ആനിന്റെയോ നബി വചനങ്ങളുടെയോ പ്രാമാണികത അംഗീകരിക്കാത്തവരെ സംബന്ധച്ചിടത്തോളം സ്വഹാബിക ളുടെ സത്യസന്ധതയ്ക്ക് അവ നല്‍കുന്ന സാക്ഷ്യം സ്വീകാര്യമാവില്ല.ഓറിയന്റലിസ്റ്റുകളുടെ ചരിത്രവിമര്‍ശനരീതി പ്രകാരം ഒരു കാലഘട്ടത്തിലെ ജനങ്ങളുടെയോ പ്രത്യേകമായ ഒരു ആദര്‍ശത്തിന്റെ വക്താക്കളുടെയോ സത്യസന്ധത നിര്‍ണയിക്കുവാനുള്ള മാനദണ്ഡമെന്താണെന്ന് ആരും വ്യക്തമാക്കിയിട്ടുമില്ല. വ്യക്തികളുടെ സത്യസന്ധത പരിശോധിച്ച് അവര്‍ പറഞ്ഞ കാര്യങ്ങളിലെ നെല്ലും പതിരും വേര്‍തിരിക്കുന്ന ആത്മനിഷ്ഠമായ അപഗ്രഥന രീതി വസ്തുനിഷ്ഠവിശകലനത്തില്‍ മാത്രം ശ്രദ്ധ കേന്ദ്രീകരിക്കുന്ന ചരിത്രവിമര്‍ശകന്‍മാര്‍ക്ക് പരിചയമുള്ളതല്ല.

എന്നാല്‍ സ്വഹാബിമാരെപ്പറ്റി ഏതൊരാള്‍ക്കും മനസ്സിലാക്കാനാവുന്ന ചില വസ്തുതകളുണ്ട്. അവര്‍ ജീവിച്ച സമൂഹം അവരുടെ സത്യസന്ധത യ്ക്ക് അന്യോന്യം സാക്ഷികളായിരുന്നുവെന്നതാണ് അതില്‍ ഏറ്റവും പ്രധാനപ്പെട്ടത്. അവര്‍ പരസ്പരം വിശ്വസിക്കുകയും പ്രവാചകനെ ക്കുറിച്ച് അവരില്‍ ആരെങ്കിലുമൊരാള്‍ എന്തെങ്കിലും പറഞ്ഞാല്‍ അത് സത്യം തന്നെയാണെന്ന് കരുതുകയും ഇക്കാര്യത്തില്‍ അവരെല്ലാ വരും പരസ്പരം സഹകരിക്കുകയും ചെയ്തിരുന്നു. 'എന്നെക്കുറിച്ച് ആരെങ്കിലും ബോധപൂര്‍വം കളവുകളെന്തെങ്കിലും പറഞ്ഞാല്‍ നരകത്തില്‍ അവന്‍ അവന്റെ ഇരിപ്പിടം തയാറാക്കിക്കൊള്ളട്ടെ' (സ്വഹീഹുല്‍ ബുഖാരി, കിതാബുല്‍ ഇൽമ് ) എന്ന് നബിയിൽ (സ) നിന്ന് പഠിച്ചവരായിരുന്നു അവര്‍. അതുകൊണ്ടു തന്നെ, അവരില്‍പെട്ട ഒരാളും നബി(സ)യെക്കുറിച്ച് എന്തെങ്കിലുമൊരു കളവു പറയാന്‍ യാതൊരു സാധ്യതയുമില്ലെന്ന് അവരെല്ലാവരും പരസ്പരം അംഗീകരിച്ചിരുന്നു. അതുകൊണ്ടാണല്ലോ നബി(സ) പറഞ്ഞുവെന്നോ ചെയ്തുവെന്നോ അനുവദിച്ചുവെന്നോ ഏതെങ്കിലു മൊരു സ്വഹാബി പറഞ്ഞാല്‍ മറ്റുള്ളവര്‍ അത് ചോദ്യംചെയ്യാതെ അംഗീകരിച്ചുവന്നത്.

ഓറിയന്റലിസ്റ്റുകളുടെയും അവരിൽ നിന്ന് മതം പഠിച്ച ഹദീഥ്നിഷേധികളുടെയും ശക്തമായ കടന്നാക്രമണ ത്തിന് വിധേയനായ സ്വഹാബി അബൂഹുറയ്‌റ(റ)നബി(സ) പറഞ്ഞതായി ഉദ്ധരിച്ച കാര്യങ്ങള്‍ മറ്റു സ്വഹാബിമാര്‍ ചോദ്യം ചെയ്യാതെ സ്വീകരിച്ചതായി വ്യക്തമാക്കുന്ന നിരവധി ഹദീഥുകളുണ്ട്. സ്ത്രീകള്‍ പച്ചകുത്തുന്നതിനെക്കുറിച്ച് പ്രവാചകന്‍(സ)എന്തെങ്കിലും പറഞ്ഞി ട്ടുണ്ടോയെന്ന് ആരാഞ്ഞ ഉമറി(റ)നോട് പച്ചകുത്തുന്നത് വിരോധിച്ചുകൊണ്ടുള്ള നബികല്‍പനയെക്കുറിച്ച് അബൂഹുറയ്‌റ(റ) തെര്യപ്പെ ടുത്തുകയും അതനുസരിച്ച് ഉമര്‍(റ)വിധിച്ചതായും വ്യക്തമാക്കുന്ന സ്വഹീഹുല്‍ ബുഖാരിയിലെ ഹദീഥ് ഉദാഹരണം.(കിതാബുല്‍ ലിബാസ്.) ഒരു സ്വഹാബി യുടെ സാക്ഷ്യം മറ്റു സ്വഹാബിമാര്‍ ചോദ്യം ചെയ്യാതെ അംഗീകരിച്ചിരുന്നുവെന്നാണല്ലോ ഇത് വ്യക്തമാക്കുന്നത്.

നബി(സ)യുടെ ജീവിതത്തെക്കുറിച്ച പരാമര്‍ശങ്ങളില്‍ സ്വഹാബിമാരാരും കളവു പറയുകയില്ലെന്ന് പരസ്പരം അംഗീകരിച്ചിരുന്നുവെന്ന് പറഞ്ഞാല്‍ ഓരോരുത്തരുടെയും സത്യസന്ധതയ്ക്ക് ഒരു ലക്ഷത്തിലധികം പേരുടെ സാക്ഷ്യമുണ്ടെന്നാണര്‍ഥം. നബി(സ)യുടെ അറഫാ പ്രസംഗത്തിനെത്തിയ സ്വഹാബിമാരുടെ എണ്ണം ഒരു ലക്ഷത്തിലധികമായിരുന്നുവെന്നാണ് കരുതപ്പെടുന്നത്. ഓരോരുത്തരുടെയും സത്യ സന്ധതയ്ക്ക് ഒരു ലക്ഷത്തിലധികം പേരുടെ സാക്ഷ്യം ലഭിച്ചാലും, വസ്തുനിഷ്ഠമായ തെളിവുകള്‍ മാത്രമെ അംഗീകരിക്കൂവെന്ന് വാശിപി ടിക്കുന്ന ഓറിയന്റലിസ്റ്റുകള്‍ക്ക്  അത് അംഗീകരിക്കുവാന്‍ കഴിഞ്ഞുകൊള്ളണമെന്നില്ല. എന്നാല്‍ ഹദീഥുകള്‍ നിവേദനം ചെയ്ത ഓരോ സ്വഹാബിയുടെയും സത്യസന്ധതയ്ക്ക് നൂറുകണക്കിനാളുകളുടെ സാക്ഷ്യമുണ്ട് എന്ന വസ്തുനിഷ്ഠ യാഥാര്‍ഥ്യത്തിനു നേരെ കണ്ണടയ്ക്കു വാന്‍ അവര്‍ക്ക് കഴിയുമോ?

ഏറെ വിമര്‍ശിക്കപ്പെട്ട അബൂഹുറയ്‌റ(റ)യുടെ കാര്യം തന്നെയെടുക്കുക. സത്യസന്ധരും വിശ്വസ്തരുമെന്ന് തെളിയിക്കപ്പെട്ട സ്വഹാബികളും താബിഉകളുമുള്‍പ്പെടുന്ന എണ്ണൂറോളം പേര്‍ അബൂഹുറയ്‌റ(റ)യില്‍ നിന്ന് ഹദീഥുകള്‍ നിവേദനം ചെയ്തിട്ടുണ്ട്.(6) ഈ എണ്ണൂറോളമാളുകളും നബി(സ)യുടെ പേരില്‍ കള്ളം പറയുന്നത് നരകപ്രവേശത്തിന് കാരണമാകുന്ന മഹാപാപ മാണെന്ന് വിശ്വസിക്കുന്നവരും ആയിരുന്നുവെന്നുറപ്പ്. അബൂഹുറയ്‌റ(റ)നബി(സ)യുടെ പേരില്‍ കളവു പറയുവാന്‍ വല്ല സാധ്യതയുമു ണ്ടെന്ന് അവര്‍ക്ക് തോന്നിയാല്‍ അവരിലൊരാള്‍പോലും അദ്ദേഹത്തില്‍നിന്ന് ഹദീഥുകള്‍ നിവേദനം ചെയ്യുമായിരുന്നില്ല. അബൂഹുറ യ്‌റ(റ)യെ കുറിച്ച് ഈ എണ്ണൂറോളം പേരുടെ സാക്ഷ്യം രേഖപ്പെടുത്തപ്പെട്ടതാണ്. ഇതിനെതിരെ അതിനെക്കാളധികം പേരുടെ സാക്ഷ്യമു ണ്ടെങ്കില്‍ മാത്രമെ അദ്ദേഹത്തിന്റെ സത്യസന്ധതയെ ചോദ്യം ചെയ്യാനായി പ്രസ്തുത സാക്ഷ്യത്തെ തെളിവായി സ്വീകരിക്കാനാവൂ.

സ്വഹാബിമാരിലാരെങ്കിലും അബൂഹുറയ്‌റ(റ)യുടെ സത്യസന്ധതയെ സംശയിച്ചിരുന്നതായി തെളിയിക്കുന്ന യാതൊരു രേഖയും ഉദ്ധരി ക്കുവാന്‍ അദ്ദേഹത്തിന്റെ വിമശകര്‍ക്ക് കഴിഞ്ഞിട്ടില്ല. അബൂഹുറയ്‌റ(റ)യുടെ സത്യസന്ധതയ്ക്ക് സാക്ഷ്യം വഹിക്കുന്ന എണ്ണൂറില ധികമാളുകളുടെ മൊഴിക്കെതിരായി സംസാരിക്കാനാകുന്ന സമകാലികനായ ഒരാളെപ്പോലും ഹാജരാക്കുവാന്‍ അവര്‍ക്ക് സാധിച്ചിട്ടി ല്ലാത്തതിനാല്‍ നൂറുകണക്കിന് സാക്ഷികളുടെ മൊഴി സ്വീകരിക്കുവാന്‍ വസ്തുനിഷ്ഠതയുടെ വക്താക്കളെന്ന് അവകാശപ്പെടുന്ന ചരിത്ര വിമര്‍ശനരീതിക്കാര്‍ നിര്‍ബന്ധിതരാണ്. അബൂഹുറയ്‌റ(റ)സത്യസന്ധനല്ലെന്ന് തെളിയിക്കുവാന്‍ ചരിത്ര വിമര്‍ശനരീതിയുടെ വക്താക്ക ളുടെ പക്കല്‍ കോപ്പുകളൊന്നുമില്ലെന്നര്‍ഥം. ഹദീഥുകള്‍ നിവേദനം ചെയ്ത മുഴുവന്‍ സ്വഹാബിമാരുടെയും സ്ഥിതി ഇതുതന്നെയാണ്. അവരുടെയെല്ലാം സത്യസന്ധതയ്ക്ക് നൂറുകണക്കിന് ആളുകളുടെ സാക്ഷ്യമുണ്ട്. തിരിച്ചാകട്ടെ, വിശ്വസ്തരും സമകാലികരുമായ ഒരാള്‍പോലും സാക്ഷ്യത്തിനില്ലതാനും!

ല്ല. നബിയെക്കുറിച്ച (സ) കൃത്യവും വസ്തുനിഷ്ഠവുമായ സംഭവവിവരണമാണ് ഹദീഥുകളിലുള്ളത്. ഹദീഥ് ശേഖരണത്തിൻെറയും ക്രോഡീകരണത്തിന്റെയും ത്യാഗപൂർണമായ ചരിത്രമറിയാത്തവരാണ് ഹദീഥുകളെക്കുറിച്ച് യാതൊരു ആദരവുമില്ലാതെ സംസാരിക്കുന്നത്. സ്വഹാബിമാരും താബിഉകളുമെല്ലാം വളരെ സൂക്ഷ്മതയോടുകൂടിയാണ് ഹദീഥുകൾ അടുത്ത തലമുറയ്ക്ക് കൈമാറിയത്. കള്ള ഹദീഥുകളിൽ നിന്ന് സമൂഹത്തെ രക്ഷിക്കുവാൻ അവർ പ്രത്യേകം ശ്രദ്ധിച്ചു. നബി (സ) യെ ക്കുറിച്ച് ഇല്ലാത്തതൊന്നും തങ്ങളുടെ നാവിലൂടെ പുറത്തുവരരുതെന്ന് അവർക്ക് നിർബന്ധമുണ്ടായിരുന്നു; അതോടൊപ്പം തന്നെ, നബിയിൽ (സ) നിന്നുള്ളതെന്ന രൂപത്തിൽ നബിയിൽ (സ) നിന്നുള്ളതല്ലാത്ത യാതൊന്നും സമൂഹത്തിൽ പ്രചരിക്കാതിരിക്കുവാനും അവർ ജാഗരൂകരായിരുന്നു. പ്രസ്തുത ജാഗ്രതയുടെ ഫലമായാണ് ഉസൂലുൽ ഹദീഥ് എന്ന ശാസ്ത്രം ഉണ്ടായിവന്നത്.

പ്രവാകാനുചരന്‍മാരില്‍ നിന്ന് മതം പഠിച്ച താബിഉകള്‍ക്കുശേഷം, അവരില്‍ നിന്ന് ഇസ്‌ലാമിക വിഷയങ്ങളെക്കുറിച്ച് മനസ്സിലാക്കിയ താബിഉത്താബിഉകളുടെ തലമുറയില്‍ ഹദീഥ് പഠന-ശേഖരണ രംഗത്ത് വമ്പിച്ച മുന്നേറ്റം തന്നെയുണ്ടായി. അതിനായി ജീവിതം ഉഴിഞ്ഞു വെച്ച ത്യാഗികളുടെ കാലമായിരുന്നു അത്. ഇസ്‌ലാമിക കര്‍മശാസ്ത്രവിഷയങ്ങള്‍ ക്രമരൂപത്തില്‍ നല്‍കികൊണ്ട് ക്രോഡീകരിക്കപ്പെട്ട ഗ്രന്ഥങ്ങളാണ് അന്നത്തെ പണ്ഡിതന്‍മാര്‍ രചിച്ചത്. ഓരോ വിഷയത്തെയും സംബന്ധിച്ച ഹദീഥുകള്‍ ആ വിഷയത്തെക്കുറിച്ച് പറയുന്ന തിനിടക്ക് ഉദ്ധരിക്കുകയായിരുന്നു അവര്‍ പൊതുവെ ചെയ്തിരുന്നത്. ഇത്തരം ഹദീഥ് ശേഖരങ്ങളെ മുസ്വന്നഫ് എന്നോ മുവത്വഅ് എന്നോ ആണ് വിളിക്കുന്നത്. ഇന്ന് ഉപലബ്ധമായ മുസന്നഫുകളില്‍ ഏറ്റവും പഴക്കമുള്ളത്, ഇമാം മാലിക്കുബ്‌നു അനസിന്റെ (ഹിജ്‌റ 93-179) അല്‍ മുവത്ത്വയാണ്. ഹിജ്‌റ 150ല്‍ മരണപ്പെട്ട ഇബ്‌നു തുറൈബിന്റെയും 161ല്‍ മരണപ്പെട്ട സുഫ്‌യാനു ഥ്ഥൗരിയുടെയും ഹിജ്‌റ 211ൽ മരണപ്പെട്ട അബ്ദുര്‍റസാഖ് അസ്വന്‍ആനിയുടെയും ഹിജ്‌റ235ല്‍ മരണപ്പെട്ട അബൂബക്കര്‍ ബ്ന്‍ അബീശൈബയുടെയും മുസ്വന്നഫുകൾ പ്രസിദ്ധങ്ങളാണ്.

കുഴപ്പങ്ങളില്‍നിന്നും വ്യതിയാനങ്ങളില്‍നിന്നും മുസ്‌ലിം ബഹുജനങ്ങളെ സംരക്ഷിച്ച് വിശുദ്ധ ക്വുര്‍ആനിലൂടെയും പ്രവാചകചര്യ യിലൂ ടെയും അവരെ നയിക്കുന്നതിനു വേണ്ടി വ്യാജ ഹദീഥുകളെയും യഥാര്‍ഥ നബിചര്യകളെയും വേര്‍തിരിച്ച് മനസ്സിലാക്കുവാനും രേഖപ്പെടു ത്തുവാനുമുള്ള ത്യാഗപൂര്‍ണമായ പണ്ഡിത പരിശ്രമത്തോടൊപ്പം തന്നെ, വാമൊഴിയായി ലഭിച്ച ഹദീഥുകളുടെ വെളിച്ചത്തില്‍ ദൈവിക മാര്‍ഗദര്‍ശനത്തിലൂടെ ജനങ്ങളെ നയിക്കുന്നതിനുവേണ്ടി അവര്‍ക്ക് മതവിധികള്‍ പറഞ്ഞുകൊടുക്കുന്നതിന്നായുള്ളപരിശ്രമങ്ങളുമുണ്ടാ യി. ഇതിന്‌വേണ്ടി പരിശ്രമിച്ച പ്രധാനപ്പെട്ട പണ്ഡിതമാരുടെ പേരില്‍ സ്ഥാപിക്കപ്പെട്ടതാണ് നാല് കര്‍മശാസ്ത്രധാരകളായ മദ്ഹബുകള്‍. തബഉത്താബിഉകളില്‍പെട്ട മദ്ഹബിന്റെ ഇമാമാണ് അബൂഹനീഫ നുഅ്മാനു ബ്‌നു ഥാബിത്. താബിഉകള്‍ക്കു ശേഷമുള്ള അടുത്ത തലമുറയില്‍-താബിഉത്താബിഉകള്‍ക്കിടയില്‍-ജീവിച്ച ഇമാം മാലിക്കുബ്‌നു അനസാണ് മദ്ഹബുകളുടെ ഇമാമായി ഗണിക്കപ്പെടുന്ന മറ്റൊരു മഹാപണ്ഡിതന്‍. മുസന്നഫുകളില്‍ ഏറ്റവും പഴക്കമുള്ള അല്‍മുവത്വയുടെ കര്‍ത്താവായ അദ്ദേഹം ഇമാം അബൂഹനീഫയുടെ സമകാലികരിലൊരാളായിരുന്നു.മാലിക്കുബ്‌നു അനസ്‌ന്റെ ശിഷ്യനായിരുന്ന അബൂഅബ്ദില്ലാ മുഹമ്മദ്ബ്‌നു ഇദ്‌രീസുശ്ശാഫിഈയാണ് മദ്ഹബിന്റെ ഇമാമായി അറിയപ്പെടുന്ന മൂന്നാമന്‍: നൂറോളം ഗ്രന്ഥങ്ങളെഴുതിയിട്ടുണ്ട് ഇമാം ശാഫിഈ. ഇമാം ശാഫിഈയുടെ ശിഷ്യനായിരുന്ന അഹ്മദ്ബ്ന്‍ മുഹമ്മദ് ബ്ന്‍ ഹന്‍ബല്‍ അബൂ അബ്ദുല്ലാ അശ്‌ശൈബാനിയാണ് നാലാമത്തെ മദ്ഹബിന്റെ ഇമാമായി അറിയപ്പെടുന്നത്.. മുസ്‌നദ്  അഹ്മദ് ബിന്‍ഹന്‍ബലാണ് അദ്ദേഹത്തിന്റെ ഏറ്റവും പ്രധാനപ്പെട്ടതും പ്രസിദ്ധവുമായ ഹദീഥ്‌ശേഖരം.(13)

പ്രവാചക നില്‍നിന്ന് ഹദീഥുകള്‍ നിവേദനം ചെയ്ത സ്വഹാബിമാരുടെ അടിസ്ഥാനത്തില്‍ ക്രോഡീകരിക്കപ്പെട്ട ഹദീഥ് ഗ്രന്ഥങ്ങളാണ് മുസ്‌നദുകള്‍ എന്നറിയപ്പെട്ടത്. ഓരോ പ്രത്യേക സ്വഹാബിയില്‍നിന്നും നിവേദനം ചെയ്യപ്പെട്ട ഹദീഥുകള്‍ പ്രത്യേക അധ്യായമായാണ് മുസ്‌നദുകളില്‍ ക്രോഡീകരിക്കപ്പെട്ടിരിക്കുന്നത്. ഇമാം ശാഫിഈയുടെ ശിഷ്യനും നാലാമത്തെ മദ്ഹബിന്റെ ഇമാമുമായ ഇമാം അഹ്മദ് ബ്ന്‍ ഹന്‍ബ ലിന്റെ ഹദീഥ് ശേഖരമാണ് മുസ്‌നദുകളില്‍ ഏറ്റവും പ്രധാനപ്പെട്ടതായി അറിയപ്പെടുന്നത്. വ്യാജ ഹദീഥുകള്‍ക്ക് ഹദീഥ് ഗ്രന്ഥങ്ങ ളില്‍ സ്ഥാനം കുറയാന്‍ മുസ്‌നദുകള്‍ നിമിത്തമായി. ഒരാള്‍ പ്രവാചകന്റെ പേരില്‍ വല്ലതും പറയുകയും അയാള്‍ക്ക് നബിയില്‍നിന്ന് അയാള്‍ വരെയുള്ള നിവേദകന്‍മാരുടെ ശൃംഖല അവതരിപ്പിക്കാന്‍ കഴിയാതിരിക്കുകയും ചെയ്താല്‍ അതിന്റെ സ്ഥാനം മുസ്‌നദുകളില്‍നിന്ന് സ്വാഭാവികമായും പുറത്തായിരിക്കും.

വിഷയക്രമത്തില്‍ ഹദീഥുകളും സഹാബിമാരുടെയും താബിഉകളുടെയും അഭിപ്രായങ്ങളും ക്രോഡീകരിച്ചുകൊണ്ടുള്ള മുസന്നഫുകള്‍ ക്കും പ്രവാചകരില്‍ നിന്നുള്ള പൂര്‍ണമായ ഇസ്‌നാദിന്റെ അടിസ്ഥാനത്തില്‍ ക്രോഡീകരിക്കപ്പെട്ട മുസ്‌നദുകളുടെയും നന്‍മകള്‍ സ്വാംശീക രിച്ചുകൊണ്ട് ഹിജ്‌റ മൂന്നാം നൂറ്റാണ്ടിന്റെ തുടക്കത്തില്‍ രചിക്കപ്പെട്ട ഹദീഥ് സമാഹാരങ്ങളാണ് 'സുനന്‍'എന്ന് അറിയപ്പെടുന്നത്. വിഷയ ക്രമത്തില്‍ ക്രോഡീകരിക്കപ്പെട്ടതും പൂര്‍ണമായ ഇസ്‌നാദോടുകൂടി ഉദ്ധരിക്കപ്പെട്ടതുമായ ഹദീഥുകളാണ് സുനന്‍ ഗ്രന്ഥങ്ങളില്‍ ക്രോഡീകരി ക്കപ്പെട്ടിരിക്കുന്നത്. ഹിജ്‌റ 227ല്‍ മരണപ്പെട്ട സഈദ്ബ്‌നു മന്‍സൂന്‍ അല്‍ഖുറാസാനിയും 255ല്‍ മരണപ്പെട്ട അബ്ദുല്ലാഹിബ്‌നു അബ്ദുറഹ്മാ ന്‍ അദ്ദാരിമിയുമാണ് ആദ്യകാല സുനനുകളുടെ കര്‍ത്താക്കള്‍.

ഹദീഥ് പഠനരംഗത്തെ നെല്ലും പതിരും വേര്‍തിരിച്ച് സംസ്‌കരിക്കുകയും പ്രവാചകനില്‍ നിന്നുള്ളതാണെന്ന് ഉറപ്പുള്ള ഹദീഥുകള്‍ മാത്രം ശേഖരിച്ച് മുസ്‌ലിംലോകത്തിന് നല്‍കുകയും ചെയ്ത മഹാ പ്രതിഭാശാലിയാണ് മുഹമ്മദ് ബ്ന്‍ ഇസ്മായീല്‍ അല്‍ ബുഖാരി (ഹിജ്‌റ 196-256). അദ്ദേഹത്തിന്റെ ത്യാഗപൂര്‍ണമായ പരിശ്രമങ്ങളുടെ ഫലമായാണ് സ്വഹീഹായ ഹദീഥുകളുടെ മാത്രമായുള്ള ഒന്നാമത്തെ സമാഹാരമായ സ്വഹീഹുല്‍ ബുഖാരി മുസ്‌ലിംലോകത്തിന് ലഭിച്ചത്. പതിനാറ് വര്‍ഷങ്ങള്‍ നീണ്ടുനിന്ന നിരന്തരമായ യാത്രകളിലൂടെ ഹദീഥുകളറിയാവുന്ന ആയിരത്തിലധികം പേരുമായി ആശയക്കൈമാറ്റം നടത്തിക്കൊണ്ട് അദ്ദേഹം ശേഖരിച്ച ഏഴു ലക്ഷത്തോളം ഹദീഥു കളില്‍നിന്ന് ഇസ്‌നാദ് പരിശോധിച്ച് ഉറപ്പു വരുത്തിയശേഷം 7397 ഹദീഥുകളെ മാത്രം ഉള്‍ക്കൊള്ളിച്ചുകൊണ്ടാണ് ബുഖാരി തന്റെ അല്‍ ജാമിഉ സ്‌സ്വഹീഹ് രചിച്ചത്. ഇതില്‍ തന്നെ പല ഹദീഥുകളും ഒരേ പ്രവാചകചര്യയുടെ തന്നെ വ്യത്യസ്ത രൂപത്തിലുള്ള ആവര്‍ത്തനങ്ങ ളാണ്. ആകെ 2602 പ്രവാചകവചനങ്ങള്‍ വ്യത്യസ്ത നിവേദകരിലൂടെ കടന്നുവന്നവയാണ് ബുഖാരിയിലുള്ള ഹദീഥുകളെന്ന് അതിന്റെ വ്യാഖ്യാതാവായ ഇബ്‌നു ഹജറുല്‍ അസ്ഖലാനി വ്യക്തമായിട്ടുണ്ട്.

അബുല്‍ ഹുസൈന്‍ മുസ്‌ലിമിബ്‌നുല്‍ ഹജ്ജാജ് അല്‍ നൈസാപൂരി (ഹി 202-261) ആണ് സ്വഹീഹായ ഹദീഥുകളെ മാത്രം ക്രോഡീകരിച്ചുകൊണ്ട് ഗ്രന്ഥരചന നടത്തിയ മറ്റൊരു മഹാവ്യക്തിത്വം. നാല്‍പത്തിമൂന്ന് അധ്യായങ്ങളിലായി 7563 ഹദീഥുകളാണ് അദ്ദേഹത്തിന്റെ സ്വഹീഹു മുസ്‌ലിമിലുള്ളത് ഇമാം ബുഖാരിയുടെയും ഇമാം മുസ്‌ലിംന്റെയും ശിഷ്യനായി രുന്ന അബൂബക്കര്‍ മുഹമ്മദ്ബ്‌നു ഇസ്ഹാക്വ്ബ്‌നു ഖുസൈമയും സ്വഹീഹായ ഹദീഥുകള്‍ മാത്രം ക്രോഡീകരിച്ചുകൊണ്ട് ഒരു ഗ്രന്ഥമെ ഴുതിയിട്ടുണ്ട്. സ്വഹീഹ് ഇബ്‌നു ഖുസൈമ എന്നാണ് അതിന്റെ പേര്.

ഇതിനുശേഷം പലരും ഹദീഥുകള്‍ ക്രോഡീകരിച്ച് ഗ്രന്ഥങ്ങളെഴുതിയെങ്കിലും മുസ്‌ലിം ലോകത്ത് പരക്കെ അറിയപ്പെട്ടത് ഇവയിലുള്ള പ്രധാനപ്പെട്ട നാല് ഹദീഥ് സമാഹാരങ്ങളാണ്. ഹിജ്‌റ 275ല്‍ അന്തരിച്ച ഇമാം അഹ്മദ് ബ്‌നു ഹന്‍ബലിന്റെ ശിഷ്യനായിരുന്ന അബൂദാവൂദ് സുലൈമാന്‍ ഇബ്‌നു അശ്അഥ് അസ്‌സിജിസ്താനി രചിച്ച സുനനു അബീദാവൂദ്, ഹിജ്‌റ 279ല്‍ അന്തരിച്ച, ഇമാം ബുഖാരിയുടെ ശിഷ്യ നായിരുന്ന അബൂഈസ മുഹമ്മദ് ബ്ന്‍ ഈസ അത്തിര്‍മിദി രചിച്ച അല്‍ജാമിഉത്തിര്‍മിദി, ഹിജ്‌റ 303ല്‍ അന്തരിച്ച, ഇമാം അബൂദാവൂ ദിന്റെ ശിഷ്യനായിരുന്ന അഹ്മദ്ബ്‌നു ശൂഐബ് അന്നസാഈ രചിച്ച സുനനുന്നസാഈ, ഹിജ്‌റ 273ല്‍ അന്തരിച്ച അബൂഅബ്ദില്ലാ മുഹ മ്മദ്ബ്‌നു യസീദുബ്‌നുമാജ രചിച്ച സുനനു ഇബ്‌നിമാജ(24) എന്നിവയാണീ ഗ്രന്ഥങ്ങള്‍. സുനനു അബൂദാവൂദില്‍ 4800 ഹദീഥുകളും ജാമി ഉത്തിര്‍മിദിയില്‍ 3950 ഹദീഥുകളും സുനനുന്നസാഇയില്‍ 5750 ഹദീഥുകളും സുനനു ഇബ്‌നുമാജയില്‍ 4485 ഹദീഥുകളുമാണുള്ളത്. കൃത്യ മായി പ്രവാചകനിലല്‍ നിന്ന് തുടങ്ങി ഗ്രന്ഥം ക്രോഡീകരിച്ചവരില്‍ അവസാനിക്കുന്ന വിശ്വസ്തരുടെ ശൃംഖലയായ ഇസ്‌നാദുള്ളവയല്ല ഈ നാല് ഹദീഥ് സമാഹാരങ്ങളിലെയും ചില ഹദീഥുകളെന്ന വസ്തുത അവയുടെ സമാഹര്‍ത്താക്കള്‍ തന്നെ സൂചിപ്പിച്ചിട്ടുണ്ട്. സഹീ ഹുല്‍ ബുഖാരിയിലെയും സഹീഹു മുസ്‌ലിമിലെയും ഹദീഥുകള്‍ മുസ്‌ലിംലോകം ചോദ്യം ചെയ്യാതെ സ്വീകരിക്കുമ്പോള്‍ മറ്റ് നാല് ഗ്രന്ഥ ങ്ങളിലെയും ഹദീഥുകള്‍ അവയുടെ ഇസ്‌നാദ് പരിശോധിച്ച ശേഷം അവ സ്വീകാര്യമാണെന്ന് ബോധ്യപ്പെട്ടതിനുശേഷം മാത്രമെ സ്വീകരിക്ക പ്പെടുകയുള്ളൂ.

മുഹമ്മദ് (സ) യുടെ ജീവിതത്തെക്കുറിച്ച് നമുക്ക് അറിവു നല്‍കുന്ന രണ്ടാമത്തെ സ്രോതസ്സായ ഹദീഥുകള്‍ എത്രത്തോളം കൃത്യവും സൂക്ഷ്മവുമായാണ് രേഖപ്പെടുത്തപ്പെട്ടതെന്ന് ഹദീഥ് നിദാനശാസ്ത്രത്തിന്റെ ഗ്രന്ഥങ്ങള്‍ പരിശോധിച്ചാല്‍ ബോധ്യമാകും. പ്രവാചക ന്‍(സ)യോടൊപ്പം സഹവസിച്ചവര്‍, തെറ്റുകളൊന്നും വരുത്താതെ, സൂക്ഷ്മവും കൃത്യവുമായി അടുത്ത തലമുറക്ക് പറഞ്ഞുകൊടുത്തതെ ന്ന് ഉറപ്പുള്ള നിവേദനം മാത്രമെ സ്വഹീഹായ ഹദീഥായി പരിഗണിക്കപ്പെടുകയുള്ളൂ. ഇത്രയ്ക്കും കൃത്യവും സൂക്ഷ്മവുമായി രേഖപ്പെ ടുത്തപ്പെട്ട മറ്റൊരു ജീവചരിത്രവുമില്ലെന്നതാണ് വാസ്തവം. ആധുനിക കാലത്തെ ചരിത്രരചനയില്‍ പോലും രചയിതാവിന്റെ വ്യക്തിത്വ ത്തെ വിമര്‍ശനവിധേയമാക്കി പറയുന്ന കാര്യങ്ങളുടെ യാഥാര്‍ഥ്യം മനസ്സിലാക്കുന്നതിനു വേണ്ടിയുള്ള സങ്കേതങ്ങള്‍ വേണ്ടവിധം വികസി പ്പിച്ചെടുക്കാന്‍ കഴിഞ്ഞിട്ടില്ല. ഒരേ വ്യക്തിയുടെ ജീവിതത്തെ രണ്ടു രൂപത്തില്‍ നോക്കിക്കാണുന്നവര്‍ എഴുതിയ ചരിത്ര ഗ്രന്ഥങ്ങളിലെ പരാമര്‍ശങ്ങളുടെ സത്യത പരിശോധിക്കുവാന്‍ നമ്മുടെ പക്കല്‍ കാര്യമാത്രപ്രസക്തമായ മാനദണ്ഡങ്ങളൊന്നുമില്ല.

നബി(സ) യോടൊപ്പം ജീവിക്കുകയും അദ്ദേഹത്തിന്റെ ജീവിതം നേര്‍ക്കുനേരെ മനസ്സിലാക്കുകയും അത് രേഖപ്പെടുത്തുകയോ മറ്റുള്ളവര്‍ക്ക് പറഞ്ഞുകൊടു ക്കുകയോ ചെയ്യുമ്പോള്‍ അബദ്ധങ്ങളോ അസത്യങ്ങളോ കടന്നുകൂടാതിരിക്കുവാന്‍ സൂക്ഷ്മത പ്രകടിപ്പിക്കുകയും ചെയ്തവരില്‍നിന്ന് നിവേദനം ചെയ്യപ്പെട്ട നബിചരിത്രമാണ് ഹദീഥുകളിലുള്ളത്. നബി(സ) യുടെ അകവും പുറവും മനസ്സിലാക്കിയവരുടെ നേര്‍ക്കുനേരെയുള്ള ചിത്രീകരണം. ആ രൂപത്തില്‍ ഒരാളുടെയും ചരിത്രം രേഖപ്പെടുത്തപ്പെട്ടിട്ടില്ല. ആത്മകഥയ്ക്ക്‌പോലും ഇത്രയ്ക്ക് സൂക്ഷ്മമായ ഒരു ജീവിതാഖ്യാനം നടത്താന്‍ കഴിയില്ല. സ്വന്തത്തിന്റെ കുറവുകള്‍ കാണാന്‍ ആത്മകഥാകാരന് കഴിയില്ലല്ലോ. ഒരു ലക്ഷത്തിലധികം പേരുടെ ദൃക്‌സാക്ഷി വിവരണത്തിന്റെ സാക്ഷ്യമാണ് സ്വഹീഹായ ഹദീഥുകള്‍ക്കുള്ളത്. നബി(സ)  മരണപ്പെടുമ്പോള്‍ ജീവിച്ചിരുന്ന സ്വഹാബിമാരുടെ എണ്ണം ഒരു ലക്ഷത്തിലധികമായിരുന്നല്ലോ.

ക്വുര്‍ആന്‍ അവതരിപ്പിക്കപ്പെടുന്ന മുറയ്ക്ക് എഴുതി സൂക്ഷിക്കാറുണ്ടായിരുന്നതുപോലെ നബി വചനങ്ങളോ കര്‍മങ്ങളോ എഴുതി സൂക്ഷിക്കുന്ന പതിവ് മുഹമ്മദ് നബി(സ)യുടെ ജീവിതകാലത്ത് ഉണ്ടായിരുന്നില്ല എന്നത് ശരിയാണ്. എന്നാല്‍ ചില സ്വഹാബികള്‍ നബി(സ)യുടെ വചനങ്ങള്‍ എഴുതിവെക്കുകയും സൂക്ഷിക്കുകയും ചെയ്തിരുന്നതായി രേഖകളുണ്ട്. ഖുര്‍ആന്‍ വചനങ്ങളും ഹദീഥുകളും തമ്മില്‍ കൂടിക്കലരരുതെ ന്ന് നിര്‍ബന്ധമുള്ളതിനാല്‍ 'ക്വുര്‍ആനല്ലാത്ത മറ്റൊന്നുംതന്നെ തന്നില്‍നിന്ന് എഴുതി സൂക്ഷിക്കരുതെന്ന് ആദ്യകാലത്ത് നബി(സ) വിലക്കിയി രുന്നു'(അബൂസഈദുല്‍ ഖുദ്‌രിയില്‍ നിന്ന് മുസ്‌ലിം ഉദ്ധരിച്ചത്) വെങ്കിലും പ്രത്യേക സന്ദര്‍ഭങ്ങളില്‍ അങ്ങനെ ചെയ്യാന്‍ നിര്‍ദേശിച്ചിരുന്നതായും കാണാന്‍ കഴിയും. മക്കാവിജയകാലത്ത് മക്കയുടെ പവിത്രതയെക്കുറിച്ച് നബി(സ) നടത്തിയ ഒരു പ്രഭാഷണം കഴിഞ്ഞപ്പോള്‍ അത് തനിക്ക് എഴുതിത്തരണമെന്ന് യമന്‍കാരനായ അബൂശാഹ് ആവശ്യപ്പെട്ടതായും അദ്ദേഹത്തിന് അത് എഴുതിക്കൊടുക്കുവാന്‍ പ്രവാചകന്‍ല നിര്‍ദേശിച്ചതായും ബുഖാരിയും മുസ്‌ലിമും രേഖപ്പെടുത്തിയിട്ടുണ്ട്. പ്രവാചകശിഷ്യനായിരുന്ന അബ്ദുല്ലാഹിബ്‌നു അംറുബ്‌നുല്‍ ആസ്വ്(റ) , ഹദീഥുകള്‍ എഴുതി സൂക്ഷി ച്ചിരുന്നതായി അബൂ ഹുറൈറ സാക്ഷ്യപ്പെടുത്തുന്ന ഹദീഥ് ബുഖാരിയിലുണ്ട്. തനിക്ക് ഹദീഥുകള്‍ എഴുതി സൂക്ഷിക്കുവാന്‍ പ്രവാച കന്‍(സ) അനുവാദം നല്‍കിയതായി അബ്ദുല്ലാഹിബ്‌നു അംറ്‌ (റ) അവകാശപ്പെട്ടതായി അഹ്മദും അബൂദാവൂദും ഉദ്ധരിച്ചിട്ടുണ്ട്.

നബി ജീവിതത്തെക്കുറിച്ച് തങ്ങള്‍ക്കറിയാവുന്ന കാര്യങ്ങള്‍ സ്വഹാബിമാരില്‍ ചിലര്‍ എഴുതി സൂക്ഷിച്ചിരുന്നുവെങ്കിലും അത് വ്യാപകമായി രുന്നില്ല. തങ്ങള്‍ നേര്‍ക്കുനേരെ കണ്ട നബിജീവിതത്തിന്റെ വ്യത്യസ്ത വശങ്ങളെപ്പറ്റി അവര്‍ മറ്റുള്ളവര്‍ക്ക് പറഞ്ഞുകൊടുക്കുന്ന രീതിയാ യിരുന്നു വ്യാപകമായി നിലനിന്നിരുന്നത്. വാമൊഴിയായാണ് പ്രധാനമായും നബിജീവിതത്തെ കുറിച്ച വര്‍ത്തമാനങ്ങള്‍ കൈമാറ്റം ചെയ്യപ്പെട്ടതെന്ന് സാരം.

രാഷ്ട്രീയവും സൈദ്ധാന്തികവുമായ ആവശ്യങ്ങള്‍ക്കുവേണ്ടി വ്യാജഹദീഥുകള്‍ നിര്‍മിക്കപ്പെടുന്ന അവസ്ഥയുണ്ടായപ്പോള്‍ അതിനെതിരെ വിശ്വാസീസമൂഹം ജാഗരൂകരായി. രണ്ടാം ഖലീഫ ഉമര്‍ (റ) തന്റെ ഭരണകാലത്ത് ഹദീഥുകള്‍ ശേഖരിച്ച് ക്രോഡീകരിക്കുവാന്‍ ആഗ്രഹി ച്ചെങ്കിലും ക്വുര്‍ആന്‍ വചനങ്ങളും ഹദീഥുകളും തമ്മില്‍ കൂടിക്കലര്‍ന്നു പോകുമോയെന്ന ഭയം കാരണം അത് ഉപേക്ഷിച്ചതായി മുഹമ്മദ് ബ്‌നു സഅദ് രേഖപ്പെടുത്തുന്നുണ്ട്. എന്നാല്‍ ഈ രംഗത്ത് ക്രിയാത്മകമായ ഒരു ഇടപെടല്‍ നടത്തിയത് രണ്ടാം ഉമര്‍ എന്നറിയപ്പെടുന്ന ഉമറുബ്‌നു അബ്ദുല്‍ അസീസ്‌ (റ) ആണ്. താബിഉകളില്‍പ്പെട്ട സുപ്രസിദ്ധനായ ഭരണാധികാരിയായിരുന്ന അദ്ദേഹത്തിന്റെ കാലമായപ്പോ ഴേക്ക് വ്യാജ ഹദീഥുകളുടെ നിര്‍മാണം വ്യാപകമായിക്കഴിഞ്ഞിരുന്നു. മദീനയിലെ അദ്ദേഹത്തിന്റെ ന്യായാധിപനായിരുന്ന അബൂബക്കര്‍ ബിനു ഹസമിന് അദ്ദേഹം എഴുതി: 'ദൈവദൂതരില്‍നിന്നുള്ള ഹദീഥുകള്‍ താങ്കള്‍ നോക്കുകയും എഴുതി രേഖപ്പെടുത്തുകയും ചെയ്യണം. കാരണം അറിവ് തേഞ്ഞുമാഞ്ഞു പോകുന്നതും ജ്ഞാനികള്‍ കാലംകഴിഞ്ഞു പോകുന്നതും ഞാന്‍ ഭയപ്പെടുന്നു. അല്ലാഹുവിന്റെ ദൂതരില്‍ നിന്നുള്ള ഹദീഥുകളല്ലാതെ മറ്റൊന്നും സ്വീകരിക്കരുത്. അറിവ് പകര്‍ന്നുകൊടുക്കുകയും അറിവില്ലാത്തവരെ പഠിപ്പിക്കുകയും ചെയ്യുക; ജ്ഞാനം എല്ലാവരും രഹസ്യമാക്കുമ്പോഴല്ലാതെ നശിക്കുകയില്ല'. ഉമര്‍ ബ്‌നു അബ്ദുല്‍ അസീസ്‌ന്റെ നിര്‍ദേശപ്രകാരം മദീനയിലെ സ്വഹാ ബികളില്‍ നിന്നും താബിഉകളില്‍നിന്നും അബൂബക്കര്‍ ബ്‌നു ഹസം (റഹ്) ഹദീഥുകള്‍ ശേഖരിച്ചു. അന്നു ജീവിച്ചിരുന്ന മഹാപണ്ഡിതനാ യിരുന്ന മുഹമ്മദ്ബ്‌നു മുസ്‌ലിബിനു ശിഹാബ് അസ്‌സുഹ്‌രിയും രണ്ടാം ഉമറിന്റെ ഭരണകാലത്ത് ഹദീഥുകള്‍ ശേഖരിക്കുകയും ക്രോഡീ കരിക്കുകയും ചെയ്യുവാന്‍ മുന്നോട്ടുവന്നു. ഇതോടൊപ്പം തന്നെ, ഇസ്‌ലാമികരാഷ്ട്രത്തിന്റെ വ്യത്യസ്ത കോണുകളിലേക്ക് ഹദീഥുകള്‍ ശേഖരിക്കുവാന്‍ ആവശ്യപ്പെട്ടുകൊണ്ട് ഉമറുബ്‌നു അബ്ദുല്‍ അസീസ് കത്തുകളയിച്ചിരുന്നുവെന്ന് അബൂനുഐമിന്റെ താരിഖുല്‍ ഇസ്ബ ഹാനില്‍ നിന്ന് ഇബ്‌നുഹജറുല്‍ അസ്ഖലാനി ഉദ്ധരിക്കുന്നുണ്ട്.(ഫത്ഹുല്‍ബാരി, വാല്യം 1, കിത്താബുല്‍ ഇല്‍മ്) ഇങ്ങനെ ശേഖരിക്കപ്പെട്ട ഹദീഥുകള്‍ ക്രോഡീകരിച്ചു രേഖപ്പെടുത്തിയത് ഇമാം സുഹ്‌രിയായിരുന്നു. അതിനുശേഷം വ്യത്യസ്ത ദേശക്കാരായ പല താബിഉകളും ഹദീഥുകള്‍ ശേഖരിക്കുവാന്‍ തുടങ്ങി. ഹിജ്‌റ 150ല്‍ അന്തരിച്ച അബ്ദുല്‍ മലിക്കു ബ്‌നു അബ്ദുല്‍ അസീസ് ബ്‌നു ജുറൈജ് മക്കയിലും ഹിജ്‌റ 157ല്‍ അന്തരിച്ച സഈദ്ബിനു അബിഅറൂബ മെസപ്പെട്ടോമിയയിലും ഹിജ്‌റ 159ല്‍ അന്തരിച്ച അബൂഅംറില്‍ ഔസാഈ സിറിയയിലും ഹിജ്‌റ 159ല്‍ തന്നെ അന്തരിച്ച മുഹമ്മദ് ബ്‌നു അബ്ദിര്‍ റഹ്മാന്‍ മദീനയിലും ഹിജ്‌റ 160ല്‍ അന്തരിച്ച സൈദ് ബ്‌നുക്വുദാമയും സുഫ്‌യാനുഥൗരിയും കൂഫയിലും ഹിജ്‌റ 165ല്‍ അന്തരിച്ച ഹമ്മാദ് ബ്‌നു സലമ ബസറയിലും വെച്ച് ഹദീഥുകള്‍ ശേഖരിക്കുകയും രേഖപ്പെടുത്തുകയും ചെയ്തതായി രേഖകളുണ്ട്.(ഇബ്‌നുല്‍ നദീമിന്റെ അല്‍ ഫിഹിരിസ്തില്‍ നിന്ന് )

സ്വഹാബിമാരും താബിഉകളുമെല്ലാം ഹദീഥുകൾ രേഖപ്പെടുത്തി സൂക്ഷിച്ചിരുന്നുവെന്ന് ഇവ വ്യക്തമാക്കുന്നുണ്ട്. എന്നാൽ അവർക്കു ശേഷം മൂന്നാം തലമുറ മുതൽക്കാണ് ഹദീഥ് രേഖീകരണം വ്യാപകമായി ആരംഭിച്ചത് . ഹദീഥ് നിദാനശാസ്ത്രം വളർച്ച പ്രാപിച്ചതും അക്കാലത്ത് തന്നെയായിരുന്നു

തീർച്ചയായും. ദൈവികവചനങ്ങളെ പ്രായോഗികമാക്കേണ്ടതെങ്ങനെയെന്ന് പഠിപ്പിക്കുവാൻ വേണ്ടി നിയോഗിക്കപ്പെട്ട മുഹമ്മദ്‌നബി(സ) കേവലമൊരു ഉപദേശിയായിരുന്നില്ല . താന്‍ ഉപദേശിക്കുന്ന കാര്യങ്ങളെല്ലാം അദ്ദേഹത്തിന്റെ ജീവിതത്തില്‍ പ്രാവര്‍ത്തികമായിക്കാണാന്‍ അദ്ദേഹത്തിന്റെ അനുയായികൾക്ക് കഴിഞ്ഞിരുന്നു. സാധാരണക്കാരായ സ്വഹാബിമാരോടൊപ്പം കേവലമൊരു സാധാരണക്കാരനെ പ്പോലെയായിരുന്നു അദ്ദേഹത്തിന്റെ ജീവിതം. നാട്ടിലും വീട്ടിലും പള്ളിയിലും അങ്ങാടിയിലും യാത്രയിലും വിശ്രമത്തിലുമെല്ലാം അനു യായികള്‍ അദ്ദേഹത്തോടൊപ്പമുണ്ടായിരുന്നു. അവര്‍ അദ്ദേഹത്തിന്റെ വാക്കുകള്‍ ശ്രവിക്കുകയും ഓര്‍മയില്‍ കുറിച്ചിടുകയും ചെയ്തു; അദ്ദേഹത്തിന്റെ പ്രാര്‍ഥനകള്‍ ശ്രദ്ധിക്കുകയും അവ അതേപോലെത്തന്നെ പിന്‍തുടരുകയും ചെയ്തു; ജീവിതവ്യവഹാരങ്ങളും നിലപാ ടുകളും സ്വഭാവങ്ങളും കൊള്ളക്കൊടുക്കലുകളും സൂക്ഷ്മമായി നിരീക്ഷിക്കുകയും അവ അനുധാവനം ചെയ്യാന്‍ പരമാവധി പരിശ്രമി ക്കുകയും ചെയ്തു. സ്വഹാബിമാരുടെയെല്ലാം ആത്മാര്‍ഥമായ പരിശ്രമമായിരുന്നു അത്. നബി(സ)യെ അനുകരിക്കുവാന്‍ അവര്‍ ആഗ്ര ഹിച്ചു; അക്കാര്യത്തിലായിരുന്നു അനുചരന്‍മാരുടെ ശ്രദ്ധ. അതുകൊണ്ടുതന്നെ പരമാവധി സമയം നബി(സ)യോടൊപ്പമുണ്ടാകണമെന്ന് അവര്‍ സ്വയം നിഷ്‌കര്‍ഷിച്ചു. തങ്ങള്‍ നബി(സ)യോടൊപ്പമില്ലാത്തപ്പോള്‍ അദ്ദേഹം എന്തൊക്കെയാണ് ചെയ്തതെന്നും പറഞ്ഞതെന്നും അവര്‍ മറ്റുള്ളവരോട് അന്വേഷിച്ചു പഠിച്ചു. നബി(സ)യെ നിരീക്ഷിക്കുവാന്‍ അവര്‍ ഊഴം നിശ്ചയിച്ചു.

വ്യത്യസ്തങ്ങളായ ജീവിതപ്രശ്‌നങ്ങളില്‍ ദൈവികവിധിയെന്താണെന്നറിയാനും, അവ പ്രയോഗവല്‍ക്കരിക്കുവാനും ഉത്‌സുകരായിരുന്നു പ്രവാചകാനുചരന്‍മാര്‍. ധര്‍മാധര്‍മങ്ങളുടെ കാര്യങ്ങളിലൊന്നും അവര്‍ സ്വന്തമായ തീരുമാനങ്ങളെടുത്തില്ല; പ്രവാചകനായിരുന്നു എല്ലാ കാര്യങ്ങളിലുമുള്ള അവരുടെ മാര്‍ഗദര്‍ശി. അദ്ദേഹത്തോട് ചോദിച്ചറിയുകയും അദ്ദേഹം ശരിയെന്ന് വിധിച്ചത് പ്രാവര്‍ത്തികമാക്കു കയും ചെയ്യുകയായിരുന്നു സ്വഹാബിമാരുടെ രീതി. തങ്ങള്‍ക്ക് അറിയാത്ത കാര്യങ്ങളിലുള്ള പ്രവാചക നിര്‍ദേശം ലഭിക്കുന്നതിനായി നാഴികകള്‍ യാത്ര ചെയ്യുവാന്‍ അവര്‍ക്ക് മടിയുണ്ടായിരുന്നില്ല. ത്യാഗങ്ങള്‍ സഹിച്ചുകൊണ്ടാണെങ്കിലും കൃത്യമായ ദൈവിക മാര്‍ഗനി ര്‍ദേശമെന്തെന്ന് മനസ്സിലാക്കിയ ശേഷം മാത്രം അത് പ്രയോഗവല്‍ക്കരിക്കണമെന്ന് അവര്‍ക്ക് നിര്‍ബന്ധമുണ്ടായിരുന്നു. ഉമര്‍ (റ) പറയുന്നു: ''ഞാനും, ഉമയ്യത്തുബ്‌നുസൈദിന്റെ സന്തതികളില്‍പ്പെട്ട എന്റെ ഒരു അയല്‍ക്കാരനും (അയാള്‍ മേലേ മദീനയിലായിരുന്നു) റസൂല്‍ തിരുമേനിലയുടെ അടുക്കല്‍ ചെല്ലുന്നതിന് ഊഴം നിശ്ചയിച്ചിരുന്നു. ഒരു ദിവസം അദ്ദേഹം ചെല്ലും, ഒരു ദിവസം ഞാന്‍ ചെല്ലും. ഞാന്‍ പോകുമ്പോള്‍ അന്നത്തെ വര്‍ത്തമാനം ഞാന്‍ അദ്ദേഹത്തിന്നു പറഞ്ഞുകൊടുക്കും. അദ്ദേഹം പോകുമ്പോള്‍ അദ്ദേഹവും അങ്ങിനെ ചെയ്യും.''(സ്വഹീഹുല്‍ ബുഖാരി, കിതാബുല്‍ ഇല്‍മ്)

പ്രവാചകന്റെ (സ) നാവിൽ നിന്ന് സ്വഹാബിമാർ ദൈവവചനങ്ങള്‍ ശ്രവിക്കുകയും, ജീവിതത്തില്‍ നിന്ന് അവ എങ്ങനെ പ്രയോഗവല്‍ക്കരിക്കണമെന്ന് പഠിക്കുകയും ചെയ്തു. ധര്‍മാധര്‍മങ്ങളെ വ്യവഛേദിക്കുന്നതിനുള്ള അവരുടെ മാനദണ്ഡം നബി(സ)യുടെ വാക്കും പ്രവൃത്തിയും അനുവാദവുമായിരുന്നു. അത് അവര്‍ പഠിക്കുകയും മറ്റുള്ളവര്‍ക്ക് പറഞ്ഞു കൊടുക്കുകയും ചെയ്തു. അദ്ദേഹത്തോടൊപ്പം കൂടുതല്‍ നേരം സഹവസിച്ചവരില്‍നിന്ന് മറ്റുള്ളവര്‍ നബിജീവിതത്തിന്റെ സൂക്ഷ്മാംശങ്ങള്‍ ചോദിച്ച റിഞ്ഞു. കുടുംബ-ലൈംഗിക ജീവിതങ്ങളില്‍ നബിമാതൃകയെപ്പറ്റി അവര്‍ അദ്ദേഹത്തിന്റെ പത്‌നിമാരില്‍നിന്നാണ് പഠിച്ചത്. യാത്രകളില്‍ നബി(സ)യോടൊപ്പമുണ്ടായിരുന്നവരോട് ചോദിച്ച് ഒപ്പമില്ലാത്തവര്‍ യാത്രാമര്യാദകളെക്കുറിച്ച് മനസ്സിലാക്കി. ഈ വിവര സംപ്രേഷണ ത്തില്‍ അവരെല്ലാം വളരെ സൂക്ഷ്മത പാലിച്ചു; അതില്‍ കളവോ അബദ്ധമോ കടന്നുകൂടാതിരിക്കാന്‍ അവര്‍ പ്രത്യേകം ശ്രദ്ധിച്ചു. ഓര്‍മയി ല്ലാത്തതൊന്നും അവര്‍ മറ്റുള്ളവരോട് പറഞ്ഞില്ല. നബിജീവിതത്തെപ്പറ്റി തങ്ങളുടെ നാവുകളില്‍നിന്ന് അബദ്ധങ്ങളൊന്നും പുറത്തുവരരു തെന്ന് അവര്‍ക്ക് നിര്‍ബന്ധമുണ്ടായിരുന്നു. അദ്ദേഹത്തെക്കുറിച്ച് കള്ളംപറയുന്നവരുടെ ഇരിപ്പിടം നിത്യനരകമായിരിക്കുമെന്ന് പഠിപ്പിക്ക പ്പെട്ടവരായിരുന്നു അവര്‍. ''എന്നെക്കുറിച്ച് ബോധപൂര്‍വം ആരെങ്കിലും നുണപറയുകയാണെങ്കില്‍ അയാള്‍ നരകത്തില്‍ ഒരു ഇരിപ്പിടം തയാറാക്കിക്കൊള്ളട്ടെ.''(സ്വഹീഹ് മുസ്‌ലിം. മുഖദ്ദിമയില്‍ അബൂഹുറയ്‌റയില്‍ നിന്ന് ഉദ്ധരിച്ചത്.)

സ്വന്തം മാതൃകാജീവിതം അനുയായികള്‍ക്കു മുമ്പില്‍ സമര്‍പ്പിക്കുകയും അത് അപ്പടി അനുകരിക്കുവാന്‍ അവരെ പ്രചോദിപ്പിക്കുകയും ചെയ്ത മുഹമ്മദ് നബി(സ), തന്റെ കാലശേഷം അവര്‍ സ്വീകരിക്കേണ്ട നിലപാട് എന്തായിരിക്കണമെന്നു കൂടി പഠിപ്പിച്ചുകൊണ്ടാണ് ഈ ലോകത്തുനിന്നു യാത്രയായത്. റസൂൽ(സ) പറഞ്ഞതായി മാലിക്‌ (റ) നിവേദനം ചെയ്ത മുവത്വയിലുളള ഒരു ഹദീഥില്‍ ഇങ്ങനെ കാണാം. ''ഞാന്‍ നിങ്ങളില്‍ രണ്ടു കാര്യങ്ങള്‍ വിട്ടുപോകുന്നു. അവ നിങ്ങള്‍ മുറുകെപിടിച്ചാല്‍ നിങ്ങള്‍ വഴിപിഴക്കുന്നതേയല്ല. അല്ലാഹുവിന്റെ കിത്താബും, എന്റെ സുന്നത്തുമാണവ.''

മുഹമ്മദ് നബി(സ)യുടെ വിയോഗത്തിനുശേഷം അദ്ദേഹം വിട്ടേച്ചുപോയ ചര്യ മുറുകെപിടിക്കുവാന്‍ സ്വഹാബിമാര്‍ ശ്രദ്ധിച്ചു. അതില്‍ നിന്ന് അല്‍പം പോലും തെറ്റിപോകാതിരിക്കുവാന്‍ അവര്‍ സൂക്ഷ്മത പാലിച്ചു. നബിജീവിതത്തിന്റെ അവസാനനാളുകളില്‍ ഇസ്‌ലാമി ലേക്ക് കടന്നുവന്നവരെ ഇക്കാര്യത്തില്‍ ദീര്‍ഘനാള്‍ നബി(സ)യോടൊപ്പം ജീവിച്ചവര്‍ സഹായിച്ചു. നബിയുടെ വാക്കിന്റെയോ പ്രവൃത്തി യുടെയോ അനുവാദത്തിന്റെയോ അംഗീകാരമില്ലാത്ത പ്രവര്‍ത്തനങ്ങളെന്തെങ്കിലും ആരെങ്കിലും ചെയ്യുന്നത് കണ്ടാല്‍ ശക്തമായ ഭാഷയി ല്‍തന്നെ സ്വഹാബിമാര്‍ അവരെ തിരുത്തി.

നബി(സ)യുടെ പിന്‍ഗാമികളായിവന്ന ഭരണാധികാരികളായ അബൂബക്കറിന്റെയും ഉമറിന്റെയും (റ) ഭരണകാലത്ത് ഭരണീയരായി ഉണ്ടായിരുന്നവരില്‍ ഭൂരിഭാഗവും മുഹമ്മദ് നബി(സ)യെ നേരിട്ട് കാണുകയും അദ്ദേഹത്തില്‍നിന്ന് മതം പഠിക്കുകയും ചെയ്തവരായി രുന്നു. പ്രവാചകാനുചരന്‍മാരില്‍ പ്രമുഖരെല്ലാം അന്ന് മദീനയിലും പരിസരപ്രദേശങ്ങളിലുമായിരുന്നു വസിച്ചിരുന്നത്. നബി(സ)യില്‍ നിന്ന് ഓരോരുത്തരും പഠിച്ചറിഞ്ഞ കാര്യങ്ങള്‍ പരസ്പരം ഉപദേശിക്കുകയും അവ പ്രാവര്‍ത്തികമാക്കുവാന്‍ പരമാവധി പരിശ്രമിക്കു കയും ചെയ്തു, അവര്‍. മൂന്നാം ഖലീഫയായ ഉഥ്മാന്‍െ(റ) ന്റ ഭരണകാലത്ത് ഇസ്‌ലാമിക രാഷ്ട്രത്തിന്റെ വിസ്തൃതി വര്‍ധിച്ചു. സ്വഹാ ബിമാര്‍ക്ക് വ്യത്യസ്തങ്ങളായ പട്ടണങ്ങളിലേക്കും ഗ്രാമങ്ങളിലേക്കും യാത്ര ചെയ്യേണ്ടിവരികയും അവിടെ അവരില്‍ ചിലര്‍  താമസമാ ക്കുകയും ചെയ്തു. അതോടൊപ്പം തന്നെ, വയോധികരായ പ്രവാചകാനുചരന്‍മാരുടെ എണ്ണം കുറഞ്ഞുകൊണ്ടുമിരുന്നു. അത്തരക്കാ രുമായി ബന്ധപ്പെട്ട് നബിമാതൃകയെക്കുറിച്ച് അറിവ് സമ്പാദിക്കുവാന്‍ ചെറുപ്പക്കാരായ സ്വഹാബിമാര്‍ പരിശ്രമിച്ചു. ദീര്‍ഘദൂരം യാത്രക ള്‍ ചെയ്തും ത്യാഗങ്ങള്‍ സഹിച്ചും നബിമാതൃകയെക്കുറിച്ച് പഠിക്കുവാനും ഓരോ വിഷയങ്ങളിലുമുള്ള നബി(സ)യുടെ നിര്‍ദേശങ്ങളെ ന്തെന്ന് ശേഖരിക്കുവാനും അവര്‍ സന്നദ്ധമായി. എത്ര ത്യാഗങ്ങള്‍ സഹിച്ചാണെങ്കിലും കുറ്റമറ്റ രീതിയില്‍ നബിചര്യ മനസ്സിലാക്കുകയും ശേഖരിക്കുകയും ചെയ്യേണ്ടതുണ്ടെന്നായിരുന്നു അവരുടെ നിലപാട്.